137
Linear Algebra Homan & Kunze 2nd edition Answers and Solutions to Problems and Exercises Typos, comments and etc... Gregory R. Grant University of Pennsylvania email: [email protected] Julyl 2017

Linear Algebra - Greg Grantgreggrant.org/hoffman_and_kunze.pdf · Linear Algebra Ho man & Kunze 2nd edition ... Section 1.2: Systems of Linear Equations 3 Solution: We must check

Embed Size (px)

Citation preview

Page 1: Linear Algebra - Greg Grantgreggrant.org/hoffman_and_kunze.pdf · Linear Algebra Ho man & Kunze 2nd edition ... Section 1.2: Systems of Linear Equations 3 Solution: We must check

Linear AlgebraHoffman & Kunze

2nd edition

Answers and Solutions to Problems and ExercisesTypos, comments and etc...

Gregory R. GrantUniversity of Pennsylvaniaemail: [email protected]

Julyl 2017

Page 2: Linear Algebra - Greg Grantgreggrant.org/hoffman_and_kunze.pdf · Linear Algebra Ho man & Kunze 2nd edition ... Section 1.2: Systems of Linear Equations 3 Solution: We must check

2

Note

This is one of the ultimate classic textbooks in mathematics. It will probably be read for generations to come. Yet Icannot find a comprehensive set of solutions online. Nor can I find lists of typos. Since I’m going through this book in somedetail I figured I’d commit some of my thoughts and solutions to the public domain so others may have an easier time than Ihave finding supporting matericals for this book. Any book this classic should have such supporting materials readily avaiable.

If you find any mistakes in these notes, please do let me know at one of these email addresses:

[email protected] or [email protected] or [email protected]

The latex source file for this document is available at http://greg.grant.org/hoffman and kunze.tex. Use it wisely.And if you cannot manage to download the link because you included the period at the end of the sentence, then maybe youare not smart enough to be reading this book in the first place.

Page 3: Linear Algebra - Greg Grantgreggrant.org/hoffman_and_kunze.pdf · Linear Algebra Ho man & Kunze 2nd edition ... Section 1.2: Systems of Linear Equations 3 Solution: We must check

Chapter 1: Linear Equations

Section 1.1: FieldsPage 3. Hoffman and Kunze comment that the term “characteristic zero” is “strange.” But the characteristic is the smallest nsuch that n · 1 = 0. In a characteristic zero field the smallest such n is 0. This must be why they use the term “characteristiczero” and it doesn’t seem that strange.

Section 1.2: Systems of Linear EquationsPage 5 Clarification: In Exercise 6 of this section they ask us to show, in the special case of two equations and two unknowns,that two homogeneous linear systems have the exact same solutions then they have the same row-reduced echelon form (weknow the converse is always true by Theorem 3, page 7). Later in Exercise 10 of section 1.4 they ask us to prove it whenthere are two equations and three unknowns. But they never tell us whether this is true in general (for abitrary numbers ofunknowns and equations). In fact is is true in general. This explanation was given on math.stackexchange:

Solutions to the homogeneous system associated with a matrix is the same as determining the null space of therelevant matrix. The row space of a matrix is complementary to the null space. This is true not only for innerproduct spaces, and can be proved using the theory of non-degenerate symmetric bilinear forms.

So if two matrices of the same order have exactly the same null space, they must also have exactly the same rowspace. In the row reduced echelon form the nonzero rows form a basis for the row space of the original matrix,and hence two matrices with the same row space will have the same row reduced echelon form.

Exercise 1: Verify that the set of complex numbers described in Example 4 is a subfield of C.

Solution: Let F = {x + y√

2 | x, y ∈ Q}. Then we must show six things:

1. 0 is in F

2. 1 is in F

3. If x and y are in F then so is x + y

4. If x is in F then so is −x

5. If x and y are in F then so is xy

6. If x , 0 is in F then so is x−1

For 1, take x = y = 0. For 2, take x = 1, y = 0. For 3, suppose x = a+b√

2 and y = c+d√

2. Then x+y = (a+c)+(b+d)√

2 ∈ F.For 4, suppose x = a + b

√2. Then −x = (−a) + (−b)

√2 ∈ F. For 5, suppose x = a + b

√2 and y = c + d

√2. Then

1

Page 4: Linear Algebra - Greg Grantgreggrant.org/hoffman_and_kunze.pdf · Linear Algebra Ho man & Kunze 2nd edition ... Section 1.2: Systems of Linear Equations 3 Solution: We must check

2 Chapter 1: Linear Equations

xy = (a + b√

2)(c + d√

2) = (ac + 2bd) + (ad + bc)√

2 ∈ F. For 6, suppose x = a + b√

2 where at least one of a or b is notzero. Let n = a2 − 2b2. Let y = a/n + (−b/n)

√2 ∈ F. Then xy = 1

n (a + b√

2)(a − b√

2) = 1n (a2 − 2b2) = 1. Thus y = x−1 and

y ∈ F.

Exercise 2: Let F be the field of complex numbers. Are the following two systems of linear equations equivalent? If so,express each equation in each system as a linear combination of the equations in the other system.

x1 − x2 = 0 3x1 + x2 = 02x1 + x2 = 0 x1 + x2 = 0

Solution: Yes the two systems are equivalent. We show this by writing each equation of the first system in terms of thesecond, and conversely.

3x1 + x2 =13

(x1 − x2) +43

(2x1 + x2)

x1 + x2 =−13

(x1 − x2) +23

(2x1 + x2)

x1 − x2 = (3x1 + x2) − 2(x1 + x2)

2x1 + x2 =12

(3x1 + x2) +12

(x1 + x2)

Exercise 3: Test the following systems of equations as in Exercise 2.

−x1 + x2 +4x3 = 0 x1 − x3= 0x1 + 3x2+8x3 = 0 x2 + x3= 0

12 x1 + x2 + 5

2 x3 = 0

Solution: Yes the two systems are equivalent. We show this by writing each equation of the first system in terms of thesecond, and conversely.

x1 − x3 = −34 (−x1 + x2 + 4x3) + 1

4 (x1 + 3x3 + 8x3)x2 + 3x3 = 1

4 (−x1 + x2 + 4x3) + 14 (x1 + 3x3 + 8x3)

and

−x1 + x2 + 4x3 = −(x1 − x3) + (x2 + 3x3)x1 + 3x2 + 8x3 = (x1 − x3) + 3(x2 + 3x3)12 x1 + x2 + 5

2 x3 = 12 (x1 − x3) + (x2 + 3x3)

Exercise 4: Test the following systems as in Exercie 2.

2x1 + (−1 + i)x2 + x4 = 0 (1 + i2 )x1+8x2 − ix3 −x4 = 0

3x2 − 2ix3 + 5x4 = 0 23 x1 −

12 x2 + x3 +7x4 = 0

Solution: These systems are not equivalent. Call the two equations in the first system E1 and E2 and the equations in thesecond system E′1 and E′2. Then if E′2 = aE1 + bE2 since E2 does not have x1 we must have a = 1/3. But then to get thecoefficient of x4 we’d need 7x4 = 1

3 x4 + 5bx4. That forces b = 43 . But if a = 1

3 and b = 43 then the coefficient of x3 would have

to be −2i 43 which does not equal 1. Therefore the systems cannot be equivalent.

Exercise 5: Let F be a set which contains exactly two elements, 0 and 1. Define an addition and multiplication by the tables:

+ 0 10 0 11 1 0

· 0 10 0 00 0 1

Page 5: Linear Algebra - Greg Grantgreggrant.org/hoffman_and_kunze.pdf · Linear Algebra Ho man & Kunze 2nd edition ... Section 1.2: Systems of Linear Equations 3 Solution: We must check

Section 1.2: Systems of Linear Equations 3

Solution: We must check the nine conditions on pages 1-2:

1. An operation is commutative if the table is symmetric across the diagonal that goes from the top left to the bottom right.This is true for the addition table so addition is commutative.2. There are eight cases. But if x = y = z = 0 or x = y = z = 1 then it is obvious. So there are six non-trivial cases. If there’sexactly one 1 and two 0’s then both sides equal 1. If there are exactly two 1’s and one 0 then both sides equal 0. So additionis associative.3. By inspection of the addition table, the element called 0 indeed acts like a zero, it has no effect when added to anotherelement.4. 1 + 1 = 0 so the additive inverse of 1 is 1. And 0 + 0 = 0 so the additive inverse of 0 is 0. In other words −1 = 1 and−0 = 0. So every element has an additive inverse.5. As stated in 1, an operation is commutative if the table is symmetric across the diagonal that goes from the top left to thebottom right. This is true for the multiplication table so multiplication is commutative.6. As with addition, there are eight cases. If x = y = z = 1 then it is obvious. Otherwise at least one of x, y or z must equal 0.In this case both x(yz) and (xy)z equal zero. Thus multiplication is associative.7. By inspection of the multiplication table, the element called 1 indeed acts like a one, it has no effect when multiplied toanother element.8. There is only one non-zero element, 1. And 1 · 1 = 1. So 1 has a multiplicative inverse. In other words 1−1 = 1.9. There are eight cases. If x = 0 then clearly both sides equal zero. That takes care of four cases. If all three x = y = z = 1then it is obvious. So we are down to three cases. If x = 1 and y = z = 0 then both sides are zero. So we’re down to the twocases where x = 1 and one of y or z equals 1 and the other equals 0. In this case both sides equal 1. So x(y + z) = (x + y)z inall eight cases.

Exercise 6: Prove that if two homogeneous systems of linear equations in two unknowns have the same solutions, then theyare equivalent.

Solution: Write the two systems as follows:

a11x + a12y = 0a21x + a22y = 0

...am1x + am2y = 0

b11x + b12y = 0b21x + b22y = 0

...bm1x + bm2y = 0

Each system consists of a set of lines through the origin (0, 0) in the x-y plane. Thus the two systems have the same solutionsif and only if they either both have (0, 0) as their only solution or if both have a single line ux + vy − 0 as their commonsolution. In the latter case all equations are simply multiples of the same line, so clearly the two systems are equivalent. Soassume that both systems have (0, 0) as their only solution. Assume without loss of generality that the first two equations inthe first system give different lines. Then

a11

a12,

a21

a22(1)

We need to show that there’s a (u, v) which solves the following system:

a11u + a12v = bi1a21u + a22v = bi2

Solving for u and v we get

u =a22bi1 − a12bi2

a11a22 − a12a21

v =a11bi2 − a21bi1

a11a22 − a12a12

By (1) a11a22 − a12a21 , 0. Thus both u and v are well defined. So we can write any equation in the second system as acombination of equations in the first. Analogously we can write any equation in the first system in terms of the second.

Page 6: Linear Algebra - Greg Grantgreggrant.org/hoffman_and_kunze.pdf · Linear Algebra Ho man & Kunze 2nd edition ... Section 1.2: Systems of Linear Equations 3 Solution: We must check

4 Chapter 1: Linear Equations

Exercise 7: Prove that each subfield of the field of complex numbers contains every rational number.

Solution: Every subfield of C has characterisitc zero since if F is a subfield then 1 ∈ F and n · 1 = 0 in F implies n · 1 = 0 inC. But we know n · 1 = 0 in C implies n = 0. So 1, 2, 3, . . . are all distinct elements of F. And since F has additive inverses−1,−2,−3, . . . are also in F. And since F is a field also 0 ∈ F. Thus Z ⊆ F. Now F has multiplicative inverses so ± 1

n ∈ F forall natural numbers n. Now let m

n be any element of Q. Then we have shown that m and 1n are in F. Thus their product m · 1

nis in F. Thus m

n ∈ F. Thus we have shown all elements of Q are in F.

Exercise 8: Prove that each field of characteristic zero contains a copy of the rational number field.

Solution: Call the additive and multiplicative identities of F 0F and 1F respectively. Define nF to be the sum of n 1F’s. SonF = 1F + 1F + · · · + 1F (n copies of 1F). Define −nF to be the additive inverse of nF . Since F has characteristic zero, ifn , m then nF , mF . For m, n ∈ Z, n , 0, let m

n F = mF · n−1F . Since F has characteristic zero, if m

n ,m′n′ then m

n F ,m′n′ F .

Therefore the map mn 7→

mn F gives a one-to-one map from Q to F. Call this map h. Then h(0) = 0F , h(1) = 1F and in general

h(x + y) = h(x) + h(y) and h(xy) = h(x)h(y). Thus we have found a subset of F that is in one-to-one correspondence to Q andwhich has the same field structure as Q.

Section 1.3: Matrices and Elementary Row OperationsPage 10, typo in proof of Theorem 4. Pargraph 2, line 6, it says kr , k but on the next line they call it k′ instead of kr. I thinkit’s best to use k′, because kr is a more confusing notation.

Exercise 1: Find all solutions to the systems of equations

(1 − i)x1 − ix2 = 02x1 + (1 − i)x2 = 0.

Solution: The matrix of coefficients is [1 − i −i

2 1 − i

].

Row reducing

[2 1 − i

1 − i −i

]→

[2 1 − i0 0

]Thus 2x1 + (1 − i)x2 = 0. Thus for any x2 ∈ C, ( 1

2 (i − 1)x2, x2) is a solution and these are all solutions.

Exercise 2: If

A =

3 −1 22 1 11 −3 0

find all solutions of AX = 0 by row-reducing A.

Solution:

1 −3 02 1 13 −1 2

→ 1 −3 0

0 7 10 8 2

→ 1 −3 0

0 1 1/70 8 2

1 0 3/70 1 1/70 0 6/7

→ 1 0 3/7

0 1 1/70 0 1

→ 1 0 0

0 1 100 0 1

.Thus A is row-equivalent to the identity matrix. It follows that the only solution to the system is (0, 0, 0).

Page 7: Linear Algebra - Greg Grantgreggrant.org/hoffman_and_kunze.pdf · Linear Algebra Ho man & Kunze 2nd edition ... Section 1.2: Systems of Linear Equations 3 Solution: We must check

Section 1.3: Matrices and Elementary Row Operations 5

Exercise 3: If

A =

6 −4 04 −2 0−1 0 3

find all solutions of AX = 2X and all solutions of AX = 3X. (The symbol cX denotes the matrix each entry of which is c timesthe corresponding entry of X.)

Solution: The system AX = 2X is 6 −4 04 −2 0−1 0 3

x

yz

= 2

xyz

which is the same as

6x − 4y = 2x

4x − 2y = 2y

−x + 3z = 2z

which is equivalent to

4x − 4y = 04x − 4y = 0−x + z = 0

The matrix of coefficients is 4 −4 04 −4 0−1 0 1

which row-reduces to 1 0 −1

0 1 −10 0 0

Thus the solutions are all elements of F3 of the form (x, x, x) where x ∈ F.

The system AX = 3X is 6 −4 04 −2 0−1 0 3

x

yz

= 3

xyz

which is the same as

6x − 4y = 3x

4x − 2y = 3y

−x + 3z = 3z

which is equivalent to

3x − 4y = 0x − 2y = 0−x = 0

Page 8: Linear Algebra - Greg Grantgreggrant.org/hoffman_and_kunze.pdf · Linear Algebra Ho man & Kunze 2nd edition ... Section 1.2: Systems of Linear Equations 3 Solution: We must check

6 Chapter 1: Linear Equations

The matrix of coefficients is 3 −4 01 −2 0−1 0 0

which row-reduces to 1 0 0

0 1 00 0 0

Thus the solutions are all elements of F3 of the form (0, 0, z) where z ∈ F.

Exercise 4: Find a row-reduced matrix which is row-equivalent to

A =

i −(1 + i) 01 −2 11 2i −1

.Solution:

A→

1 −2 1i −(1 + i) 01 2i −1

→ 1 −2 1

0 −1 + i −i0 2 + 2i −2

→ 1 −2 1

0 1 1−i2

0 2 + 2i −2

1 −2 10 1 i−1

20 0 0

→ 1 0 i

0 1 i−12

0 0 0

Exercise 5: Prove that the following two matrices are not row-equivalent: 2 0 0

a −1 0b c 3

1 1 2−2 0 −11 3 5

.Solution: Call the first matrix A and the second matrix B. The matrix A is row-equivalent to

A′ =

1 0 00 1 00 0 1

and the matrix B is row-equivalent to

B′ =

1 0 1/20 1 3/20 0 0

.By Theorem 3 page 7 AX = 0 and A′X = 0 have the same solutions. Similarly BX = 0 and B′X = 0 have the same solutions.Now if A and B are row-equivalent then A′ and B′ are row equivalent. Thus if A and B are row equivalent then A′X = 0 andB′X = 0 must have the same solutions. But B′X = 0 has infinitely many solutions and A′X = 0 has only the trivial solution(0, 0, 0). Thus A and B cannot be row-equivalent.

Exercise 6: Let

A =

[a bc d

]be a 2 × 2 matrix with complex entries. Suppose that A is row-reduced and also that a + b + c + d = 0. Prove that there areexactly three such matrices.

Page 9: Linear Algebra - Greg Grantgreggrant.org/hoffman_and_kunze.pdf · Linear Algebra Ho man & Kunze 2nd edition ... Section 1.2: Systems of Linear Equations 3 Solution: We must check

Section 1.3: Matrices and Elementary Row Operations 7

Solution: Case a , 0: Then to be in row-reduced form it must be that a = 1 and A =

[1 bc d

]which implies c = 0, so

A =

[1 b0 d

]. Suppose d , 0. Then to be in row-reduced form it must be that d = 1 and b = 0, so A =

[1 00 1

]which

implies a + b + c + d , 0. So it must be that d = 0, and then it follows that b = −1. So a , 0⇒ A =

[1 −10 0

].

Case a = 0: Then A =

[0 bc d

]. If b , 0 then b must equal 1 and A =

[0 1c d

]which forces d = 0. So A =

[0 1c 0

]which implies (since a + b + c + d = 0) that c = −1. But c cannot be −1 in row-reduced form. So it must be that b = 0. So

A =

[0 0c d

]. If c , 0 then c = 1, d = −1 and A =

[0 01 −1

]. Otherwise c = 0 and A =

[0 00 0

].

Thus the three possibilities are: [0 00 0

],

[1 −10 0

],

[0 01 −1

].

Exercise 7: Prove that the interchange of two rows of a matrix can be accomplished by a finite sequence of elementary rowoperations of the other two types.

Solution: Write the matrix as

R1R2R3...

Rn

.

WOLOG we’ll show how to exchange rows R1 and R2. First add R2 to R1:

R1 + R2R2R3...

Rn

.

Next subtract row one from row two:

R1 + R2−R1R3...

Rn

.

Next add row two to row one again

R2−R1R3...

Rn

.

Finally multiply row two by −1:

R2R1R3...

Rn

.

Page 10: Linear Algebra - Greg Grantgreggrant.org/hoffman_and_kunze.pdf · Linear Algebra Ho man & Kunze 2nd edition ... Section 1.2: Systems of Linear Equations 3 Solution: We must check

8 Chapter 1: Linear Equations

Exercise 8: Consider the system of equations AX = 0 where

A =

[a bc d

]is a 2 × 2 matrix over the field F. Prove the following:

(a) If every entry of A is 0, then every pair (x1, x2) is a solution of AX = 0.(b) If ad − bc , 0, the system AX = 0 has only the trivial solution x1 = x2 = 0.(c) If ad − bc = 0 and some entry of A is different from 0, then there is a solution (x0

1, x02) such that (x1, x2) is a solution if

and only if there is a scalar y such that x1 = yx01, x2 = yx0

2.

Solution:

(a) In this case the system of equations is

0 · x1 + 0 · x2 = 00 · x1 + 0 · x2 = 0

Clearly any (x1, x2) satisfies this system since 0 · x = 0 ∀ x ∈ F.

(b) Let (u, v) ∈ F2. Consider the system:

a · x1 + b · x2 = u

c · x1 + d · x2 = v

If ad − bc , 0 then we can solve for x1 and x2 explicitly as

x1 =du − bvad − bc

x2 =av − cuad − bc

.

Thus there’s a unique solution for all (u, v) and in partucular when (u, v) = (0, 0).

(c) Assume WOLOG that a , 0. Then ad − bc = 0⇒ d = cba . Thus if we multiply the first equation by c

a we get the secondequation. Thus the two equations are redundant and we can just consider the first one ax1 + bx2 = 0. Then any solution is ofthe form (− b

a y, y) for arbitrary y ∈ F. Thus letting y = 1 we get the solution (−b/a, 1) and the arbitrary solution is of the formy(−b/a, 1) as desired.

Section 1.4: Row-Reduced Echelon MatricesExercise 1: Find all solutions to the following system of equations by row-reducing the coefficient matrix:

13

x1 + 2x2 − 6x3 = 0

−4x1 + 5x3 = 0−3x1 + 6x2 − 13x3 = 0

−73

x1 + 2x2 −83

x3 = 0

Solution: The coefficient matrix is 13 2 −6−4 0 5−3 6 −13− 7

3 2 − 83

Page 11: Linear Algebra - Greg Grantgreggrant.org/hoffman_and_kunze.pdf · Linear Algebra Ho man & Kunze 2nd edition ... Section 1.2: Systems of Linear Equations 3 Solution: We must check

Section 1.4: Row-Reduced Echelon Matrices 9

This reduces as follows:

1 6 −18−4 0 5−3 6 −13−7 6 −8

1 6 −180 24 −670 24 −670 48 −134

1 6 −180 24 −670 0 00 0 0

1 6 −180 1 −67/240 0 00 0 0

1 0 −5/40 1 −67/240 0 00 0 0

Thus

x −54

z = 0

y −6724

z = 0

Thus the general solution is ( 54 z, 67

24 z, z) for arbitrary z ∈ F.

Exercise 2: Find a row-reduced echelon matrix which is row-equivalent to

A =

1 −i2 2u 1 + i

.What are the solutions of AX = 0?

Solution: A row-reduces as follows:

1 −i1 1i 1 + i

→ 1 −i

0 1 + i0 i

→ 1 −i

0 10 i

→ 1 0

0 10 0

Thus the only solution to AX = 0 is (0, 0).

Exercise 3: Describe explicitly all 2 × 2 row-reduced echelon matrices.

Solution: [1 00 1

],

[1 x0 0

],

[0 10 0

],

[0 00 0

]Exercise 4: Consider the system of equations

x1 − x2 + 2x3 = 12x1 + 2x3 = 1x1 − 3x2 + 4x3 = 2

Does this system have a solution? If so, describe explicitly all solutions.

Solution: The augmented coefficient matrix is 1 −1 2 12 0 2 11 −3 4 2

We row reduce it as follows:

1 −1 2 10 2 −2 −10 −2 2 1

→ 1 −1 2 1

0 1 −1 −1/20 0 0 0

→ 1 0 1 1/2

0 1 −1 −1/20 0 0 0

Page 12: Linear Algebra - Greg Grantgreggrant.org/hoffman_and_kunze.pdf · Linear Algebra Ho man & Kunze 2nd edition ... Section 1.2: Systems of Linear Equations 3 Solution: We must check

10 Chapter 1: Linear Equations

Thus the system is equivalent to

x1 + x3 = 1/2x2 − x3 = −1/2

Thus the solutions are parameterized by x3. Setting x3 = c gives x1 = 1/2 − c, x2 = c − 1/2. Thus the general solution is(12 − c, c − 1

2 , c)

for c ∈ R.

Exercise 5: Give an example of a system of two linear equations in two unkowns which has no solutions.

Solution:x + y = 0

x + y = 1

Exercise 6: Show that the system

x1 − 2x2 + x3 + 2x4 = 1x1 + x2 − x3 + x4 = 2

x1 + 7x2 − 5x3 − x4 = 3

has no solution.

Solution: The augmented coefficient matrix is as follows 1 −2 1 2 11 1 −1 1 21 7 −5 −1 3

This row reduces as follows:

1 −2 1 2 10 3 −2 −1 10 9 −6 −3 2

→ 1 −2 1 2 1

0 3 −2 −1 10 0 0 0 −1

At this point there’s no need to continue because the last row says 0x1 + 0x2 + 0x3 + 0x4 = −1. But the left hand side of thisequation is zero so this is impossible.

Exercise 7: Find all solutions of

2x1 − 3x2 − 7x3 + 5x4 + 2x5 = −2x1 − 2x2 − 4x3 + 3x4 + x5 = −22x1 − 4x3 + 2x4 + x5 = 3

x1 − 5x2 − 7x3 + 6x4 + 2x5 = −7

Solution: The augmented coefficient matrix is 2 −3 −7 5 2 −21 −2 −4 3 1 −22 0 −4 2 1 31 −5 −7 6 2 −7

Page 13: Linear Algebra - Greg Grantgreggrant.org/hoffman_and_kunze.pdf · Linear Algebra Ho man & Kunze 2nd edition ... Section 1.2: Systems of Linear Equations 3 Solution: We must check

Section 1.4: Row-Reduced Echelon Matrices 11

We row-reduce it as follows

1 −2 −4 3 1 −22 −3 −7 5 2 −22 0 −4 2 1 31 −5 −7 6 2 −7

1 −2 −4 3 1 −20 1 1 −1 0 20 4 4 −4 −1 70 −3 −3 3 1 −5

1 0 −2 1 1 20 1 1 −1 0 20 0 0 0 −1 −10 0 0 0 1 1

1 0 −2 1 0 10 1 1 −1 0 20 0 0 0 1 10 0 0 0 0 0

Thus

x1 − 2x3 + x4 = 1x2 + x3 − x4 = 2

x5 = 1

Thus the general solution is given by (1 + 2x3 − x4, 2 + x4 − x3, x3, x4, 1) for arbitrary x3, x4 ∈ F.

Exercise 8: Let

A =

3 −1 22 1 11 −3 0

.For which (y1, y2, y3) does the system AX = Y have a solution?

Solution: The matrix A is row-reduced as follows:

1 −3 00 7 10 8 2

→ 1 −3 0

0 7 10 1 1

→ 1 −3 0

0 1 10 7 1

1 −3 00 1 10 0 −6

→ 1 −3 0

0 1 10 0 1

→ 1 0 0

0 1 00 0 1

Thus for every (y1, y2, y3) there is a (unique) solution.

Exercise 9: Let 3 −6 2 −1−2 4 1 30 0 1 11 −2 1 0

.For which (y1, y2, y3, y4) does the system of equations AX = Y have a solution?

Solution: We row reduce as follows3 −6 2 −1 y1−2 4 1 3 y20 0 1 1 y31 −2 1 0 y4

1 −2 1 0 y43 −6 2 −1 y1−2 4 1 3 y20 0 1 1 y3

1 −2 1 0 y40 0 −1 −1 y1 − 3y40 0 3 3 y2 + 2y40 0 1 1 y3

1 −2 1 0 y40 0 0 0 y1 − 3y4 + y30 0 0 0 y2 + 2y4 + 3y30 0 1 1 y3

1 −2 1 0 y40 0 1 1 y30 0 0 0 y1 − 3y4 + y30 0 0 0 y2 + 2y4 + 3y3

Page 14: Linear Algebra - Greg Grantgreggrant.org/hoffman_and_kunze.pdf · Linear Algebra Ho man & Kunze 2nd edition ... Section 1.2: Systems of Linear Equations 3 Solution: We must check

12 Chapter 1: Linear Equations

Thus (y1, y2, y3, y4) must satisfy

y1 + y3 − 3y4 = 0y2 + 3y3 + 2y4 = 0

The matrix for this system is [1 0 1 −30 1 3 2

]of which the general solution is (−y3 + 3y4,−3y3 − 2y4, y3, y4) for arbitrary y3, y4 ∈ F. These are the only (y1, y2, y3, y4) forwhich the system AX = Y has a solution.

Exercise 10: Suppose R and R′ are 2× 3 row-reduced echelon matrices and that the system RX = 0 and R′X = 0 have exactlythe same solutions. Prove that R = R′.

Solution: There are seven possible 2 × 3 row-reduced echelon matrices:

R1 =

[0 0 00 0 0

](2)

R2 =

[1 0 a0 1 b

](3)

R3 =

[1 a 00 0 1

](4)

R4 =

[1 a b0 0 0

](5)

R5 =

[0 1 a0 0 0

](6)

R6 =

[0 1 00 0 1

](7)

R7 =

[0 0 10 0 0

](8)

We must show that no two of these have exactly the same solutions. For the first one R1, any (x, y, z) is a solution and that’snot the case for any of the other Ri’s. Consider next R7. In this case z = 0 and x and y can be anything. We can have z , 0 forR2, R3 and R5. So the only ones R7 could share solutions with are R3 or R6. But both of those have restrictions on x and/or yso the solutions cannot be the same. Also R3 and R6 cannot have the same solutions since R6 forces y = 0 while R3 does not.

Thus we have shown that if two Ri’s share the same solutions then they must be among R2, R4, and R5.

The solutions for R2 are (−az,−bz, z), for z arbitrary. The solutions for R4 are (−a′y−b′z, y, z) for y, z arbitrary. Thus (−b′, 0, 1)is a solution for R4. Suppose this is also a solution for R2. Then z = 1 so it is of the form (−a,−b, 1) and it must be that(−b′, 0, 1) = (−a,−b, 1). Comparing the second component implies b = 0. But if b = 0 then R2 implies y = 0. But R4 allowsfor arbitrary y. Thus R2 and R4 cannot share the same solutions.

The solutions for R2 are (−az,−bz, z), for z arbitrary. The solutions for R5 are (x,−a′z, z) for x, z arbitrary. Thus (0,−a′, 1) isa solution for R5. As before if this is a solution of R2 then a = 0. But if a = 0 then R2 forces x = 0 while in R5 x can bearbitrary. Thus R2 and R5 cannot share the same solutions.

The solutions for R4 are (−ay − bz, y, z) for y, z arbitrary. The solutions for R5 are (x,−a′z, z) for x, z arbitrary. Thus settingx = 1, z = 0 gives (1, 0, 0) is a solution for R5. But this cannot be a solution for R4 since if y = z = 0 then first component

Page 15: Linear Algebra - Greg Grantgreggrant.org/hoffman_and_kunze.pdf · Linear Algebra Ho man & Kunze 2nd edition ... Section 1.2: Systems of Linear Equations 3 Solution: We must check

Section 1.5: Matrix Multiplication 13

must also be zero.

Thus we have shown that no two Ri and R j have the same solutions unless i = j.

NOTE: This fact is actually true in general not just for 2 × 3 (search for “1832109” on math.stackexchange).

Section 1.5: Matrix MultiplicationPage 18: Typo in the last paragraph where it says “the columns of B are the 1 × n matrices . . . ” it should be n × 1 not 1 × n.

Page 20: Typo in the Definition, it should say “An m × m matrix is said to be an elementary matrix”... Otherwise it doesn’tmake sense that you can obtain an m × n matrix from an m × m matrix by an elementary row operation unless m = n.

Exercise 1: Let

A =

[2 −1 11 2 1

], B =

31−1

, C = [1 − 1].

Compute ABC and CAB.

Solution:AB =

[44

],

so

ABC =

[44

]· [1 − 1] =

[4 −44 −4

].

and

CBA = [1 − 1] ·[

44

]= [0].

Exercise 2: Let

A =

1 −1 12 0 13 0 1

, B =

2 −21 34 4

.Verify directly that A(AB) = A2B.

Solution:

A2 =

1 −1 12 0 13 0 1

· 1 −1 1

2 0 13 0 1

=

2 −1 15 −2 36 −3 4

.And

AB =

1 −1 12 0 13 0 1

· 2 −2

1 34 4

=

5 −18 010 −2

.

Page 16: Linear Algebra - Greg Grantgreggrant.org/hoffman_and_kunze.pdf · Linear Algebra Ho man & Kunze 2nd edition ... Section 1.2: Systems of Linear Equations 3 Solution: We must check

14 Chapter 1: Linear Equations

Thus

A2B =

2 −1 15 −2 36 −3 4

· 2 −2

1 34 4

=

7 −320 −425 −5

. (9)

And

A(AB) =

1 −1 12 0 13 0 1

· 5 −1

8 010 −2

7 −3

20 −425 −5

. (10)

Comparing (9) and (10) we see both calculations result in the same matrix.

Exercise 3: Find two different 2 × 2 matrices A such that A2 = 0 but A , 0.

Solution: [0 10 0

],

[0 01 0

]are two such matrices.

Exercise 4: For the matrix A of Exercise 2, find elementary matrices E1, E2, . . . , Ek such that

Ek · · · E2E1A = I.

Solution:

A =

1 −1 12 0 13 0 1

E1A =

1 0 0−2 1 00 0 1

1 −1 1

2 0 13 0 1

=

1 −1 10 2 −13 0 1

E2(E1A) =

1 0 00 1 0−3 0 1

1 −1 1

0 2 −13 0 1

=

1 −1 10 2 −10 3 0

E3(E2E1A) =

1 0 00 1/2 00 0 1

1 −1 1

0 2 −10 3 0

=

1 −1 10 1 −1/20 3 0

E4(E3E2E1A) =

1 1 00 1 00 0 1

1 −1 1

0 1 −1/20 3 0

=

1 0 1/20 1 −1/20 3 0

E5(E4E3E2E1A) =

1 0 00 1 00 −3 1

1 0 1/2

0 1 −1/20 3 0

=

1 0 1/20 1 −1/20 0 3/2

Page 17: Linear Algebra - Greg Grantgreggrant.org/hoffman_and_kunze.pdf · Linear Algebra Ho man & Kunze 2nd edition ... Section 1.2: Systems of Linear Equations 3 Solution: We must check

Section 1.5: Matrix Multiplication 15

E6(E5E4E3E2E1A) =

1 0 00 1 00 0 2/3

1 0 1/2

0 1 −1/20 0 3/2

=

1 0 1/20 1 −1/20 0 1

E7(E6E5E4E3E2E1A) =

1 0 −1/20 1 00 0 1

1 0 1/2

0 1 −1/20 0 1

=

1 0 00 1 −1/20 0 1

E8(E7E6E5E4E3E2E1A) =

1 0 00 1 1/20 0 1

1 0 0

0 1 −1/20 0 1

=

1 0 00 1 00 0 1

Exercise 5: Let

A =

1 −12 21 0

, B =

[3 1−4 4

].

Is there a matrix C such that CA = B?

Solution: To find such a C =

[a b cd e f

]we must solve the equation

[a b cd e f

] 1 −12 21 0

=

[3 1−4 4

].

This gives a system of equations

a + 2b + c = 3−a + 2b = 1

d + 2e + f = −4−d + 2e = 4

We row-reduce the augmented coefficient matrix1 2 1 0 0 0 3−1 2 0 0 0 0 10 0 0 1 2 1 −40 0 0 −1 2 0 4

1 0 1/2 0 0 0 10 1 1/4 0 0 0 10 0 0 1 0 1/2 −40 0 0 0 1 1/4 0

.Setting c = f = 4 gives the solution

C =

[−1 0 4−6 −1 4

].

Checking: [−1 0 4−6 −1 4

] 1 −12 21 0

=

[3 1−4 4

].

Exercise 6: Let A be an m× n matrix and B an n× k matrix. Show that the columns of C = AB are linear combinations of thecolumns of A. If α1, . . . , αn are the columns of A and γ1, . . . , γk are the columns of C then

γ j =

n∑r=1

Br jαr.

Page 18: Linear Algebra - Greg Grantgreggrant.org/hoffman_and_kunze.pdf · Linear Algebra Ho man & Kunze 2nd edition ... Section 1.2: Systems of Linear Equations 3 Solution: We must check

16 Chapter 1: Linear Equations

Solution: The i j-th entry of AB is∑k

r=1 AirBr j. Since the term Br j is independent of i, we can view the sum independent ofi as

∑nr=1 Br jαr where αr is the r-th column of A. I’m not sure what more to say, this is pretty immediately obvious from the

definition of matrix multiplication.

Exercise 7: Let A and B be 2 × 2 matrices such that AB = I. Prove that BA = I.

Solution: Suppose

A =

[a bc d

], B =

[x yz w

].

AB =

[ax + bz ay + bwcx + dz cy + dw

].

Then AB = I implies the following system in u, r, s, t has a solution

au + bs = 1cu + ds = 0ar + bt = 0cr + dt = 1

because (x, y, z,w) is one such solution. The augmented coefficient matrix of this system isa 0 b 0 1c 0 d 0 00 a 0 b 00 c 0 d 1

. (11)

As long as ad − bc , 0 this system row-reduces to the following row-reduced echelon form1 0 0 0 d/(ad − bc)0 1 0 0 −b/(ad − bc)0 0 1 0 −c/(ad − bc)0 0 0 1 a/(ad − bc)

Thus we see that necessarily x = d/(ad − bc), y = −b/(ad − bc), z = −c/(ad − bc), w = a/(ad − bc). Thus

B =

[d/(ad − bc) −b/(ad − bc)−c/(ad − bc) a/(ad − bc)

].

Now it’s a simple matter to check that[d/(ad − bc) −b/(ad − bc)−c/(ad − bc) a/(ad − bc)

[a bc d

]=

[1 00 1

].

The loose end is that we assumed ad − bc , 0. To tie up this loose end we must show that if AB = I then necessarilyad − bc , 0. Suppose that ad − bc = 0. We will show there is no solution to (11), which contradicts the fact that (x, y, z,w)is a solution. If a = b = c = d = 0 then obviously AB , I. So suppose WOLOG that a , 0 (because by elementary rowoperations we can move any of the four elements to be the top left entry). Subtracting c

a times the 3rd row from the 4th rowof (11) gives

a 0 b 0 1c 0 d 0 00 a 0 b 00 c − c

a a 0 d − ca b 1

.

Page 19: Linear Algebra - Greg Grantgreggrant.org/hoffman_and_kunze.pdf · Linear Algebra Ho man & Kunze 2nd edition ... Section 1.2: Systems of Linear Equations 3 Solution: We must check

Section 1.5: Matrix Multiplication 17

Now c − ca a = 0 and since ad − bc = 0 also d − c

a b = 0. Thus we geta 0 b 0 1c 0 d 0 00 a 0 b 00 0 0 0 1

.and it follows that (11) has no solution.

Exercise 8: Let

C =

[C11 C12C21 C22

]be a 2 × 2 matrix. We inquire when it is possible to find 2 × 2 matrices A and B such that C = AB − BA. Prove that suchmatrices can be found if and only fi C11 + C22 = 0.

Solution: We want to know when we can solve for a, b, c, d, x, y, z,w such that[c11 c12c21 c22

]=

[a bc d

] [x yz w

]−

[a bc d

] [x yz w

]The right hand side is equal to [

bz − cy ay + bw − bx − dycx + dz − az − cw cy − bz

]Thus the question is equivalent to asking: when can we choose a, b, c, d so that the following system has a solution for x, y, z,w

bz − cy = c11

ay + bw − bx − dy = c12

cx + dz − az − cw = c21

cy − bz = c22

(12)

The augmented coefficient matrix for this system is0 −c b 0 c11−b a − d 0 b c12c 0 d − a −c c210 c −b 0 c22

This matrix is row-equivalent to

0 −c b 0 c11−b a − d 0 b c12c 0 d − a −c c210 0 0 0 c11 + c22

from which we see that necessarily c11 + c22 = 0.

Suppose conversely that c11 + c22 = 0. We want to show ∃ A, B such that C = AB − BA.

We first handle the case when c11 = 0. We know c11 + c22 = 0 so also c22 = 0. So C is in the form[0 c12

c21 0

].

In this case let

A =

[0 c12−c21 0

], B =

[−1 00 0

].

Page 20: Linear Algebra - Greg Grantgreggrant.org/hoffman_and_kunze.pdf · Linear Algebra Ho man & Kunze 2nd edition ... Section 1.2: Systems of Linear Equations 3 Solution: We must check

18 Chapter 1: Linear Equations

ThenAB − BA

=

[0 c12−c21 0

] [−1 00 0

]−

[−1 00 0

] [0 c12−c21 0

]

=

[0 0

c21 0

]−

[0 −c120 0

]

=

[0 c12

c21 0

]= C.

So we can assume going forward that c11 , 0. We want to show the system (12) can be solved. In other words we have tofind a, b, c, d such that the system has a solution in x, y, z,w. If we assume b , 0 and c , 0 then this matrix row-reduces to thefollowing row-reduced echelon form

1 0 (d − a)/c −1 d−abc c11 −

c12b

0 1 −d/c 0 −c11/c0 0 0 0 −

c11b (d − a) + c21 + c12c

b0 0 0 0 c11 + c22

We see that necessarily

−c11

b(d − a) + c21 +

c12cb

= 0.

Since c11 , 0, we can set a = 0, b = c = 1 and d = c12+c21c11

. Then the system can be solved and we get a solution for anychoice of z and w. Setting z = w = 0 we get x = c21 and y = c11.

Summarizing, if c11 , 0 then:a = 0

b = 1

c = 1

d = (c12 + c21)/c11

x = c21

y = c11

z = 0

w = 0

For example if C =

[2 13 −2

]then A =

[0 11 2

]and B =

[3 −20 0

]. Checking:

[0 11 2

] [3 −20 0

]−

[3 −20 0

] [0 11 2

]=

[2 13 −2

].

Page 21: Linear Algebra - Greg Grantgreggrant.org/hoffman_and_kunze.pdf · Linear Algebra Ho man & Kunze 2nd edition ... Section 1.2: Systems of Linear Equations 3 Solution: We must check

Section 1.6: Invertible Matrices 19

Section 1.6: Invertible MatricesExercise 1: Let

A =

1 2 1 0−1 0 3 51 −2 1 1

.Find a row-reduced echelon matrix R which is row-equivalent to A and an invertible 3 × 3 matrix P such that R = PA.

Solution: As in Exercise 4, Section 1.5, we row reduce and keep track of the elementary matrices involved. It takes ninesteps to put A in row-reduced form resulting in the matrix

P =

3/8 −1/4 3/81/4 0 −1/41/8 1/4 1/8

.Exercise 2: Do Exercise 1, but with

A =

2 0 i1 −3 −ii 1 1

.Solution: Same story as Exercise 1, we get to the identity matrix in nine elementary steps. Multiplying those nine elementarymatrices together gives

P =

1/3 − 29+3i

301−3i10

0 − 3+i10

1−3i10

−i/3 3+i15

3+i5

.Exercise 3: For each of the two matrices 2 5 −1

4 −1 26 4 1

, 1 −1 2

3 2 40 1 −2

use elementary row operations to discover whether it is invertible, and to find the inverse in case it is.

Solution: For the first matrix we row-reduce the augmented matrix as follows: 2 5 −1 1 0 04 −1 2 0 1 06 4 1 0 0 1

2 5 −1 1 0 00 −11 4 −2 1 00 −11 4 −3 0 1

2 5 −1 1 0 00 −11 4 −2 1 00 0 0 −1 −1 1

At this point we see that the matrix is not invertible since we have obtained an entire row of zeros.

For the second matrix we row-reduce the augmented matrix as follows: 1 −1 2 1 0 03 2 4 0 1 00 1 −2 0 0 1

Page 22: Linear Algebra - Greg Grantgreggrant.org/hoffman_and_kunze.pdf · Linear Algebra Ho man & Kunze 2nd edition ... Section 1.2: Systems of Linear Equations 3 Solution: We must check

20 Chapter 1: Linear Equations

1 −1 2 1 0 00 5 −2 −3 1 00 1 −2 0 0 1

1 −1 2 1 0 00 1 −2 0 0 10 5 −2 −3 1 0

1 0 0 1 0 10 1 −2 0 0 10 0 8 −3 1 −5

1 0 0 1 0 10 1 −2 0 0 10 0 1 −3/8 1/8 −5/8

1 0 0 1 0 10 1 0 −3/4 1/4 −1/40 0 1 −3/8 1/8 −5/8

Thus the inverse matrix is 1 0 1

−3/4 1/4 −1/43/8 1/8 −5/8

Exercise 4: Let

A =

5 0 01 5 00 1 5

.For which X does there exist a scalar c such that AX = cX?

Solution: 5 0 01 5 00 1 5

x

yz

= c

xyz

implies

5x = cx (13)x + 5y = cy (14)y + 5z = cz (15)

Now if c , 5 then (13) implies x = 0, and then (14) implies y = 0, and then (15) implies z = 0. So it is true for

000

with c = 0.

If c = 5 then (14) implies x = 0 and (15) implies y = 0. So if c = 5 any such vector must be of the form

00z

and indeed any

such vector works with c = 5.

So the final answer is any vector of the form

00z

.

Page 23: Linear Algebra - Greg Grantgreggrant.org/hoffman_and_kunze.pdf · Linear Algebra Ho man & Kunze 2nd edition ... Section 1.2: Systems of Linear Equations 3 Solution: We must check

Section 1.6: Invertible Matrices 21

Exercise 5: Discover whether 1 2 3 40 2 3 40 0 3 40 0 0 4

is invertible, and find A−1 if it exists.

Solution: We row-reduce the augmented matrix as follows:1 2 3 4 1 0 0 00 2 3 4 0 1 0 00 0 3 4 0 0 1 00 0 0 4 0 0 0 1

1 0 0 0 1 −1 0 00 2 3 4 0 1 0 00 0 3 4 0 0 1 00 0 0 4 0 0 0 1

1 0 0 0 1 −1 0 00 2 0 0 0 1 −1 00 0 3 4 0 0 1 00 0 0 4 0 0 0 1

1 0 0 0 1 −1 0 00 2 0 0 0 1 −1 00 0 3 0 0 0 1 −10 0 0 4 0 0 0 1

1 0 0 0 1 −1 0 00 1 0 0 0 1/2 −1/2 00 0 1 0 0 0 1/3 −1/30 0 0 1 0 0 0 1/4

Thus the A does have an inverse and

A−1 =

1 −1 0 00 1/2 −1/2 00 0 1/3 −1/30 0 0 1/4

.Exercise 6: Suppose A is a 2 × 1 matrix and that B is a 1 × 2 matrix. Prove that C = AB is not invertible.

Solution: Write A =

[a1a2

]and B = [b1 b2]. Then

AB =

[a1b1 a1b2a2b1 a2b2

].

If any of a1, a2, b1 or b2 equals zero then AB has an entire row or an entire column of zeros. A matrix with an entire row orcolumn of zeros is not invertible. Thus assume a1, a2, b1 and b2 are non-zero. Now if we add −a2/a1 of the first row to thesecond row we get [

a1b1 a1b20 0

].

Thus AB is not row-equivalent to the identity. Thus by Theorem 12 page 23, AB is not invertible.

Page 24: Linear Algebra - Greg Grantgreggrant.org/hoffman_and_kunze.pdf · Linear Algebra Ho man & Kunze 2nd edition ... Section 1.2: Systems of Linear Equations 3 Solution: We must check

22 Chapter 1: Linear Equations

Exercise 7: Let A be an n × n (square) matrix. Prove the following two statements:(a) If A is invertible and AB = 0 for some n × n matrix B then B = 0.(b) If A is not invertible, then there exists an n × n matrix B such that AB = 0 but B , 0.

Solution:

(a) 0 = A−10 = A−1(AB) = (A−1A)B = IB = B. Thus B = 0.

(b) By Theorem 13 (ii) since A is not invertible AX = 0 must have a non-trivial solution v. Let B be the matrix all of whosecolumns are equal to v. Then B , 0 but AB = 0.

Exercise 8: Let

A =

[a bc d

].

Prove, using elementary row operations, that A is invertible if and only if (ad − bc) , 0.

Solution: Suppose

A =

[a bc d

], B =

[x yz w

].

Then

AB =

[ax + bz ay + bwcx + dz cy + dw

].

Then AB = I implies the following system in u, r, s, t has a solution

au + bs = 1cu + ds = 0ar + bt = 0cr + dt = 1

because (x, y, z,w) is one such solution. The augmented coefficient matrix of this system isa 0 b 0 1c 0 d 0 00 a 0 b 00 c 0 d 1

. (16)

As long as ad − bc , 0 this system row-reduces to the following row-reduced echelon form1 0 0 0 d/(ad − bc)0 1 0 0 −b/(ad − bc)0 0 1 0 −c/(ad − bc)0 0 0 1 a/(ad − bc)

Thus we see that x = d/(ad − bc), y = −b/(ad − bc), z = −c/(ad − bc), w = a/(ad − bc) and

A−1 =

[d/(ad − bc) −b/(ad − bc)−c/(ad − bc) a/(ad − bc)

].

Now it’s a simple matter to check that[d/(ad − bc) −b/(ad − bc)−c/(ad − bc) a/(ad − bc)

[a bc d

]=

[1 00 1

].

Page 25: Linear Algebra - Greg Grantgreggrant.org/hoffman_and_kunze.pdf · Linear Algebra Ho man & Kunze 2nd edition ... Section 1.2: Systems of Linear Equations 3 Solution: We must check

Section 1.6: Invertible Matrices 23

Now suppose that ad − bc = 0. We will show there is no solution. If a = b = c = d = 0 then obviously A has no inverse.So suppose WOLOG that a , 0 (because by elementary row and column operations we can move any of the four elementsto be the top left entry, and elementary row and column operations do not change a matrix’s status as being invertible or not).Subtracting c

a times the 3rd row from the 4th row of (16) givesa 0 b 0 1c 0 d 0 00 a 0 b 00 c − c

a a 0 d − ca b 1

.Now c − c

a a = 0 and since ad − bc = 0 also d − ca b = 0. Thus we get

a 0 b 0 1c 0 d 0 00 a 0 b 00 0 0 0 1

.and it follows that A is not invertible.

Exercise 9: An n×n matrix A is called upper-triangular if ai j = 0 for i > j, that is, if every entry below the main diagonal is0. Prove that an upper-triangular (square) matrix is invertible if and only if every entry on its main diagonal is different fromzero.

Solution: Suppose that aii , 0 for all i. Then we can divide row i by aii to give a row-equivalent matrix which has all oneson the diagonal. Then by a sequence of elementary row operations we can turn all off diagonal elements into zeros. We cantherefore row-reduce the matrix to be equivalent to the identity matrix. By Theorem 12 page 23, A is invertible.

Now suppose that some aii = 0. If all aii’s are zero then the last row of the matrix is all zeros. A matrix with a row of zeroscannot be row-equivalent to the identity so cannot be invertible. Thus we can assume there’s at least one i such that aii , 0.Let i′ be the largest such index, so that ai′i′ = 0 and aii , 0 for all i > i′. We can divide all rows with i > i′ by aii to give oneson the diagonal for those rows. We can then add multiples of those rows to row i′ to turn row i′ into an entire row of zeros.Since again A is row-equivalent to a matrix with an entire row of zeros, it cannot be invertible.

Exercise 10: Prove the following generalization of Exercise 6. If A is an m × n matrix and B is an n × m matrix and n < m,then AB is not invertible.

Solution: There are n colunms in A so the vector space generated by those columns has dimension no greater than n. Allcolumns of AB are linear combinations of the columns of A. Thus the vector space generated by the columns of AB is con-tained in the vector space generated by the columns of A. Thus the column space of AB has dimension no greater than n.Thus the column space of the m×m matrix AB has dimension less or equal to n and n < m. Thus the columns of AB generatea space of dimension strictly less than m. Thus AB is not invertible.

Exercise 11: Let A be an n × m matrix. Show that by means of a finite number of elementary row and/or column operationsone can pass from A to a matrix R which is both ‘row-reduced echelon’ and ‘column-reduced echelon,’ i.e., Ri j = 0 if i , j,Rii = 1, 1 ≤ i ≤ r, Rii = 0 if i > r. Show that R = PAQ, where P is an invertible m×m matrix and Q is an invertible n×n matrix.

Solution: First put A in row-reduced echelon form, R′. So ∃ an invertible m ×m matrix P such that R′ = PA. Each row of R′

is either all zeros or starts (on the left) with zeros, then has a one, then may have non-zero entries after the one. Suppose rowi has a leading one in the j-th column. The j-th column has zeros in all other places except the i-th, so if we add a multipleof this column to another column then it only affects entries in the i-th row. Therefore a sequence of such operations can turnthis row into a row of all zeros and a single one.

Page 26: Linear Algebra - Greg Grantgreggrant.org/hoffman_and_kunze.pdf · Linear Algebra Ho man & Kunze 2nd edition ... Section 1.2: Systems of Linear Equations 3 Solution: We must check

24 Chapter 1: Linear Equations

Let B be the n × n matrix such that Brr = 1 and Brs = 0 ∀ r , s except B jk , 0. Then AB equals A with B jk times columnj added to column k. B is invertible since any such operation can be undone by another such operation. By a sequence ofsuch operations we can turn all values after the leading one into zeros. Let Q be a product of all of the elementary matrices Binvolved in this transformation. Then PAQ is in row-reduced and column-reduced form.

Exercise 12: The result of Example 16 suggests that perhaps the matrix1 1

2 · · · 1n

12

13 · · · 1

n+1...

......

1n

1n+1 · · · 1

2n−1

is invertible and A−1 has integer entries. Can you prove that?

Solution: This problem seems a bit hard for this book. There are a class of theorems like this, in particular these are calledHilbert Matrices and a proof is given in this article on arxiv by Christian Berg called Fibonacci numbers and orthogonalpolynomials (http://arxiv.org/pdf/math/0609283v2.pdf). See Theorem 4.1. Also there might be a more elementaryproof in this discussion on mathoverflow.net where two proofs are given:http://mathoverflow.net/questions/47561/deriving-inverse-of-hilbert-matrix.Also see http://vigo.ime.unicamp.br/HilbertMatrix.pdf where a general formula for the i, j entry of the inverse isgiven explicitly as

(−1)i+ j(i + j − 1)(n + i − 1

n − j

)(n + j − 1

n − i

)(i + j − 1

i − 1

)2

Page 27: Linear Algebra - Greg Grantgreggrant.org/hoffman_and_kunze.pdf · Linear Algebra Ho man & Kunze 2nd edition ... Section 1.2: Systems of Linear Equations 3 Solution: We must check

Chapter 2: Vector Spaces

Section 2.1: Vector SpacesPage 29, three lines into the first paragraph of text they refer to “two operations”. This could be confusing since they justsaid a vector space is a composite object consisting of a field and a set with a rule. There are two sets, the field F and the setof vectors V . Really there are four operations going around. There is addition and multiplication in the field F, and there isaddition also in V , that’s three. But there’s also multiplication of an element of F and an element of V . That’s why there aretwo distributive rules. But keep in mind we do not multiply elements of V together - there’s no multiplication within V , onlywithin F and between F and V . So anyway, why did they say “two” operations? They’re clearly ignoring the two operationsin the field and just talking about operations involving vectors. You get the point.

Exercise 1: If F is a field, verify that Fn (as defined in Example 1) is a vector space over the field F.

Solution: Example 1 starts with any field and defines the objects, the addition rule and the scalar multiplication rule. Wemust show the set of n-tuples satisfies the eight properties required in the definition.

1) Addition is commutative. Let α = (x1, . . . , xn) and β = (y1, . . . , yn) be two n-tuples. Then α + β = (x1 + y1, . . . , xn + yn).And since F is commutative this equals (y1 + x1, . . . , yn + xn), which equals β + α. Thus α + β = β + α.

2) Addition is associative. Let α = (x1, . . . , xn), β = (y1, . . . , yn) and γ = (z1, . . . , zn) be three n-tuples. Then (α + β) + γ =

((x1 +y1)+z1, . . . , (xn +yn)+zn). And since F is associative this equals (x1 +(y1 +z1), . . . , xn +(yn +zn)), which equals α+(β+γ).

3) We must show there is a unique vector 0 in V such that α + 0 = α ∀ α ∈ V . Consider (0F , . . . , 0F) the vector of all0’s of length n, where 0F is the zero element of F. Then this vector satisfies the property that (0F , . . . , 0F) + (x1, . . . , xn) =

(0F + x1, . . . , 0F + xn) = (x1, . . . , xn) since 0F + x = x ∀ x ∈ F. Thus (0F , . . . , 0F) + α = α ∀α ∈ V . We must just show thisvector is unique with respect to this property. Suppose β = (x1, . . . , xn) also satisfies the property that β+ α = α for all α ∈ V .Let α = (0F , . . . , 0F). Then (x1, . . . , xn) = (x1 + 0F , . . . , xn + 0F) = (x1, . . . , xn) + (0F , . . . , 0F) and by definition of β this equals(0F , . . . , 0F). Thus (x1, . . . , xn) = (0F , . . . , 0F). Thus β = α and the zero element is unique.

4) We must show for each vector α there is a unique vector β such that α + β = 0. Suppose α = (x1, . . . , xn). Let β =

(−x1, . . . ,−xn). Then β has the required property α + β = 0. We must show β is unique with respect to this property. Supposealso β′ = (x′1, . . . , x

′n) also has this property. Then α+β = 0 and α+β′ = 0. So β = β+0 = β+(α+β′) = (β+α)+β′ = 0+β′ = β′.

5) Let 1F be the multiplicative identity in F. Then 1F · (x1, . . . , xn) = (1 · x1, . . . , 1 · xn) = (x1, . . . , xn) since 1F · x = x ∀ x ∈ F.Thus 1Fα = α ∀ α ∈ V .

6) Let α = (x1, . . . , xn). Then (c1c2)α = ((c1c2)x1, . . . , (c1c2)xn) and since multiplication in F is associative this equals(c1(c2x1), . . . , c1(c2xn)) = c1(c2x1, . . . c2xn) = c1 · (c2α).

7) Let α = (x1, . . . , xn) and β = (y1, . . . , yn). Then c(α+β) = c(x1 +y1, . . . , xn +yn) = (c(x1 +y1), . . . , c(xn +yn)) and since multi-plication is distributive over addition in F this equals (cx1+cy1, . . . , cxn+xyn). This then equals (cx1, . . . , cxn)+(cy1, . . . , cyn) =

25

Page 28: Linear Algebra - Greg Grantgreggrant.org/hoffman_and_kunze.pdf · Linear Algebra Ho man & Kunze 2nd edition ... Section 1.2: Systems of Linear Equations 3 Solution: We must check

26 Chapter 2: Vector Spaces

c(x1, . . . , xn) + c(y1, . . . , yn) = cα + cβ. Thus c(α + β) = cα + cβ.

8) Let α = (x1, . . . , xn). Then (c1 + c2)α = ((c1 + c2)x1, . . . , (c1 + c2)xn) and since multiplication distributes over addition inF this equals (c1x1 + c2x1, . . . , c1xn + c2xn) = (c1x1, . . . , c1xn) + (c2x1, . . . c2xn) = c1(x1, . . . , xn) + c2(x1, . . . , xn) = c1α + c2α.Thus (c1 + c2)α = c1α + c2α.

Exercise 2: If V is a vector space over the field F, verify that

(α1 + α2) + (α3 + α4) = [α2 + (α3 + α1)] + α4

for all vectors α1, α2, α3 and α4 in V .

Solution: This follows associativity and commutativity properties of V:

(α1 + α2) + (α3 + α4)

= (α2 + α1) + (α3 + α4)

= α2 + [α1 + (α3 + α4)]

= α2 + [(α1 + α3) + α4]

= [α2 + (α1 + α3)] + α4

= [α2 + (α3 + α1)] + α4.

Exercise 3: If C is the field of complex numbers, which vectors in C3 are linear combinations of (1, 0,−1), (0, 1, 1), and(1, 1, 1)?

Solution: If we make a matrix out of these three vectors

A =

1 0 10 1 1−1 1 1

then if we row-reduce the augmented matrix 1 0 1 1 0 0

0 1 1 0 1 0−1 1 1 0 0 1

we get 1 0 0 0 1 −1

0 1 0 −1 2 −10 0 1 1 −1 1

.Therefore the matrix is invertible and AX = Y has a solution X = A−1Y for any Y . Thus any vector Y ∈ C3 can be written as alinear combination of the three vectors. Not sure what the point was of making the base field C.

Exercise 4: Let V be the set of all pairs (x, y) of real numbers, and let F be the field of real numbers. Define

(x, y) + (x1, y1) = (x + x1, y + y1)

c(x, y) = (cx, y).

Is V , with these operations, a vector space over the field of real numbers?

Page 29: Linear Algebra - Greg Grantgreggrant.org/hoffman_and_kunze.pdf · Linear Algebra Ho man & Kunze 2nd edition ... Section 1.2: Systems of Linear Equations 3 Solution: We must check

Section 2.1: Vector Spaces 27

Solution: No it is not a vector space because (0, 2) = (0, 1) + (0, 1) = 2(0, 1) = (2 · 0, 1) = (0, 1). Thus we must have(0, 2) = (0, 1) which implies 1 = 2 which is a contradiction in the field of real numbers.

Exercise 5: On Rn, define two operationsα ⊕ β = α − β

c · α = −cα.

The operations on the right are the usual ones. Which of the axioms for a vector space are satisfied by (Rn,⊕, ·)?

Solution:

1) ⊕ is not commutative since (0, . . . , 0)⊕ (1, . . . , 1) = (−1, . . . ,−1) while (1, . . . , 1)⊕ (0, . . . , 0) = (1, . . . , 1). And (1, . . . , 1) ,(−1, . . . ,−1).

2) ⊕ is not associative since ((1, . . . , 1) ⊕ (1, . . . , 1)) ⊕ (2, . . . , 2) = (0, . . . , 0) ⊕ (2, . . . , 2) = (−2, . . . ,−2) while (1, . . . , 1) ⊕((1, . . . , 1) ⊕ (2, . . . , 2)) = (1, . . . , 1) ⊕ (−1, . . . ,−1) = (2, . . . , 2).

3) There does exist a right additive identity, i.e. a vector 0 that satisfies α + 0 = α for all α. The vector β = (0, . . . , 0) satisfiesα + β = α for all α. And if β′ = (b1, . . . , bn) also satisfies (x1, . . . , xn) + β′ = (x1, . . . , xn) then xi − bi = xi for all i and thusbi = 0 for all i. Thus β = (0, . . . , 0) is unique with respect to the property α + β = α for all α.

4) There do exist right additive inverses. For the vector α = (x1, . . . , xn) clearly only α itself satisfies α ⊕ α = (0, . . . , 0).

5) The element 1 does not satisfy 1·α = α for any non-zero α since otherwise we would have 1·(x1, . . . , xn) = (−x1, . . . ,−xn) =

(x1, . . . , xn) only if xi = 0 for all i.

6) The property (c1c2) · α = c1 · (c2 · α) does not hold since (c1c2)α = (−c1c2)α while c1(c2α) = c1(−c2α) = (−c1(−c2α)) =

+c1c2α. Since c1c2 , −c1c2 for all c1, c2 they are not always equal.

7) It does hold that c · (α ⊕ β) = c · α ⊕ c · β. Firstly, c · (α ⊕ β) = c · (α − β) = −c(α − β) = −cα + cβ. And secondlyc · α ⊕ c · β = (−cα) ⊕ (−cβ) = −cα − (−cβ) = −cα + cβ. Thus they are equal.

8) It does not hold that (c1 + c2) · α = (c1 · α) ⊕ (c2 · α). Firstly, (c1 + c2) · α = −(c1 + c2)α = −c1α − c2α. Secondly,c1 · α ⊕ c2 · α = (−c1 · α) ⊕ (−c2 · α) = −c1α + c2α. Since −c1α − c2α , −c1α − c2α for all c1, c2 they are not equal.

Exercise 6: Let V be the set of all complex-valued functions f on the real line such that (for all t in R)

f (−t) = f (t).

The bar denotes complex conjugation. Show that V , with the operations

( f + g)(t) = f (t) + g(t)

(c f )(t) = c f (t)

is a vector space over the field of real numbers. Give an example of a function in V which is not real-valued.

Solution: We will use the basic fact that a + b = a + b and ab = a · b.

Before we show V satisfies the eight properties we must first show vector addition and scalar multiplication as defined are ac-tually well-defined in the sense that they are indeed operations on V . In other words if f and g are two functions in V then wemust show that f + g is in V . In other words if f (−t) = f (t) and g(−t) = g(t) then we must show that ( f + g)(−t) = ( f + g)(t).

Page 30: Linear Algebra - Greg Grantgreggrant.org/hoffman_and_kunze.pdf · Linear Algebra Ho man & Kunze 2nd edition ... Section 1.2: Systems of Linear Equations 3 Solution: We must check

28 Chapter 2: Vector Spaces

This is true because ( f + g)(−t) = f (−t) + g(−t) = f (t) + g(t) = ( f (t) + g(t) = ( f + g)(t).

Similary, if c ∈ R, (c f )(−t) = c f (−t) = c f (t) = c f (t) since c = c when c ∈ R.

Thus the operations are well defined. We now show the eight properties hold:

1) Addition on functions in V is defined by adding in C to the values of the functions in C. Thus since C is commutative,addition in V inherits this commutativity.

2) Similar to 1, since C is associative, addition in V inherits this associativity.

3) The zero function g(t) = 0 is in V since −0 = 0. And g satisfies f +g = f for all f ∈ V . Thus V has a right additive identity.

4) Let g be the function g(t) = − f (t). Then g(−t) = − f (−t) = − f (t) = − f (t) = g(t). Thus g ∈ V . And ( f + g)(t) = f (t) + g(t) =

f (t) − f (t) = 0. Thus g is a right additive inverse for f .

5) Clearly 1 · f = f since 1 is the multiplicative identity in R.

6) As before, associativity in C implies (c1c2) f = c1(c2 f ).

7) Similarly, the distributive property in C implies c( f + g) = c f + cg.

8) Similarly, the distributive property in C implies (c1 + c2) f = c1 f + c2 f .

An example of a function in V which is not real valued is f (x) = ix. Since f (1) = i f is not real-valued. And f (−x) = −ix = ixsince x ∈ R, so f ∈ V .

Exercise 7: Let V be the set of pairs (x, y) of real numbers and let F be the field of real numbers. Define

(x, y) + (x1, y1) = (x + x1, 0)

c(x, y) = (cx, 0).

Is V , with these operations, a vector space?

Solution: This is not a vector space because there would have to be an additive identity element (a, b) which has the propertythat (a, b) + (x, y) = (x, y) for all (x, y) ∈ V . But this is impossible, because (a, b) + (0, 1) = (a, 0) , (0, 1) no matter what (a, b)is. Thus V does not satisfy the third requirement of having an additive identity element.

Section 2.2: SubspacesPage 39, typo in Exercise 3. It says R5, should be R4.

Exercise 1: Which of the following sets of vectors α = (a1, . . . , an) in Rn are subspaces of Rn (n ≥ 3)?

(a) all α such that a1 ≥ 0;

(b) all α such that a1 + 3a2 = a3;

(c) all α such that a2 = a21;

(d) all α such that a1a2 = 0;

Page 31: Linear Algebra - Greg Grantgreggrant.org/hoffman_and_kunze.pdf · Linear Algebra Ho man & Kunze 2nd edition ... Section 1.2: Systems of Linear Equations 3 Solution: We must check

Section 2.2: Subspaces 29

(e) all α such that a2 is rational.

Solution:

(a) This is not a subspace because for (1, . . . , 1) the additive inverse is (−1, . . . ,−1) which does not satisfy the condition.

(b) Suppose (a1, a2, a3, . . . , an) and (b1, b2, b3, . . . , bn) satisfy the condition and let c ∈ R. By Theorem 1 (page 35) we mustshow that c(a1, a2, a3, . . . , an)+(b1, b2, b3, . . . , bn) = (ca1+b1, . . . , can+bn) satisfies the condition. Now (ca1+b1)+3(ca2+b2) =

c(a1 + 3a2) + (b1 + 3b2) = c(a3) + (b3) = ca3 + b3. Thus it does satisfy the condition so V is a vector space.

(c) This is not a vector space because (1, 1) satisfies the condition since 12 = 1, but (1, 1, . . . ) + (1, 1, . . . ) = (2, 2, . . . ) and(2, 2, . . . ) does not satisfy the condition because 22 , 2.

(d) This is not a subspace. (1, 0, . . . ) and (0, 1, . . . ) both satisfy the condition, but their sum is (1, 1, . . . ) which does not satisfythe condition.

(e) This is not a subspace. (1, 1, . . . , 1) satisfies the condition, but π(1, 1, . . . , 1) = (π, π, . . . , π) does not satisfy the condition.

Exercise 2: Let V be the (real) vector space of all functions f from R into R. Which of the following sets of functions aresubspaces of V?

(a) all f such that f (x2) = f (x)2;

(b) all f such that f (0) = f (1);

(c) all f such that f (3) = 1 + f (−5);

(d) all f such that f (−1) = 0;

(e) all f which are continuous.

Solution:

(a) Not a subspace. Let f (x) = x and g(x) = x2. Then both satisfy the condition: f (x2) = x2 = ( f (x))2 and g(x2) = (x2)2 =

(g(x))2. But ( f + g)(x) = x + x2 and ( f + g)(x2) = x2 + x4 while [( f + g)(x)]2 = (x + x2)2 = x4 + 2x3 + x2. These are not equalpolynomials so the condition does not hold for f + g.

(b) Yes a subspace. Suppose f and g satisfy the property. Let c ∈ R. Then (c f +g)(0) = c f (0)+g(0) = c f (1)+g(1) = (c f +g)(1).Thus (c f + g)(0) = (c f + g)(1). By Theorem 1 (page 35) the set of all such functions constitute a subspace.

(c) Not a subspae. Let f (x) be the function defined by f (3) = 1 and f (x) = 0 for all x , 3. Let g(x) be the function definedby g(−5) = 0 and g(x) = 1 for all x , −5. Then both f and g satisfy the condition. But ( f + g)(3) = f (3) + g(3) = 1 + 1 = 2,while 1 + ( f + g)(−5) = 1 + f (−5) + g(−5) = 1 + 0 + 0 = 1. Since 1 , 2, f + g does not satisfy the condition.

(d) Yes a subspace. Suppose f and g satisfy the property. Let c ∈ R. Then (c f + g)(−1) = c f (−1) + g(−1) = c · 0 + 0 = 0.Thus (c f + g)(−1) = 0. By Theorem 1 (page 35) the set of all such functions constitute a subspace.

(e) Yes a subspace. Let f and g be continuous functions from R to R and let c ∈ R. Then we know from basic results of realanalysis that the sum and product of continuous functions are continuous. Since the function c 7→ c is continuous as wellas f and g, it follows that c f +g is continuous. By Theorem 1 (page 35) the set of all cotinuous functions constitute a subspace.

Exercise 3: Is the vector (3,−1, 0,−1) in the subspace of R5 (sic) spanned by the vectors (2,−1, 3, 2), (−1, 1, 1,−3), and(1, 1, 9,−5)?

Page 32: Linear Algebra - Greg Grantgreggrant.org/hoffman_and_kunze.pdf · Linear Algebra Ho man & Kunze 2nd edition ... Section 1.2: Systems of Linear Equations 3 Solution: We must check

30 Chapter 2: Vector Spaces

Solution: I assume they meant R4. No, (3,−1, 0,−1) is not in the subspace. If we row reduce the augmented matrix2 −1 1 3−1 1 1 −13 1 9 02 −3 −5 −1

we obtain

1 0 2 20 1 3 10 0 0 −70 0 0 −2

.The two bottom rows are zero rows to the left of the divider, but the values to the right of the divider in those two rows arenon-zero. Thus the system does not have a solution (see comments bottom of page 24 and top of page 25).

Exercise 4: Let W be the set of all (x1, x2, x3, x4, x5) in R5 which satisfy

2x1 − x2 +43

x3 − x4 = 0

x1 +23

x3 − x5 = 0

9x1 − 3x2 + 6x3 − 3x4 − 3x5 = 0.

Find a finite set of vectors which spans W.

Solution: The matrix of the system is 2 −1 4/3 −1 01 0 2/3 0 −19 −3 6 −3 −3

.Row reducing to reduced echelon form gives 1 0 2/3 0 −1

0 1 0 1 −20 0 0 0 0

.Thus the system is equivalent to

x1 + 2/3x3 − x5 = 0

x2 + x4 − 2x5 = 0.

Thus the system is parametrized by (x3, x4, x5). Setting each equal to one and the other two to zero (as in Example 15, page42), in turn, gives the three vectors (−2/3, 0, 1, 0, 0), (0,−1, 0, 1, 0) and (1, 2, 0, 0, 1). These three vectors therefore span W.

Exercise 5: Let F be a field and let n be a positive integer (n ≥ 2). Let V be the vector space of all n × n matrices over F.Which of the following sets of matrices A in V are subspaces of V?

(a) all invertible A;

(b) all non-invertible A;

(c) all A such that AB = BA, where B is some fixed matrix in V;

(d) all A such that A2 = A.

Page 33: Linear Algebra - Greg Grantgreggrant.org/hoffman_and_kunze.pdf · Linear Algebra Ho man & Kunze 2nd edition ... Section 1.2: Systems of Linear Equations 3 Solution: We must check

Section 2.2: Subspaces 31

Solution:

(a) This is not a subspace. Let A =

[1 00 1

]and let B =

[−1 00 −1

]. Then both A and B are invertible, but A+B =

[0 00 0

]which is not invertible. Thus the subset is not closed with respect to matrix addition. Therefore it cannot be a subspace.

(b) This is not a subspace. Let A =

[1 00 0

]and let B =

[0 00 1

]. Then neither A nor B is invertible, but A + B =

[1 00 1

]which is invertible. Thus the subset is not closed with respect to matrix addition. Therefore it cannot be a subspace.

(c) This is a subspace. Suppose A1 and A2 satisfy A1B = BA1 and A2B = BA2. Let c ∈ F be any constant. Then(cA1 + A2)B = cA1B + A2B = cBA1 + BA2 = B(cA1) + BA2 = B(cA1 + A2). Thus cA1 + A2 satisfy the criteria. ByTheorem 1 (page 35) the subset is a subspace.

(d) This is a subspace if F equals Z/2Z, otherwise it is not a subspace.

Suppose first that char(F) , 2. Let A =

[1 00 1

]. Then A2 = A. But A + A =

[2 00 2

]and

[2 00 2

]2

=

[4 00 4

],[

2 00 2

]. Thus A + A does not satisfy the criteria so the subset cannot be a subspace.

Suppose now that F = Z/2Z. Suppose A and B both satisfy A2 = A and B2 = B. Let c ∈ F be any scalar. Then(cA + B)2 = c2A2 + 2cAB + B2. Now 2 = 0 so this reduces to c2A2 + B2. If c = 0 then this reduces to B2 which equalsB, if c = 1 then this reduces to A2 + B2 which equals A + B. In both cases (cA + B)2 = cA + B. Thus in this case by Theorem1 (page 35) the subset is a subspace.

Finally suppose char(F) = 2 but F is not Z/2Z. Then |F| > 2. The polynomial x2 − x = 0 has at most two solutions in F, so

∃ c ∈ F such that c2 , c. Consider the identity matrix I =

[1 00 1

]. Then I2 = I. If such matrices form a subspace then it

must be that cI is also in the subspace. Thus it must be that (cI)2 = cI. Which is equivalent to c2 = c, which contradicts theway c was chosen.

Exercise 6:

(a) Prove that the only subspaces of R1 are R1 and the zero subspace.

(b) Prove that a subspace of R2 is R2, or the zero subspace, or consists of all scalar multiples of some fixed vector in R2.(The last type of subspace is, intuitively, a straight line through the origin.)

(c) Can you describe the subspaces of R3?

Solution:

(a) Let V be a subspace of R1. Suppose ∃ v ∈ V with v , 0. Then v is a vector but it is also simply an element of R. Letα ∈ R. Then α = α

v · v where αv is a scalar in the base field R. Since cv ∈ V ∀ c ∈ R, it follows that α ∈ V . Thus we have

shown that if V , {0} then V = R1.

(b) We know the subsests {(0, 0)} (example 6a, page 35) and R2 (example 1, page 29) are subspaces of R2. Also for any vectorv in any vector space over any field F, the set {cv | c ∈ F} is a subspace (Theorem 3, page 37). Thus we will be done if weshow that if V is a subspace of R2 and there exists v1, v2 ∈ V such that v1 and v2 do not lie on the same line, then V = R2.Equivalently we must show that any vector w ∈ R2 can be written as a linear combination of v1 and v2 whenever v1 and v2 arenot co-linear. Equivalently, by Theorem 13 (iii) (page 23), it suffices to show that if v1 = (a, b) and v2 = (c, d) are not colinear,then the matrix A = [vT

1 vT2 ] is invertible. Suppose a , 0 and let x = c/a. Then xa = c, and since v1 and v2 are not colinear, it

follows that xb , d. Thus equivalently ad − bc , 0. It follows now from Exercise 1.6.8 pae 27 that if v1 and v2 not colinearthen the matrix AT is invertible. Finally AT is invertible implies A is invertible, since clearly (AT )−1 = (A−1)T . Similarly if

Page 34: Linear Algebra - Greg Grantgreggrant.org/hoffman_and_kunze.pdf · Linear Algebra Ho man & Kunze 2nd edition ... Section 1.2: Systems of Linear Equations 3 Solution: We must check

32 Chapter 2: Vector Spaces

a = 0 then it must be that b , 0 so we can make the same argument. So in all cases A is invertible.

(c) The subspaces are the zero subspace {0, 0, 0}, lines {cv | c ∈ R} for fixed v ∈ R3, planes {c1v1 + c2v2 | c1, c2 ∈ R} forfixed v1, v2 ∈ R

3 and the whole space R3. By Theorem 3 we know these all are subspaces, we just must show they are theonly subspaces. It suffices to show that if v1, v2 and v3 are not co-planar then the space generated by v1, v2, v3 is all of R3.Equivalently we must show if v1 and v2 are not co-linear, and v3 is not in the plane generated by v1, v2 then any vector w ∈ R3

can be written as a linear combination of v1, v2, v3. Equivalently, by Theorem 13 (iii) (page 23), it suffices to show the matrixA = [v1 v2 v3] is invertible. A is invertible⇔ AT is invertible, since clearly (AT )−1 = (A−1)T . Now v3 is in the plane generatedby v1, v2 ⇔ v3 can be written as a linear combination of v1 and v2 ⇔ AT is row equivalent to a matrix with one of its rowsequal to all zeros (this follows from Theorem 12, page 23)⇔ AT is not invertible. Thus v3 is not in the plane generated byv1, v2 ⇔ A is invertible.

Exercise 7: Let W1 and W2 be subspaces of a vector space V such that the set-theoretic union of W1 and W2 is also a subspace.Prove that one of the spaces Wi is contained in the other.

Solution: Assume the space generated by W1 and W2 is equal to their set-theoretic union W1 ∪W2. Suppose W1 * W2 andW2 * W1. We wish to derive a contradiction. So suppose ∃ w1 ∈ W1 \ W2 and ∃ w2 ∈ W2 \ W1. Consider w1 + w2. Byassumption this is in W1 ∪W2, so ∃ w′1 ∈ W1 such that w1 + w2 = w′1 or ∃ w′2 ∈ W2 such that w1 + w2 = w′2. If the former, thenw2 = w′1 − w1 ∈ W1 which contradicts the assumption that w2 < W1. Likewise the latter implies the contradiction w1 ∈ W2.Thus we are done.

Exercise 8: Let V be the vector space of all functions from R into R; let Ve be the susbset of even functions, f (−x) = f (x);let Vo be the subset of odd functions, f (−x) = − f (x).

(a) Prove that Ve and Vo are subspaces of V .

(b) Prove that Ve + Vo = V .

(c) Prove that Ve ∩ Vo = {0}.

Solution:

(a) Let f , g ∈ Ve and c ∈ R. Let h = c f + g. Then h(−x) = c f (−x) + g(−x) = c f (x) + g(x) = h(x). So h ∈ Ve. By Theorem 1(page 35) Ve is a subspace. Now let f , g ∈ Vo and c ∈ R. Let h = c f +g. Then h(−x) = c f (−x)+g(−x) = −c f (x)−g(x) = −h(x).So h ∈ Vo. By Theorem 1 (page 35) Vo is a subspace.

(b) Let f ∈ V . Let fe(x) =f (x)+ f (−x)

2 and fo =f (x)− f (−x)

2 . Then fe is even and fo is odd and f = fe + fo. Thus we have writtenf as g + h where g ∈ Ve and h ∈ Vo.

(c) Let f ∈ Ve∩Vo. Then f (−x) = − f (x) and f (−x) = f (x). Thus f (x) = − f (x) which implies 2 f (x) = 0 which implies f = 0.

Exercise 9: Let W1 and W2 be subspaces of a vector space V such that W1 + W2 = V and W1 ∩W2 = {0}. Prove that for eachvector α in V there are unique vectors α1 in W1 and α2 in W2 such that α = α1 + α2.

Solution: Let α ∈ W. Suppose α = α1 + α2 for αi ∈ Wi and α = β1 + β2 for βi ∈ Wi. Then α1 + α2 = β1 + β2 which impliesα1 − β1 = β2 − α2. Thus α1 − β1 ∈ W1 and α1 − β1 ∈ W2. Since W1 ∩W2 = {0} it follows that α1 − β1 = 0 and thus α1 = β1.Similarly, β2 − α2 ∈ W1 ∩W2 so also α2 = β2.

Section 2.3: Bases and DimensionExercise 1: Prove that if two vectors are linearly dependent, one of them is a scalar multiple of the other.

Page 35: Linear Algebra - Greg Grantgreggrant.org/hoffman_and_kunze.pdf · Linear Algebra Ho man & Kunze 2nd edition ... Section 1.2: Systems of Linear Equations 3 Solution: We must check

Section 2.3: Bases and Dimension 33

Solution: Suppose v1 and v2 are linearly dependent. If one of them, say v1, is the zero vector then it is a scalar multiple of theother one v1 = 0 · v2. So we can assume both v1 and v2 are non-zero. Then if ∃ c1, c2 such that c1v1 + c2v2 = 0, both c1 and c2must be non-zero. Therefore we can write v1 = − c2

c1v2.

Exercise 2: Are the vectorsα1 = (1, 1, 2, 4), α2 = (2,−1,−5, 2)

α3 = (1,−1,−4, 0), α4 = (2, 1, 1, 6)

linearly independent in R4?

Solution: By Corollary 3, page 46, it suffices to determine if the matrix whose rows are the αi’s is invertible. By Theorem 12(ii) we can do this by row reducing the matrix

1 1 2 42 −1 −5 21 −1 −4 02 1 1 6

.

1 1 2 42 −1 −5 21 −1 −4 02 1 1 6

1 1 2 40 −3 −9 −60 −2 −6 −40 −1 −3 −2

swap→

rows

1 1 2 40 −1 −3 −20 −3 −9 −60 −2 −6 −4

1 1 2 40 1 3 20 −3 −9 −60 −2 −6 −4

1 1 2 40 1 3 20 0 0 00 0 0 0

Thus the four vectors are not linearly independent.

Exercise 3: Find a basis for the subspace of R4 spanned by the four vectors of Exercise 2.

Solution: In Section 2.5, Theorem 9, page 56, it will be proven that row equivalent matrices have the same row space. Theproof of this is almost immediate so there seems no easier way to prove it than to use that fact. If you multiply a matrix Aon the left by another matrix P, the rows of the new matrix PA are linear combinations of the rows of the original matrix.Thus the rows of PA generate a subspace of the space generated by the rows of A. If P is invertible, then the two spacesmust be contained in each other since we can go backwards with P−1. Thus the rows of row-equivalent matrices generate thesame space. Thus using the row reduced form of the matrix in Exercise 2, it must be that the space is two dimensoinal andgenerated by (1, 1, 2, 4) and (0, 1, 3, 2).

Exercise 4: Show that the vectors

α1 = (1, 0,−1), α2 = (1, 2, 1), α3 = (0,−3, 2)

form a basis for R3. Express each of the standard basis vectors as linear combinations of α1, α2, and α3.

Solution: By Corollary 3, page 46, to show the vectors are linearly independent it suffices to show the matrix whose rows arethe αi’s is invertible. By Theorem 12 (ii) we can do this by row reducing the matrix

A =

1 0 −11 2 10 −3 2

. 1 0 −1

1 2 10 −3 2

→ 1 0 −1

0 2 20 −3 2

→ 1 0 −1

0 1 10 −3 2

→ 1 0 −1

0 1 10 0 5

→ 1 0 −1

0 1 10 0 1

→ 1 0 0

0 1 00 0 1

.Now to write the standard basis vectors in terms of these vectors, by the discussion at the bottom of page 25 through page 26,we can row-reduce the augmented matrix 1 0 −1 1 0 0

1 2 1 0 1 00 −3 2 0 0 1

.

Page 36: Linear Algebra - Greg Grantgreggrant.org/hoffman_and_kunze.pdf · Linear Algebra Ho man & Kunze 2nd edition ... Section 1.2: Systems of Linear Equations 3 Solution: We must check

34 Chapter 2: Vector Spaces

This gives 1 0 −1 1 0 01 2 1 0 1 00 −3 2 0 0 1

1 0 −1 1 0 00 2 2 −1 1 00 −3 2 0 0 1

1 0 −1 1 0 00 1 1 −1/2 1/2 00 −3 2 0 0 1

1 0 −1 1 0 00 1 1 −1/2 1/2 00 0 5 −3/2 3/2 1

1 0 −1 1 0 00 1 1 −1/2 1/2 00 0 1 −3/10 3/10 1/5

1 0 0 7/10 3/10 1/50 1 0 −1/5 1/5 −1/50 0 1 −3/10 3/10 1/5

.Thus if

P =

7/10 3/10 1/5−1/5 1/5 −1/5−3/10 3/10 1/5

then PA = I, so we have

710α1 +

310α2 +

15α3 = (1, 0, 0)

−15α1 +

15α2 −

15α3 = (0, 1, 0)

−3

10α1 +

310α2 +

15α3 = (0, 0, 1).

Exercise 5: Find three vectors in R3 which are linearly dependent, and are such that any two of them are linearly independent.

Solution: Let v1 = (1, 0, 0), v2 = (0, 1, 0) and v3 = (1, 1, 0). Then v1 + v2 − v3 = (0, 0, 0) so they are linearly dependent. Weknow v1 and v2 are linearly independent as they are two of the standard basis vectors (see Example 13, page 41). Supposeav1 + bv3 = 0. Then (a + b, b, 0) = (0, 0, 0). The second coordinate implies b = 0 and then the first coordinate in turn impliesa = 0. Thus v1 and v3 are linearly independent. Analogously v2 and v3 are linearly independent.

Exercise 6: Let V be the vector space of all 2 × 2 matrices over the field F. Prove that V has dimension 4 by exhibiting abasis for V which has four elements.

Solution: Let

v11 =

[1 00 0

], v12 =

[0 10 0

]

v21 =

[0 01 0

], v22 =

[0 00 1

]

Page 37: Linear Algebra - Greg Grantgreggrant.org/hoffman_and_kunze.pdf · Linear Algebra Ho man & Kunze 2nd edition ... Section 1.2: Systems of Linear Equations 3 Solution: We must check

Section 2.3: Bases and Dimension 35

Suppose av11 + bv12 + cv21 + dv22 =

[0 00 0

]. Then

[a bc d

]=

[0 00 0

],

from which it follows immediately that a = b = c = d = 0. Thus v11, v12, v21, v22 are linearly independent.

Now let[

a bc d

]be any 2 × 2 matrix. Then

[a bc d

]= av11 + bv12 + cv21 + dv22. Thus v11, v12, v21, v22 span the space of

2 × 2 matrices.

Thus v11, v12, v21, v22 are both linearly independent and they span the space of all 2 × 2 matrices. Thus v11, v12, v21, v22constitue a basis for the space of all 2 × 2 matrices.

Exercise 7: Let V be the vector space of Exercise 6. Let W1 be the set of matrices of the form[x −xy z

]and let W2 be the set of matrices of the form [

a b−a c

].

(a) Prove that W1 and W2 are subspaces of V .

(b) Find the dimension of W1, W2, W1 + W2, and W1 ∩W2.

Solution:

(a) Let A =

[x −xy z

]and B =

[x′ −x′

y′ z′

]be two elements of W1 and let c ∈ F. Then

cA + B =

[cx + x′ −cx − x′

cy + y′ cz + z′

]=

[a −au v

]where a = cx + x′, u = cy + y′ and v = cz + z′. Thus cA + B is in the form of an element of W1. Thus cA + B ∈ W1. ByTheorem 1 (page 35) W1 is a subspace.

Now let A =

[a b−a d

]and B =

[a′ b′

−a′ d′

]be two elements of W1 and let c ∈ F.Then

cA + B =

[ca + a′ cb + b′

−ca − a′ cd + d′

]=

[x y−x z

]where x = ca + a′, y = cb + b′ and z = cd + d′. Thus cA + B is in the form of an element of W2. Thus cA + B ∈ W2. ByTheorem 1 (page 35) W2 is a subspace.

(b) Let

A1 =

[1 −10 0

], A2 =

[0 01 0

], A2 =

[0 00 1

].

Then A1, A2, A3 ∈ W1 and

c1A1 + c2A2 + c3A3 =

[c1 −c1c2 c3

]=

[0 00 0

]

Page 38: Linear Algebra - Greg Grantgreggrant.org/hoffman_and_kunze.pdf · Linear Algebra Ho man & Kunze 2nd edition ... Section 1.2: Systems of Linear Equations 3 Solution: We must check

36 Chapter 2: Vector Spaces

implies c1 = c2 = c3 = 0. So A1, A2, A3 are linearly independent. Now let A =

[x −xy z

]be any element of W1. Then

A = xA1 + yA2 + zA3. Thus A1, A2, A3 span W1. Thus {A1, A2, A3} form a basis for W1. Thus W1 has dimension three.

Let

A1 =

[1 0−1 0

], A2 =

[0 10 0

], A2 =

[0 00 1

].

Then A1, A2, A3 ∈ W2 and

c1A1 + c2A2 + c3A3 =

[c1 c2−c1 c3

]=

[0 00 0

]implies c1 = c2 = c3 = 0. So A1, A2, A3 are linearly independent. Now let A =

[x y−x z

]be any element of W2. Then

A = xA1 + yA2 + zA3. Thus A1, A2, A3 span W2. Thus {A1, A2, A3} form a basis for W2. Thus W2 has dimension three.

Let V be the space of 2 × 2 matrices. We showed in Exercise 6 that the dim(V) = 4. Now W1 ⊆ W1 + W2 ⊆ V . Thus by

Corollary 1, page 46, 3 ≤ dim(W1 + W2) ≤ 4. Let A =

[1 0−1 0

]. Then A ∈ W2 and A < W1. Thus W1 + W2 is strictly bigger

than W1. Thus 4 ≥ dim(W1 + W2) > dim(W1) = 3. Thus dim(W1 + W2) = 4.

Suppose A =

[a bc d

]is in W1 ∩ W2. Then A ∈ W1 ⇒ a = −b and A ∈ W2 ⇒ a = −c. So A =

[a −a−a b

]. Let

A1 =

[1 −1−1 0

], A2 =

[0 00 1

]. Suppose aA1 + bA2 = 0. Then

[a −a−a b

]=

[0 00 0

],

which implies a = b = 0. Thus A1 and A2 are linearly independent. Let A =

[a −a−a b

]∈ W1 ∩W2. Then A = aA1 + bA2.

So A1, A2 span W1 ∩W2. Thus {A1, A2} is a basis for W1 ∩W2. Thus dim(W1 ∩W2) = 2.

Exercise 8: Again let V be the space of 2×2 matrices over F. Find a basis {A1, A2, A3, A4} for V such that A2j = A j for each j.

Solution: Let V be the space of all 2 × 2 matrices. Let

A1 =

[1 00 1

], A2 =

[0 00 1

]

A3 =

[1 10 0

], A4

[0 01 1

]Then A2

i = Ai ∀ i. Now

aA1 + bA2 + cA3 + dA4 =

[a + c c

d b + d

]=

[0 00 0

]implies c = d = 0 which in turn implies a = b = 0. Thus A1, A2, A3, A4 are linearly independent. Thus they span a subspaceof A of dimension four. But by Exercise 6, A also has dimension four. Thus by Corollary 1, page 46, the subspace spannedby A1, A2, A3, A4 is the entire space. Thus {A1, A2, A3, A4} is a basis.

Exercise 9: Let V be a vector space over a subfield F of the complex numbers. Suppose α, β, and γ are linearly independentvectors in V . Prove that (α + β), (β + γ), and (γ + α) are linearly independent.

Page 39: Linear Algebra - Greg Grantgreggrant.org/hoffman_and_kunze.pdf · Linear Algebra Ho man & Kunze 2nd edition ... Section 1.2: Systems of Linear Equations 3 Solution: We must check

Section 2.3: Bases and Dimension 37

Solution: Suppose a(α+ β) + b(β+ γ) + c(γ + α) = 0. Rearranging gives (a + c)α+ (a + b)β+ (b + c)γ = 0. Since α, β, and γare linearly independent it follows that a + c = a + b = b + c = 0. This gives a system of equations in a, b, c with matrix 1 1 0

1 0 10 1 1

.This row-reduces as follows: 1 1 0

1 0 10 1 1

→ 1 1 0

0 −1 10 1 1

→ 1 1 0

0 1 −10 1 1

→ 1 0 1

0 1 −10 0 2

→ 1 0 1

0 1 −10 0 1

→ 1 0 0

0 1 00 0 1

.Since this row-reduces to the identity matrix, by Theorem 7, page 13, the only solution is a = b = c = 0. Thus (α+β), (β+γ),and (γ + α) are linearly independent.

Exercise 10: Let V be a vector space over the field F. Suppose there are a finite number of vectors α1, . . . , αr in V whichspan V . Prove that V is finite-dimensional.

Solution: If any αi’s are equal to zero then we can remove them from the set and the remaining αi’s still span V . Thus we canassume WLOG that αi , 0 ∀ i. If α1, . . . , αr are linearly independent, then {α1, . . . , αr} is a basis and dim(V) = r < ∞. On theother hand if α1, . . . , αr are linearly dependent, then ∃ c1, . . . , cr ∈ F, not all zero, such that c1α1 + · · · + crαr = 0. SupposeWLOG that cr , 0. Then αr = − c1

crα1 − · · · −

cr−1crαr−1. Thus αr is in the subspace spanned by α1, . . . , αr−1. Thus α1, . . . , αr−1

spans V . If α1, . . . , αr−1 are linearly independent then {α1, . . . , αr−1} is a basis and dim(V) = r − 1 < ∞. If α1, . . . , αr−1 arelinearly dependent then arguing as before (with possibly re-indexing) we can produce α1, . . . , αr−2 which span V . Continuingin this way we must eventually arrive at a linearly independent set, or arrive at a set that consists of a single element, that stillspans V . If we arrive at a single element v1 then {v1} is linearly independent since cv1 = 0⇒ c = 0 (see comments after (2-9)page 31). Thus we must eventually arrive at a finite set that is spans and is linearly independent. Thus we must eventuallyarrive at a finite basis, which implies dim(V) < ∞.

Exercise 11: Let V be the set of all 2 × 2 matrices A with complex entries which satisfy A11 + A22 = 0.

(a) Show that V is a vector space over the field of real numbers, with the usual operations of matrix addition and multipli-cation of a matrix by a scalar.

(b) Find a basis for this vector space.

(c) Let W be the set of all matrices A in V such that A21 = −A12 (the bar denotes complex conjugation). Prove that W is asubspace of V and find a basis for W.

Solution: (a) It is clear from inspection of the definition of a vector space (pages 28-29) that a vector space over a field F isa vector space over every subfield of F, because all properties (e.g. commutativity and associativity) are inherited from theoperations in F. Let M be the vector space of all 2 × 2 matrices over C (M is a vector space, see example 2 page 29). Wewill show V is a subspace M as a vector space over C. It will follow from the comment above that V is a vector space over R.Now V is a subset of M, so using Theorem 1 (page 35) we must show whenever A, B ∈ V and c ∈ C then cA + B ∈ V . Let

A, B ∈ V . Write A =

[x yz w

]and B =

[x′ y′

z′ w′

]. Then

x + w = x′ + w′ = 0. (17)

cA + B =

[cx + x′ cy + y′

cz + z′ cw + w′

]To show cA + B ∈ V we must show (cx + x′) + (cw + w′) = 0. Rearranging the left hand side gives c(x + w) + (x′ + w′) whichequals zero by (17).

Page 40: Linear Algebra - Greg Grantgreggrant.org/hoffman_and_kunze.pdf · Linear Algebra Ho man & Kunze 2nd edition ... Section 1.2: Systems of Linear Equations 3 Solution: We must check

38 Chapter 2: Vector Spaces

(b) We can write the general element of V as

A =

[a + bi e + f ig + hi −a − bi

].

Let

v1 =

[1 00 −1

], v2 =

[i 00 −i

],

v3 =

[0 10 0

], v4 =

[0 i0 0

],

v5 =

[0 01 0

], v6 =

[0 0i 0

].

Then A = av1 + bv2 + ev3 + f v4 + gv5 + hv6 so v1, v2, v3, v4, v5, v6 span V . Suppose av1 + bv2 + ev3 + f v4 + gv5 + hv6 = 0. Then

av1 + bv2 + ev3 + f v4 + gv5 + hv6 =

[a + bi e + f ig + hi −a − bi

]=

[0 00 0

]implies a = b = c = d = e = f = g = h = 0 because a complex number u + vi = 0⇔ u = v = 0. Thus v1, v2, v3, v4, v5, v6 arelinearly independent. Thus {v1, . . . , v6} is a basis for V as a vector space over R, and dim(V) = 6.

(c) Let A, B ∈ W and c ∈ R. By Theorem 1 (page 35) we must show cA + B ∈ W. Write A =

[x y−y −x

]and B =[

x′ y′

−y′ −x′

], where x, y, x′, y′ ∈ C. Then

cA + B =

[cx + x′ cy + y′

−cy − y′ −cx − x′

].

Since −cy − y′ = −(cy + y′), it follows that cA + B ∈ W. Note that we definitely need c ∈ R for this to be true.

It remains to find a basis for W. We can write the general element of W as

A =

[a + bi e + f i−e + f i −a − bi

].

Let

v1 =

[1 00 −1

], v2 =

[i 00 −i

],

v3 =

[0 1−1 0

], v4 =

[0 ii 0

].

Then A = av1 + bv2 + ev3 + f v4 so v1, v2, v3, v4 span V . Suppose av1 + bv2 + ev3 + f v4 = 0. Then

av1 + bv2 + ev3 + f v4 =

[a + bi e + f i−e + f i −a − bi

]=

[0 00 0

]implies a = b = e = f = 0 because a complex number u + vi = 0⇔ u = v = 0. Thus v1, v2, v3, v4 are linearly independent.Thus {v1, . . . , v4} is a basis for V as a vector space over R, and dim(V) = 4.

Exercise 12: Prove that the space of m × n matrices over the field F has dimension mn, by exhibiting a basis for this space.

Solution: Let M be the space of all m × n matrices. Let Mi j be the matrix of all zeros except for the i, j-th place which isa one. We claim {Mi j | 1 ≤ i ≤ m, 1 ≤ j ≤ n} constitute a basis for M. Let A = (ai j) be an arbitrary marrix in M. Then

Page 41: Linear Algebra - Greg Grantgreggrant.org/hoffman_and_kunze.pdf · Linear Algebra Ho man & Kunze 2nd edition ... Section 1.2: Systems of Linear Equations 3 Solution: We must check

Section 2.4: Coordinates 39

A =∑

i j ai jMi j. Thus {Mi j} span M. Suppose∑

i j ai jMi j = 0. The left hand side equals the matrix (ai j) and this equals thezero matrix if and only if every ai j = 0. Thus {Mi j} are linearly independent as well. Thus the nm matrices constitute a basisand M has dimension mn.

Exercise 13: Discuss Exercise 9, when V is a vector space over the field with two elements described in Exercise 5, Section1.1.

Solution: If F has characteristic two then (α + β) + (β + γ) + (γ + α) = 2α + 2β + 2γ = 0 + 0 + 0 = 0 since in a field ofcharacteristic two, 2 = 0. Thus in this case (α + β), (β + γ) and (γ + α) are linearly dependent. However any two of them arelinearly independent. For example suppose a1(α + β) + a2(β + γ) = 0. The LHS equals a1α + a2γ + (a1 + a2)β. Since α, β, γare linearly independent, this is zero only if a1 = 0, a2 = 0 and a1 + a2 = 0. In particular a1 = a2 = 0, so α + β and β + γ arelinearly independent.

Exercise 14: Let V be the set of real numbers. Regard V as a vector space over the field of rational numbers, with the usualoperations. Prove that this vector space is not finite-dimensional.

Solution: We know that Q is countable and R is uncountable. Since the set of n-tuples of things from a countable set iscountable, Qn is countable for all n. Now, suppose {r1, . . . , rn} is a basis for R over Q. Then every element of R can be writtenas a1r1 + · · · + anrn. Thus we can map n-tuples of rational numbers onto R by (a1, . . . , an) 7→ a1r1 + · · · + anrn. Thus thecardinality of R must be less or equal to Qn. But the former is uncountable and the latter is countable, a contradiction. Thusthere can be no such finite basis.

Section 2.4: Coordinates

Exercise 1: Show that the vectorsα1 = (1, 1, 0, 0), α2 = (0, 0, 1, 1)

α3 = (1, 0, 0, 4), α4 = (0, 0, 0, 2)

form a basis for R4. Find the coordinates of each of the standard basis vectors in the ordered basis {α1, α2, α3, α4}.

Solution: Using Theorem 7, page 52, if we calculate the inverse of

P =

1 0 1 01 0 0 00 1 0 00 1 4 2

.then the columns of P−1 will give the coefficients to write the standard basis vectors in terms of the αi’s. We do this byrow-reducing the augmented matrix

1 0 1 0 1 0 0 01 0 0 0 0 1 0 00 1 0 0 0 0 1 00 1 4 2 0 0 0 1

.The left side must reduce to the identity whlie the right side transforms to the inverse of P. Row reduction gives

1 0 1 0 1 0 0 01 0 0 0 0 1 0 00 1 0 0 0 0 1 00 1 4 2 0 0 0 1

.

Page 42: Linear Algebra - Greg Grantgreggrant.org/hoffman_and_kunze.pdf · Linear Algebra Ho man & Kunze 2nd edition ... Section 1.2: Systems of Linear Equations 3 Solution: We must check

40 Chapter 2: Vector Spaces

1 0 1 0 1 0 0 00 0 −1 0 −1 1 0 00 1 0 0 0 0 1 00 1 4 2 0 0 0 1

.

1 0 1 0 1 0 0 00 1 0 0 0 0 1 00 0 −1 0 −1 1 0 00 1 4 2 0 0 0 1

.

1 0 1 0 1 0 0 00 1 0 0 0 0 1 00 0 −1 0 −1 1 0 00 0 4 2 0 0 −1 1

.

1 0 1 0 1 0 0 00 1 0 0 0 0 1 00 0 1 0 1 −1 0 00 0 4 2 0 0 −1 1

.

1 0 0 0 0 1 0 00 1 0 0 0 0 1 00 0 1 0 1 −1 0 00 0 0 2 −4 4 −1 1

.

1 0 0 0 0 1 0 00 1 0 0 0 0 1 00 0 1 0 1 −1 0 00 0 0 1 −2 2 −1/2 1/2

.Thus {α1, . . . , α4} is a basis. Call this basis β. Thus (1, 0, 0, 0) = α3 − 2α4, (0, 1, 0, 0) = α1 − α3 + 2α4, (0, 0, 1, 0) = α2 −

12α4

and (0, 0, 0, 1) = 12α4.

Thus [(1, 0, 0, 0)]β = (0, 0, 1,−2), [(0, 1, 0, 0)]β = (1, 0,−1, 2), [(0, 0, 1, 0)]β = (0, 1, 0,−1/2) and [(0, 0, 0, 1)]β = (0, 0, 0, 1/2).

Exercise 2: Find the coordinate matrix of the vector (1, 0, 1) in the basis of C3 consisting of the vectors (2i, 1, 0), (2,−1, 0),(0, 1 + i, 1 − i), in that order.

Solution: Using Theorem 7, page 52, the answer is P−1

101

where

P =

2i 2 01 −1 1 + i0 0 1 − i

.We find P−1 by row-reducing the augmented matrix 2i 2 0 1 0 0

1 −1 1 + i 0 1 00 0 1 − i 0 0 1

.The right side will transform into the P−1. Row reducing: 2i 2 0 1 0 0

1 −1 1 + i 0 1 00 0 1 − i 0 0 1

Page 43: Linear Algebra - Greg Grantgreggrant.org/hoffman_and_kunze.pdf · Linear Algebra Ho man & Kunze 2nd edition ... Section 1.2: Systems of Linear Equations 3 Solution: We must check

Section 2.4: Coordinates 41

1 −1 1 + i 0 1 02i 2 0 1 0 00 0 1 − i 0 0 1

1 −1 1 + i 0 1 00 2 + 2i 2 − 2i 1 −2i 00 0 1 − i 0 0 1

1 −1 1 + i 0 1 00 1 −i 1−i

4−1−i

2 00 0 1 − i 0 0 1

1 0 1 1−i

41−i2 0

0 1 −i 1−i4

−1−i2 0

0 0 1 − i 0 0 1

1 0 1 1−i

41−i2 0

0 1 −i 1−i4

−1−i2 0

0 0 1 0 0 1+i2

1 0 0 1−i

41−i2

−1−i2

0 1 0 1−i4

−1−i2

−1+i2

0 0 1 0 0 1+i2

Therefore

P−1

101

=

1−i4

1−i2

−1−i2

1−i4

−1−i2

−1+i2

0 0 1+i2

101

=

−1−3i

4−1+i

41+i2

Thus (1, 0, 1) = −1−3i

4 (2i, 1, 0) + −1+i4 (2,−1, 0) + 1+i

2 (0, 1 + i, 1 − i).

Exercise 3: Let B = {α1, α2, α3} be the ordered basis for R3 consisting of

α1 = (1, 0,−1), α2 = (1, 1, 1), α3 = (1, 0, 0).

What are the coordinates of the vector (a, b, c) in the ordered basis B?

Solution: Using Theorem 7, page 52, the answer is P−1

abc

where

P =

1 1 10 1 0−1 1 0

.We find P−1 by row-reducing the augmented matrix 1 1 1 1 0 0

0 1 0 0 1 0−1 1 0 0 0 1

.The right side will transform into the P−1. Row reducing: 1 1 1 1 0 0

0 1 0 0 1 0−1 1 0 0 0 1

.

Page 44: Linear Algebra - Greg Grantgreggrant.org/hoffman_and_kunze.pdf · Linear Algebra Ho man & Kunze 2nd edition ... Section 1.2: Systems of Linear Equations 3 Solution: We must check

42 Chapter 2: Vector Spaces

1 1 1 1 0 00 1 0 0 1 00 2 1 1 0 1

.→

1 0 1 1 −1 00 1 0 0 1 00 0 1 1 −2 1

.→

1 0 0 0 1 −10 1 0 0 1 00 0 1 1 −2 1

.Therefore,

P−1

abc

=

0 1 −10 1 01 −2 1

a

bc

=

b − cb

a − 2b + c

Thus the answer is

[(a, b, c)]B = (b − c, b, a − 2b + c).

Exercise 4: Let W be the subspace of C3 spanned by α1 = (1, 0, i) and α2 = (1 + i, 1, −1).

(a) Show that α1 and α2 form a basis for W.

(b) Show that the vectors β1 = (1, 1, 0) and β2 = (1, i, 1 + i) are in W and form another basis for W.

(c) What are the coordinates of α1 and α2 in the ordered basis {β1, β2} for W?

Solution: (a) To show α1 and α2 form a basis of the space they generate we must show they are linearly independent. In otherwords that aα1 + bα2 = 0 ⇒ a = b = 0. Equivalently we need to show neither is a multiple of the other. If α2 = cα1 thenfrom the second coordinate it follows that c = 0 which would imply α2 = (0, 0, 0), which it does not. So {α1, α2} is a basis forthe space they genearate.

(b) Since the first coordinate of both β1 and β2 is one, it’s clear that neither is a multiple of the other. So they generate a twodimensional subspace of C3. If we show β1 and β2 can be written as linear combinations of α1 and α2 then since the spacesgenerated by them both have dimension two, by Corollary 1, page 46, they must be equal. To show β1 and β2 can be writtenas linear combinations of α1 and α2 we row-reduce the augmented matrix 1 1 + i 1 1

0 1 1 ii −1 0 1 + i

.Row reduction follows: 1 1 + i 1 1

0 1 1 ii −1 0 1 + i

→ 1 1 + i 1 1

0 1 1 i0 −i −i 1

→ 1 0 −i 2 − i

0 1 1 i0 0 0 0

Thus β1 = −iα1 + α2 and β2 = (2 − i)α1 + iα2.

(c) We have to write the βi’s in terms of the αi’s, basically the opposite of what we did in part b. In this case we row-reducethe augmented matrix 1 1 1 1 + i

1 i 0 10 1 + i i −1

→ 1 1 1 1 + i

0 −1 + i −1 −i0 1 + i i −1

→ 1 1 1 1 + i

0 −1 + i −1 −i0 0 0 0

Page 45: Linear Algebra - Greg Grantgreggrant.org/hoffman_and_kunze.pdf · Linear Algebra Ho man & Kunze 2nd edition ... Section 1.2: Systems of Linear Equations 3 Solution: We must check

Section 2.4: Coordinates 43

1 1 1 1 + i0 1 1+i

2−1+i

2

0 0 0 0

1 0 1−i2

3+i2

0 1 1+i2

−1+i2

0 0 0 0

Thus α1 = 1−i

2 β1 + 1+i2 β2 and α2 = 3+i

2 β1 + −1+i2 β2. So finally, if B is the basis {β1, β2} then

[α1]B =

(1 − i

2,

1 + i2

)

[α2]B =

(3 + i

2,−1 + i

2

).

Exercise 5: Let α = (x1, x2) and β = (y1, y2) be vectors in R2 such that

x1y1 + x2y2 = 0, x21 + x2

2 = y21 + y2

2 = 1.

Prove thatB = {α, β} is a basis for R2. Find the coordinates of the vector (a, b) in the ordered basisB = {α, β}. (The conditionson α and β say, geometrically, that α and β are perpendicular and each has length 1.)

Solution: It suffices by Corollary 1, page 46, to show α and β are linearly indepdenent, because then they generate a subspaceof R2 of dimension two, which therefore must be all of R2. The second condition on x1, x2, y1, y2 implies that neither α nor βare the zero vector. To show two vectors are linearly independent we only need show neither is a non-zero scalar multiple ofthe other. Suppose WLOG that β = cα for some c ∈ R, and since neither vector is the zero vector, c , 0. Then y1 = cx1 andy2 = cx2. Thus the conditions on x1, x2, y1, y2 implies

0 = x1y1 + x2y2 = cx21 + cx2

2 = c(x21 + x2

2) = c · 1 = c.

Thus c = 0, a contradiction.

It remains to find the coordinates of the arbitrary vector (a, b) in the ordered basis {α, β}. To find the coordinates of (a, b) wecan row-reduce the augmented matrix [

x1 y1 ax2 y2 b

].

It cannot be that both x1 = x2 = 0 so assume WLOG that x1 , 0. Also it cannot be that both y1 = y2 = 0. Assume first thaty1 , 0. Since order matters we cannot assume y1 , 0 WLOG, so we must consider both cases. Then note that x1y1 + x2y2 = 0implies

x2y2

x1y1= −1 (18)

Thus if x1y2 − x2y1 = 0 then x2x1

=y2y1

from which (18) implies(

x2x1

)2= −1, a contradiction. Thus we can conclude that

x1y2 − x2y1 , 0. We use this in the following row reduction to be sure we are not dividing by zero.[x1 y1 ax2 y2 b

]→

[1 y1/x1 a/x1x2 y2 b

]→

[1 y1/x1 a/x10 y2 −

x2y1x1

b − x2ax1

]=

[1 y1/x1 a/x1

0 x1y2−x2y1x1

bx1−ax2x1

]

[1 y1/x1 a/x1

0 1 bx1−ax2x1y2−x2y1

]→

1 0 ay2−by1x1y2−x2y1

0 1 bx1−ax2x1y2−x2y1

Now if we substitute y1 = −x2y2/x1 into the numerator and denominator of ay2−by1

x1y2−x2y1and use x2

1 + x22 = 1 it simplifies to

ax1 + bx2. Similarly ay2−by1x1y2−x2y1

simplifies to ay1 + by2. So we get[1 0 ax1 + bx20 1 ay1 + by2

].

Page 46: Linear Algebra - Greg Grantgreggrant.org/hoffman_and_kunze.pdf · Linear Algebra Ho man & Kunze 2nd edition ... Section 1.2: Systems of Linear Equations 3 Solution: We must check

44 Chapter 2: Vector Spaces

Now assume y2 , 0 (and we continue to assume x1 , 0 since we assumed that WLOG). In this case

y1

y2= −

x2

x1(19)

So if x1y2 − x2y1 = 0 then x2y1x1y2

= 1. But then (19) implies(

x2x1

)2= −1 a contradition. So also in this case we can assume

x1y2 − x2y1 , 0 and so we can do the same row-reduction as before. Thus in all cases

(ax1 + bx2)α + (ay1 + by2)β = (a, b)

or equivalently(ax1 + bx2)(x1, x2) + (ay1 + by2)(y1, y2) = (a, b).

Exercise 6: Let V be the vector space over the complex numbers of all functions from R into C, i.e., the space of all complex-valued functions on the real line. Let f1(x) = 1, f2(x) = eix, f3(x) = e−ix.

(a) Prove that f1, f2, and f3 are linearly independent.

(b) Let g1(x) = 1, g2(x) = cos x, g3(x) = sin x. Find an invertible 3 × 3 matrix P such that

g j =

3∑i=1

Pi j fi.

Solution: Suppose a + beix + ce−ix = 0 as functions of x ∈ R. In other words a + beix + ce−ix = 0 for all x ∈ R. Let y = eix.Then y , 0 and a + by + c

y = 0 which implies ay + by2 + c = 0. This is at most a quadratic polynomial in y thus can be zerofor at most two values of y. But eix takes infinitely many different values as x varies in R, so ay + by2 + c cannot be zero forall y = eix, so this is a contradiction.

We know that eix = cos(x) + i sin(x). Thus e−ix = cos(x) − i sin(x). Adding these gives 2 cos(x) = eix + e−ix. Thuscos(x) = 1

2 eix + 12 e−ix. Subtracting instead of adding the equations gives eix − e−ix = 2i sin(x). Thus sin(x) = 1

2i eix − 1

2i e−ix or

equivalently sin(x) = − i2 eix + i

2 e−ix. Thus the requested matrix is

P =

1 0 00 1/2 −i/20 1/2 i/2

.Exercise 7: Let V be the (real) vector space of all polynomial functions from R into R of degree 2 or less, i.e., the space ofall functions f of the form

f (x) = c0 + c1x + c2x2.

Let t be a fixed real number and define

g1(x) = 1, g2(x) = x + t, g3(x) = (x + t)2.

Prove that B = {g1, g2, g3} is a basis for V . Iff (x) = c0 + c1x + c2x2

what are the coordinates of f in this ordered basis B?

Solution: We know V has dimension three (it follows from Example 16, page 43, that {1, x, x2} is a basis). Thus by Corollary2 (b), page 45, it suffices to show {g1, g2, g3} span V . We need to solve for u, v,w the equation

c2x2 + c1x + c0 = u + v(x + t) + w(x + t)2.

Rearrangingc2x2 + c1x + c0 = wx2 + (v + 2wt)x + (u + vt + wt2).

Page 47: Linear Algebra - Greg Grantgreggrant.org/hoffman_and_kunze.pdf · Linear Algebra Ho man & Kunze 2nd edition ... Section 1.2: Systems of Linear Equations 3 Solution: We must check

Section 2.6: Computations Concerning Subspaces 45

It follows thatw = c2

v = c1 − 2c2t

u = c0 − c1t + c2t2.

Thus {g1, g2, g3} do span V and the coordinates of f (x) = c2x2 + c1x + c0 are

(c2, c1 − 2c2t, c0 − c1t + c2t2).

Section 2.6: Computations Concerning SubspacesExercise 1: Let s < n and A an s × n matrix with entries in the field F. Use Theorem 4 (not its proof) to show that there is anon-zero X in Fn×1 such that AX = 0.

Solution: Let α1, α2, . . . , αn be the colunms of A. Then αi ∈ F s×1 ∀ i. Thus {α1, . . . , αn} are n vectors in F s×1. But F s×1

has dimension s < n thus by Theorem 4, page 44, α1, . . . , αn cannot be linearly independent. Thus ∃ x1, . . . , xn ∈ F such thatx1α1 + · · · + xnαn = 0. Thus if

X =

x1...

xn

then AX = x1α1 + · · · + xnαn = 0.

Exercise 2: Letα1 = (1, 1,−2, 1), α2 = (3, 0, 4,−1), α3 = (−1, 2, 5, 2).

Letα = (4,−5, 9,−7), β = (3, 1,−4, 4), γ = (−1, 1, 0, 1).

(a) Which of the vectors α, β, γ are in the subspace of R4 spanned by the αi?

(b) Which of the vectors α, β, γ are in the subspace of C4 spanned by the αi?

(c) Does this suggest a theorem?

Solution:

(a) We use the approach of row-reducing the matrix whose rows are given by the αi: 1 1 −2 13 0 4 −1−1 2 5 2

1 1 −2 10 −3 10 −40 3 3 3

1 1 −2 10 0 13 −10 1 1 1

1 0 −3 00 1 1 10 0 13 −1

Page 48: Linear Algebra - Greg Grantgreggrant.org/hoffman_and_kunze.pdf · Linear Algebra Ho man & Kunze 2nd edition ... Section 1.2: Systems of Linear Equations 3 Solution: We must check

46 Chapter 2: Vector Spaces

1 0 −3 00 1 1 10 0 1 −1/13

1 0 0 −3/130 1 0 14/130 0 1 −1/13

Let ρ1 = (1, 0, 0,−3/13), ρ2 = (0, 1, 0, 14/13) and ρ3 = (0, 0, 1,−1/13). Thus elements of the subspace spanned by the αi areof the form b1ρ1 + b2ρ2 + b3ρ3

=(b1, b2, b3,

113 (14b2 − 3b1 − b3)

).

• α = (4,−5, 9,−7). We have b1 = 4, b2 = −5 and b3 = 9. Thus if α is in the subspace it must be that

113

(14(−5) − 3(4) − 9) ?= b4

where b4 = −7. Indeed the left hand side does equal −7, so α is in the subspace.

• β = (3, 1,−4, 4). We have b1 = 3, b2 = 1, b3 = −4. Thus if β is in the subspace it must be that

113

(14 − 3(3) + 4) ?= b4

where b4 = 4. But the left hand side equals 9/13 , 4 so β is not in the subspace.

• γ = (−1, 1, 0, 1). We have b1 = −1, b2 = 1, b3 = 0. Thus if γ is in the subspace it must be that

113

(14 − 3(−1) − 0) ?= b4

where b4 = 1. But the left hand side equals 17/13 , 1 so γ is not in the subspace.

(b) Nowhere in the above did we use the fact that the field was R instead of C. The only equations we had to solve are linearequations with real coefficients, which have solutions in R if and only if they have solutions in C. Thus the same results hold:α is in the subspace while β and γ are not.

(c) This suggests the following theorem: Suppose F is a subfield of the field E and α1, . . . , αn are a basis for a subspace ofFn, and α ∈ Fn. Then α is in the subspace of Fn generated by α1, . . . , αn if and only if α is in the subspace of En generatedby α1, . . . , αn.

Exercise 3: Consider the vectors in R4 defined by

α1 = (−1, 0, 1, 2), α2 = (3, 4,−2, 5), α3 = (1, 4, 0, 9).

Find a system of homogeneous linear equations for which the space of solutions is exactly the subspace of R4 spanned by thethree given vectors.

Solution: We use the approach of row-reducing the matrix whose rows are given by the αi: −1 0 1 23 4 −2 51 4 0 9

→ 1 0 −1 −2

0 4 1 110 4 1 11

→ 1 0 −1 −2

0 1 1/4 11/40 0 0 0

.Let ρ1 = (1, 0,−1,−2) and ρ2 = (0, 1, 1/4, 11/4). Then the arbitrary element of the subspace spanned by α1 and α2 is of theform b1ρ1 + b2ρ2 for arbitrary b1, b2 ∈ R. Expanding we get

b1ρ1 + b2ρ2 = (b1, b2, −b1 +14

b2, −2b2 +114

b2).

Page 49: Linear Algebra - Greg Grantgreggrant.org/hoffman_and_kunze.pdf · Linear Algebra Ho man & Kunze 2nd edition ... Section 1.2: Systems of Linear Equations 3 Solution: We must check

Section 2.6: Computations Concerning Subspaces 47

Thus the equations that must be satisfied for (x, y, z,w) to be in the subspace are{z = −x + 1

4 yw = −2x + 11

4 y.

or equivalently {−x + 1

4 y − z = 0−2x + 11

4 y − w = 0.

Exercise 4: In C3 letα1 = (1, 0,−i), α2 = (1 + i, 1 − i, 1), α3 = (i, i, i).

Prove that these vectors form a basis for C3. What are the coordinates of the vector (a, b, c) in this basis?

Solution: We use the approach of row-reducing the augmented matrix: 1 0 −i 1 0 01 + i 1 − i 1 0 1 0

i i i 0 0 1

1 0 −i 1 0 00 1 − i i −1 − i 1 00 i i − 1 −i 0 1

1 0 −i 1 0 00 1 −1+i

2 −i 1+i2 0

0 i i − 1 −i 0 1

1 0 −i 1 0 00 1 −1+i

2 −i 1+i2 0

0 0 −1+3i2 −1 − i 1−i

2 1

1 0 −i 1 0 00 1 −1+i

2 −i 1+i2 0

0 0 1 −2+4i5

−2−i5

−1−3i5

1 0 0 1−2i

51−2i

53−i5

0 1 0 1−2i5

1+3i5

−2−i5

0 0 1 −2+4i5

−2−i5

−1−3i5

Since the left side transformed into the identity matrix we know that {α1, α2, α3} form a basis for C3. We used the vectors toform the rows of the augmented matrix not the columns, so the matrix on the right is (PT)−1 from (2-17). But (PT)−1 = (P−1)T,so the coordinate matrix of (a, b, c) with respect to the basis B = {α1, α2, α3} are given by

[(a, b, c)]B = (P−1)T

abc

=

1−2i

5 a + 1−2i5 b + −2+4i

5 c1−2i

5 a + 1+3i5 b + −2−i

5 c3−i5 a + −2−i

5 b + −1−3i5 c

.Exercise 5: Give an explicit description of the type (2-25) for the vectors

β = (b1, b2, b3, b4, b5)

Page 50: Linear Algebra - Greg Grantgreggrant.org/hoffman_and_kunze.pdf · Linear Algebra Ho man & Kunze 2nd edition ... Section 1.2: Systems of Linear Equations 3 Solution: We must check

48 Chapter 2: Vector Spaces

in R5 which are linear combinations of the vectors

α1 = (1, 0, 2, 1,−1), α2 = (−1, 2,−4, 2, 0)

α3 = (2,−1, 5, 2, 1), α4 = (2, 1, 3, 5, 2).

Solution: We row-reduce the matrix whose rows are given by the αi’s.1 0 2 1 −1−1 2 −4 2 02 −1 5 2 12 1 3 5 2

1 0 2 1 −10 2 −2 3 −10 −1 1 0 30 1 −1 3 4

1 0 2 1 −10 1 −1 3 40 0 0 −3 −90 0 0 3 7

1 0 2 1 −10 1 −1 3 40 0 0 1 30 0 0 3 7

1 0 2 1 −40 1 −1 0 −50 0 0 1 30 0 0 0 −2

1 0 2 0 −40 1 −1 0 −50 0 0 1 30 0 0 0 1

1 0 2 0 00 1 −1 0 00 0 0 1 00 0 0 0 1

Let ρ1 = (1, 0, 2, 0, 0), ρ2 = (0, 1,−1, 0, 0), ρ3 = (0, 0, 0, 1, 0) and ρ4 = (0, 0, 0, 0, 1). Then the general element that is a linearcombination of the αi’s is b1ρ1 + b2ρ2 + b3ρ3 + b4ρ4 = (b1, b2, 2b1 − b2, b3, b4).

Exercise 6: Let V be the real vector space spanned by the rows of the matrix

A =

3 21 0 9 01 7 −1 −2 −12 14 0 6 16 42 −1 13 0

.(a) Find a basis for V .

(b) Tell which vectors (x1, x2, x3, x4, x5) are elements of V .

Page 51: Linear Algebra - Greg Grantgreggrant.org/hoffman_and_kunze.pdf · Linear Algebra Ho man & Kunze 2nd edition ... Section 1.2: Systems of Linear Equations 3 Solution: We must check

Section 2.6: Computations Concerning Subspaces 49

(c) If (x1, x2, x3, x4, x5) is in V what are its coordinates in the basis chosen in part (a)?

Solution: We row-reduce the matrix 3 21 0 9 01 7 −1 −2 −12 14 0 6 16 42 −1 13 0

1 7 −1 −2 −10 0 3 15 30 0 2 10 30 0 5 25 6

1 7 −1 −2 −10 0 1 5 10 0 2 10 30 0 5 25 6

1 7 0 3 00 0 1 5 10 0 0 0 10 0 0 0 1

1 7 0 3 00 0 1 5 00 0 0 0 10 0 0 0 0

(a) A basis for V is given by the non-zero rows of the reduced matrix

ρ1 = (1, 7, 0, 3, 0), ρ2 = (0, 0, 1, 5, 0), ρ3 = (0, 0, 0, 0, 1).

(b) Vectors of V are any of the form b1ρ1 + b2ρ2 + b3ρ3

= (b1, 7b1, b2, 3b1 + 5b2, b3)

for arbitrary b1, b2, b3 ∈ R.

(c) By the above, the element (x1, x2, x3, x4, x5) in V must be of the form x1ρ1 + x3ρ2 + x5ρ3. In other words if B = {ρ1, ρ2, ρ3}

is the basis for V given in part (a), then the coordinate matrix of (x1, x2, x3, x4, x5) is

[(x1, x2, x3, x4, x5)]B =

x1x3x5

.Exercise 7: Let A be an m × n matrix over the field F, and consider the system of equations AX = Y . Prove that this systemof equations has a solution if and only if the row rank of A is equal to the row rank of the augmented matrix of the system.

Solution: To solve the system we row-reduce the augmented matrix [A |Y] resulting in an augmented matrix [R |Z] where Ris in reduced echelon form and Z is an m× 1 matrix. If the last k rows of R are zero rows then the system has a solution if andonly if the last k entries of Z are also zeros. Thus the only non-zero entries in Z are in the non-zero rows of R. These rowsare already linearly independent, and they clearly remain independent regardless of the augmented values. Thus if there aresolutions then the rank of the augmented matrix is the same as the rank of R. Conversely, if there are non-zero entries in Z inany of the last k rows then the system has no solutions. We want to show that those non-zero rows in the augmented matrix arelinearly independent from the non-zero rows of R, so we can conclude that the rank of R is less than the rank of [R |Z]. Let S

Page 52: Linear Algebra - Greg Grantgreggrant.org/hoffman_and_kunze.pdf · Linear Algebra Ho man & Kunze 2nd edition ... Section 1.2: Systems of Linear Equations 3 Solution: We must check

50 Chapter 2: Vector Spaces

be the set of rows of [R |Z] that contain all rows where R is non-zero, plus one additional row r where Z is non-zero. Supposea linear combination of the elements of S equals zero. Since c · r = 0⇔ r = 0, at least one of the elements of S different fromr must have a non-zero coefficient. Suppose row r′ ∈ S has non-zero coefficient c in the linear combination. Suppose theleading one in row r′ is in position i. Then the i-th coordinate of the linear combination is also c, because except for the onein the i-th position, all other entries in the i-th column of [R |Z] are zero. Thus there can be no non-zero coefficients. Thusthe set S is linearly independent and |S | = |R| + 1. Thus the system has a solution if and only if the rank of R is the same asthe rank of [R |Z]. Now A has the same rank as R and [R |Z] has the same rank as [A |Y] since they differ by elementary rowoperations. Thus the system has a solution if and only if the rank of A is the same as the rank of [A |Y].

Page 53: Linear Algebra - Greg Grantgreggrant.org/hoffman_and_kunze.pdf · Linear Algebra Ho man & Kunze 2nd edition ... Section 1.2: Systems of Linear Equations 3 Solution: We must check

Chapter 3: Linear Transformations

Section 3.1: Linear TransformationsExercise 1: Which of the following functions T from R2 into R2 are linear transformations?

(a) T (x1, x2) = (1 + x1, x2);

(b) T (x1, x2) = (x2, x1);

(c) T (x1, x2) = (x21, x2);

(d) T (x1, x2) = (sin x1, x2);

(e) T (x1, x2) = (x1 − x2, 0).

Solution:

(a) T is not a linear transformation because T (0, 0) = (1, 0) and according to the comments after Example 5 on page 68 weknow that it must always be that T (0, 0) = (0, 0).

(b) T is a linear transformation. Let α = (x1, x2) and β = (y1, y2). Then T (cα+β) = T ((cx1+y1, cx2+y2)) = (cx2+y2, cx1+y1) =

c(x2, x1) + (y2, y1) = cT (α) + T (β).

(c) T is not a linear transformation. If T were a linear transformation then we’d have (1, 0) = T ((−1, 0)) = T (−1 · (1, 0)) =

−1 · T (1, 0) = −1 · (1, 0) = (−1, 0) which is a contradiction, (1, 0) , (−1, 0).

(d) T is not a linear transformation. If T were a linear transformation then (0, 0) = T (π, 0) = T (2(π/2, 0)) = 2T ((π/2, 0)) =

2(sin(π/2), 0) = 2(1, 0) = (2, 0) which is a contradiction, (0, 0) , (2, 0).

(e) T is a linear transformation. Let Q =

[1 0−1 0

]. Then (identifying R2 with R1×2) T (x1, x2) = [x1 x2]Q so from Example

4, page 68, (with P being the identity matrix), it follows that T is a linear transformation.

Exercise 2: Find the range, rank, null space, and nullity for the zero transformation and the identity transformation on afinite-dimensional vector space V .

Solution: Suppose V has dimension n. The range of the zero transformation is the zero subspace {0}; the range of the identitytransformation is the whole space V . The rank of the zero transformation is the dimension of the range which is zero; the rankof the identity transformation is the rank of the whole space V which is n. The null space of the zero transformation is thewhole space V; the null space of the identity transformation is the zero subspace {0}. The nullity of the zero transformation isthe dimension of its null space, which is the whole space, so is n; the nullity of the identity transformation is the dimensionof its null space, which is the zero space, so is 0.

51

Page 54: Linear Algebra - Greg Grantgreggrant.org/hoffman_and_kunze.pdf · Linear Algebra Ho man & Kunze 2nd edition ... Section 1.2: Systems of Linear Equations 3 Solution: We must check

52 Chapter 3: Linear Transformations

Exercise 3: Describe the range and the null space for the differentiation transformation of Example 2. Do the same for theintegration transformation of Example 5.

Solution: V is the space of polynomals. The range of the differentiiation transformation is all of V since if f (x) =

c0 + c1x + · · · + cnxn then f (x) = (Dg)(x) where g(x) = c0x + c12 x2 + c2

3 x3 + · · · + cnn+1 xn+1. The null space of the differ-

entiation transformation is the set of constant polynomials since (Dc)(x) = 0 for constants c ∈ F.

The range of the integration transformation is all polynomials with constant term equal to zero. Let f (x) = c1x + c2x2 + · · · +

xncn. Then f (x) = (Tg)(x) where g(x) = c1 +2c2x+3c3x2 + · · ·+ncnxn−1. Clearly the integral transoformation of a polynomialhas constant term equal to zero, so this is the entire range of the integration transformation. The null space of the integrationtransformation is the zero space {0} since the (indefinite) integral of any other polynomial is non-zero.

Exercise 4: Is there a linear transformation T from R3 into R2 such that T (1,−1, 1) = (1, 0) and T (1, 1, 1) = (0, 1)?

Solution: Yes, there is such a linear transformation. Clearly α1 = (1,−1, 1) and α2 = (1, 1, 1) are linearly independent. ByCorollary 2, page 46, ∃ a third vector α3 such that {α1, α2, α3) is a basis for R3. By Theorem 1, page 69, there is a lineartransformation that takes α1, α2, α3 to any three vectors we want. Therefore we can find a linear transformation that takesα1 7→ (1, 0), α2 7→ (0, 1) and α3 7→ (0, 0). (We could have used any vector instead of (0, 0).)

Exercise 5: Ifα1 = (1,−1), β1 = (1, 0)

α2 = (2,−1), β2 = (0, 1)

α3 = (−3, 2), β3 = (1, 1)

is there a linear transformation T from R2 to R2 such that Tαi = βi for i = 1, 2 and 3?

Solution: No there is no such transformation. If there was then since {α1, α2} is a basis for R2 their images determine T com-pletely. Now α3 = −α1−α2, thus it must be that T (α3) = T (−α1−α2) = −T (α1)−T (α2) = −(1, 0)− (0, 1) = (−1,−1) , (1, 1).Thus no such T can exist.

Exercise 6: Describe explicitly (as in Exercises 1 and 2) the linear transformation T from F2 into F2 such that T ε1 = (a, b),T ε2 = (c, d).

Solution: I’m not 100% sure I understand what they want here. Let A be the matrix[

a bc d

]. Then the range of T is

the row-space of A which can have dimension 0, 1, or 2 depending on the row-rank. Explicitly it is all vectors of the formx(a, b) + y(c, d) = (ax + cy, bx + dy) where x, y are arbitrary elements of F. The rank is the dimension of this row-space, whichis 0 if a = b = c = d = 0 and if not all a, b, c, d are zero then by Exercise 1.6.8, page 27, the rank is 2 if ad − bc , 0 andequals 1 if ad − bc = 0.

Now let A be the matrix[

a cb d

]. Then the null space is the solution space of AX = 0. Thus the nullity is 2 if a = b = c =

d = 0, and if not all a, b, c, d are zero then by Exercise 1.6.8, page 27 and Theorem 13, page 23, is 0 if ad − bc , 0 and is 1 ifad − bc = 0.

Exercise 7: Let F be a subfield of the complex numbers and let T be the function from F3 into F3 defined by

T (x1, x2, x3) = (x1 − x2 + 2x3, 2x1 + x2,−x1 − 2x2 + 2x3).

(a) Verify that T is a linear transformation.

(b) If (a, b, c) is a vector in F3, what are the conditions on a, b, and c that the vector be in the range of T? What is the rankof T?

Page 55: Linear Algebra - Greg Grantgreggrant.org/hoffman_and_kunze.pdf · Linear Algebra Ho man & Kunze 2nd edition ... Section 1.2: Systems of Linear Equations 3 Solution: We must check

Section 3.1: Linear Transformations 53

(c) What are the conditions on a, b, and c that (a, b, c) be in the null space of T? What is the nullity of T?

Solution: (a) Let

P =

1 −1 22 1 0−1 −2 2

.Then T can be represented by x1

x2x3

7→ P

x1x2x3

.By Example 4, page 68, this is a linear transformation, where we’ve identified F3 with F3×1 and taken Q in Example 4 to bethe identity matrix.

(b) The range of T is the column space of P, or equivalently the row space of

PT =

1 2 −1−1 1 −22 0 2

.We row reduce the matrix as follows

1 2 −10 3 −30 −4 4

.→

1 2 −10 1 −10 0 0

.→

1 0 10 1 −10 0 0

.Let ρ1 = (1, 0, 1) and ρ2 = (0, 1,−1). Then elements of the row space are elements of the form b1ρ1 + b2ρ2 = (b1, b2, b1 − b2).Thus the rank of T is two and (a, b, c) is in the range of T as long as c = a − b.

Alternatively, we can row reduce the augmented matrix 1 −1 2 a2 1 0 b−1 −2 2 c

1 −1 2 a0 3 −4 b − 2a0 −3 4 a + c

1 −1 2 a0 3 −4 b − 2a0 0 0 −a + b + c

1 −1 2 a0 1 −4/3 (b − 2a)/40 0 0 −a + b + c

Page 56: Linear Algebra - Greg Grantgreggrant.org/hoffman_and_kunze.pdf · Linear Algebra Ho man & Kunze 2nd edition ... Section 1.2: Systems of Linear Equations 3 Solution: We must check

54 Chapter 3: Linear Transformations

1 0 2/3 (b + 2a)/40 1 −4/3 (b − 2a)/40 0 0 −a + b + c

from which we arrive at the condition −a + b + c = 0 or equivalently c = a − b.

(c) We must find all X =

abc

such that PX = 0 where P is the matrix from part (a). We row reduce the matrix

1 −1 22 1 0−1 −2 2

1 −1 20 3 −40 −3 4

1 −1 20 3 −40 0 0

1 −1 20 1 −4/30 0 0

1 0 2/30 1 −4/30 0 0

Therefore {

a + 23 c = 0

b − 43 c = 0

So elements of the null space of T are of the form (− 23 c, 4

3 c, c) for arbitrary c ∈ F and the dimension of the null space (thenullity) equals one.

Exercise 8: Describe explicitly a linear transformation from R3 to R3 which has as its range the subspace spanned by (1, 0,−1)and (1, 2, 2).

Solution: By Theorem 1, page 69, (and its proof) there is a linear transformation T from R3 to R3 such that T (1, 0, 0) =

(1, 0,−1), T (0, 1, 0) = (1, 0,−1) and T (0, 0, 1) = (1, 2, 2) and the range of T is exactly the subspace generated by

{T (1, 0, 0),T (0, 1, 0),T (0, 0, 1)} = {(1, 0,−1), (1, 2, 2)}.

Exercise 9: Let V be the vector space of all n × n matrices over the field F, and let B be a fixed n × n matrix. If

T (A) = AB − BA

verify that T is a linear transformation from V into V .

Solution: T (cA1 + A2) = (cA1 + A2)B − B(cA1 + A2) = cA1B + A2B − cBA1 − BA2

= c(A1B − BA1) + (A2B − BA2) = cT (A1) + T (A2).

Page 57: Linear Algebra - Greg Grantgreggrant.org/hoffman_and_kunze.pdf · Linear Algebra Ho man & Kunze 2nd edition ... Section 1.2: Systems of Linear Equations 3 Solution: We must check

Section 3.2: The Algebra of Linear Transformations 55

Exercise 10: Let V be the set of all complex numbers regarded as a vector space over the field of real numbers (usual oper-ations). Find a function from V into V which is a linear transformation on the above vector space, but which is not a lineartransformation on C1, i.e., which is not complex linear.

Solution: Let T : V → V be given by a + bi 7→ a. Let z = a + bi and w = a′ + b′i and c ∈ R. Then T (cz + w) =

T ((ca + a′) + (cb + b′)i) = ca + a′ = cT (a + bi) + T (a′ + b′i) = aT (z) + T (w). Thus T is real linear. However, if T werecomplex linear then we must have 0 = T (i) = T (i · 1) = i · T (1) = i · 1 = i. But 0 , i so this is a contradiction. Thus T is notcomplex linear.

Exercise 11: Let V be the space of n × 1 matrices over F and let W be the space of m × 1 matrices over F. Let A be a fixedm × n matrix over F and let T be the linear transformation from V into W defined by T (X) = AX. Prove that T is the zerotransformation if and only if A is the zero matrix.

Solution: If A is the zero matrix then clearly T is the zero transformation. Conversely, suppose A is not the zero matrix,suppose the k-th column Ak has a non-zero entry. Then T (εk) = Ak , 0.

Exercise 12: Let V be an n-dimensional vector space over the field F and let T be a linear transformation from V into V suchthat the range and null space of T are identical. Prove that n is even. (Can you give an example of such a linear transformationT?)

Solution: From Theorem 2, page 71, we know rank(T ) + nullity(T ) = dim V . In this case we are assuming both terms on theleft hand side are equal, say equal to m. Thus m + m = n or equivalently n = 2m which implies n is even.

The simplest example is V = {0} the zero space. Then trivially the range and null space are equal. To give a less trivialexample assume V = R2 and define T by T (1, 0) = (0, 0) and T (0, 1) = (1, 0). We can do this by Theorem 1, page 69 because{(1, 0), (0, 1)} is a basis for R2. Then clearly the range and null space are both equal to the subspace of R2 generated by (1, 0).

Exercise 13: Let V be a vector space and T a linear transformation from V into V . Prove that the following two statementsabout T are equivalent.

(a) The intersection of the range of T and the null space of T is the zero subspace of V .

(b) If T (Tα) = 0, then Tα = 0.

Solution: (a)⇒ (b): Statement (a) says that nothing in the range gets mapped to zero except for 0. In other words if x is inthe range of T then T x = 0⇒ x = 0. Now Tα is in the range of T , thus T (Tα) = 0⇒ Tα = 0.

(b)⇒ (a): Suppose x is in both the range and null space of T . Since x is in the range, x = Tα for some α. But then x in thenull space of T implies T (x) = 0 which implies T (Tα) = 0. Thus statement (b) implies Tα = 0 or equivalently x = 0. Thusthe only thing in both the range and null space of T is the zero vector 0.

Section 3.2: The Algebra of Linear TransformationsPage 76: Typo in line 1: It says Ai j, . . . , Am j, it should say A1 j, . . . , Am j.

Exercise 1: Let T and U be the linear operators on R2 defined by

T (x1, x2) = (x2, x1) and U(x1, x2) = (x1, 0).

(a) How would you describe T and U geometrically?

(b) Give rules like the ones defining T and U for each of the transformations (U + T ), UT , TU, T 2, U2.

Page 58: Linear Algebra - Greg Grantgreggrant.org/hoffman_and_kunze.pdf · Linear Algebra Ho man & Kunze 2nd edition ... Section 1.2: Systems of Linear Equations 3 Solution: We must check

56 Chapter 3: Linear Transformations

Solution: (a) Geometrically, in the x−y plane, T is the reflection about the diagonal x = y and U is a projection onto the x-axis.

(b)

• (U + T )(x1, x2) = (x2, x1) + (x1, 0) = (x1 + x2, x1).

• (UT )(x1, x2) = U(x2, x1) = (x2, 0).

• (TU)(x1, x2) = T (x1, 0) = (0, x1).

• T 2(x1, x2) = T (x2, x1) = (x1, x2), the identity function.

• U2(x1, x2) = U(x1, 0) = (x1, 0). So U2 = U.

Exercise 2: Let T be the (unique) linear operator on C3 for which

T ε1 = (1, 0, i), T ε2 = (0, 1, 1), T ε3 = (i, 1, 0).

Is T invertible?

Solution: By Theorem 9 part (v), top of page 82, T is invertible if {T ε1,T ε2,T ε3} is a basis of C3. Since C3 has dimensionthree, it suffices (by Corollary 1 page 46) to show T ε1,T ε2,T ε3 are linearly independent. To do this we row reduce the matrix 1 0 i

0 1 1i 1 0

to row-reduced echelon form. If it reduces to the identity then its rows are independent, otherwise they are dependent. Rowreduction follows: 1 0 i

0 1 1i 1 0

→ 1 0 i

0 1 10 1 1

→ 1 0 i

0 1 10 0 0

This is in row-reduced echelon form not equal to the identity. Thus T is not invertible.

Exercise 3: Let T be the linear operator on R3 defined by

T (x1, x2, x3) = (3x1, x1 − x2, 2x1 + x2 + x3).

Is T invertible? If so, find a rule for T−1 like the one which defines T .

Solution: The matrix representation of the transformation is x1x2x3

7→ 3 0 0

1 −1 02 1 1

· x1

x2x3

where we’ve identified R3 with R3×1. T is invertible if the matrix of the transformation is invertible. To determine this werow-reduce the matrix - we row-reduce the augmented matrix to determine the inverse for the second part of the Exercise. 3 0 0 1 0 0

1 −1 0 0 1 02 1 1 0 0 1

1 −1 0 0 1 03 0 0 1 0 02 1 1 0 0 1

Page 59: Linear Algebra - Greg Grantgreggrant.org/hoffman_and_kunze.pdf · Linear Algebra Ho man & Kunze 2nd edition ... Section 1.2: Systems of Linear Equations 3 Solution: We must check

Section 3.2: The Algebra of Linear Transformations 57

1 −1 0 0 1 00 3 0 1 −3 00 3 1 0 −2 1

1 −1 0 0 1 00 1 0 1/3 −1 00 3 1 0 −2 1

1 0 0 1/3 0 00 1 0 1/3 −1 00 0 1 −1 1 1

Since the left side transformed into the identity, T is invertible. The inverse transformation is given by x1

x2x3

7→ 1/3 0 0

1/3 −1 0−1 1 1

· x1

x2x3

So

T−1(x1, x2, x3) = (x1/3, x1/3 − x2, −x1 + x2 + x3).

Exercise 4: For the linear operator T of Exercise 3, prove that

(T 2 − I)(T − 3I) = 0.

Solution: Working with the matrix representation of T we must show

(A2 − I)(A − 3I) = 0

where

A =

3 0 01 −1 02 1 1

.Calculating:

A2 =

3 0 01 −1 02 1 1

3 0 0

1 −1 02 1 1

=

9 0 02 1 09 0 1

Thus

A2 − I =

8 0 02 0 09 0 0

.Also

A − 3I =

0 0 01 −4 02 1 −2

Thus

(A2 − I)(A − 3I) =

8 0 02 0 09 0 0

· 0 0 0

1 −4 02 1 −2

Page 60: Linear Algebra - Greg Grantgreggrant.org/hoffman_and_kunze.pdf · Linear Algebra Ho man & Kunze 2nd edition ... Section 1.2: Systems of Linear Equations 3 Solution: We must check

58 Chapter 3: Linear Transformations

=

0 0 00 0 00 0 0

.Exercise 5: Let C2×2 be the complex vector space of 2 × 2 matrices with complex entries. Let

B =

[1 −1−4 4

]and let T be the linear operator on C2×2 defined by T (A) = BA. What is the rank of T? Can you describe T 2?

Solution: An (ordered) basis for C2×2 is given by

A11 =

[1 00 0

], A21 =

[0 01 0

]

A12 =

[0 10 0

], A22 =

[0 00 1

].

If we identify C2×2 with C4 by [a bc d

]7→ (a, b, c, d)

then sinceA11 7→ A11 − 4A21

A21 7→ −A11 + 4A21

A12 7→ A12 − 4A22

A22 7→ −A12 + 4A22

the matrix of the transformation is given by 1 −4 0 0−1 4 0 00 0 1 −40 0 −1 4

.To find the rank of T we row-reduce this matrix:

1 −4 0 00 0 1 −40 0 0 00 0 0 0

.It has rank two so the rank, so the rank of T is 2.

T 2(A) = T (T (A)) = T (BA) = B(BA) = B2A. Thus T 2 is given by multiplication by a matrix just as T is, but multiplicationwith B2 instead of B. Explicitly

B2 =

[1 −1−4 4

] [1 −1−4 4

]=

[5 −5−20 20

].

Exercise 6: Let T be a linear transformation from R3 into R2, and let U be a linear transformation from R2 into R3. Provethat the transformation UT is not invertible. Generalize the theorem.

Page 61: Linear Algebra - Greg Grantgreggrant.org/hoffman_and_kunze.pdf · Linear Algebra Ho man & Kunze 2nd edition ... Section 1.2: Systems of Linear Equations 3 Solution: We must check

Section 3.2: The Algebra of Linear Transformations 59

Solution: Let {α1, α2, α3} be a basis for R3. Then T (α1),T (α2),T (α3) must be linearly dependent in R2, because R2 hasdimension 2. So suppose b1T (α1) + b2T (α2) + b3T (α3) = 0 and not all b1, b2, b3 are zero. Then b1α1 + b2α2 + b3α3 , 0 and

UT (b1α1 + b2α2 + b3α3)

= U(T (b1α1 + b2α2 + b3α3))

= U(b1T (α1) + b2T (α2) + b3T (α3)

= U(0) = 0.

Thus (by the definition at the bottom of page 79) UT is not non-singular and thus by Theorem 9, page 81, UT is not invertible.

The obvious generalization is that if n > m and T : Rn → Rm and U : Rm → Rn are linear transformations, then UT is notinvertible. The proof is an immediate generalization the proof of the special case above, just replace α3 with . . . , αn.

Exercise 7: Find two linear operators T and U on R2 such that TU = 0 but UT , 0.

Solution: Identify R2 with R2×1 and let T and U be given by the matrices

A =

[1 00 0

], B =

[0 10 0

].

More precisely, for

X =

[xy

].

let T be given by X 7→ AX and let U be given by X 7→ BX. Thus TU is given by X 7→ ABX and UT is given by X 7→ BAX.But BA = 0 and AB , 0 so we have the desired example.

Exercise 8: Let V be a vector space over the field F and T a linear operator on V . If T 2 = 0, what can you say about therelation of the range of T to the null space of T? Give an example of a linear operator T on R2 such that T 2 = 0 but T , 0.

Solution: If T 2 = 0 then the range of T must be contained in the null space of T since if y is in the range of T then y = T xfor some x so Ty = T (T x) = T 2x = 0. Thus y is in the null space of T .

To give an example of an operator where T 2 = 0 but T , 0, let V = R2×1 and let T be given by the matrix

A =

[0 10 0

].

Specifically, for

X =

[xy

].

let T be given by X 7→ AX. Since A , 0, T , 0. Now T 2 is given by X 7→ A2X, but A2 = 0. Thus T 2 = 0.

Exercise 9: Let T be a linear operator on the finite-dimensional space V . Suppose there is a linear operator U on V suchthat TU = I. Prove that T is invertible and U = T−1. Give an example which shows that this is false when V is not finite-dimensional. (Hint: Let T = D, be the differentiation operator on the space of polynomial functions.)

Solution: By the comments in the Appendix on functions, at the bottom of page 389, we see that simply because TU = I asfunctions, then necessarily T is onto and U is one-to-one. It then follows immediately from Theorem 9, page 81, that T isinvertible. Now TT−1 = I = TU and multiplying on the left by T−1 we get T−1TT−1 = T−1TU which implies (I)T−1 = (I)Uand thus U = T−1.

Page 62: Linear Algebra - Greg Grantgreggrant.org/hoffman_and_kunze.pdf · Linear Algebra Ho man & Kunze 2nd edition ... Section 1.2: Systems of Linear Equations 3 Solution: We must check

60 Chapter 3: Linear Transformations

Let V be the space of polynomial functions in one variable over R. Let D be the differentiation operator and let T be theoperator “multiplication by x” (exactly as in Example 11, page 80). As shown in Example 11, UT = I while TU , I. Thusthis example fulfills the requirement.

Exercise 10: Let A be an m × n matrix with entries in F and let T be the linear transformation from Fn×1 into Fm×1 definedby T X = AX. Show that if m < n it may happen that T is onto without being non-singular. Similarly, show that if m > n wemay have T non-singular but not onto.

Solution: Let B = {α1, . . . , αn} be a basis for Fn×1 and let B′ = {β1, . . . , βm} be a basis for Fm×1. We can define a lineartransformation from Fn×1 to Fm×1 uniquely by specifying where each member of B goes in Fm×1. If m < n then we can definea linear transformation that maps at least one member of B to each member of B′ and maps at least two members of B to thesame member of B′. Any linear transformation so defined must necessarily be onto without being one-to-one. Similarly, ifm > n then we can map each member of B to a unique member of B′ with at least one member of B′ not mapped to by anymember of B. Any such transformation so defined will necessarily be one-to-one but not onto.

Exercise 11: Let V be a finite-dimensional vector space and let T be a linear operator on V . Suppose that rank(T 2) = rank(T ).Prove that the range and null space of T are disjoint, i.e., have only the zero vector in common.

Solution: Let {α1, . . . , αn} be a basis for V . Then the rank of T is the number of linearly independent vectors in the set{Tα1, . . . ,Tαn}. Suppose the rank of T equals k and suppose WLOG that {Tα1, . . . ,Tαk} is a linearly independent set (it mightbe that k = 1, pardon the notation). Then {Tα1, . . . ,Tαk} give a basis for the range of T . It follows that {T 2α1, . . . ,T 2αk} spanthe range of T 2 and since the dimension of the range of T 2 is also equal to k, {T 2α1, . . . ,T 2αk} must be a basis for the rangeof T 2. Now suppose v is in the range of T . Then v = c1Tα1 + · · · + ckTαk. Suppose v is also in the null space of T . Then0 = T (v) = T (c1Tα1 + · · · + ckTαk) = c1T 2α1 + · · · + ckT 2αk. But {T 2α1, . . . ,T 2αk} is a basis, so T 2α1, . . . ,T 2αk are linearlyindependent, thus it must be that c1 = · · · = ck = 0, which implies v = 0. Thus we have shown that if v is in both the range ofT and the null space of T then v = 0, as required.

Exercise 12: Let p,m, and n be positive integers and F a field. Let V be the space of m × n matrices over F and W the spaceof p × n matrices over F. Let B be a fixed p × m matrix and let T be the linear transformation from V into W defined byT (A) = BA. Prove that T is invertible if and only if p = m and B is an invertible m × m matrix.

Solution: We showed in Exercise 2.3.12, page 49, that the dimension of V is mn and the dimension of W is pn. By Theorem9 page (iv) we know that an invertible linear transformation must take a basis to a basis. Thus if there’s an invertible lineartransformation between V and W it must be that both spaces have the same dimension. Thus if T is inverible then pn = mnwhich implies p = m. The matrix B is then invertible because the assignment B 7→ BX is one-to-one (Theorem 9 (ii), page81) and non-invertible matrices have non-trivial solutions to BX = 0 (Theorem 13, page 23). Conversely, if p = n and B isinvertible, then we can define the inverse transformation T−1 by T−1(A) = B−1A and it follows that T is invertible.

Section 3.3: IsomorphismExercise 1: Let V be the set of complex numbers and let F be the field of real numbers. With the usual operations, V is avector space over F. Describe explicitly an isomorphism of this space onto R2.

Solution: The natural isomorphism from V to R2 is given by a + bi 7→ (a, b). Since i acts like a placeholder for addition in C,(a + bi) + (c + di) = (a + c) + (b + d)i 7→ (a + c, b + d) = (a, b) + (c, d). And c(a + bi) = ca + cbi 7→ (ca, cb) = c(a, b). Thus thisis a linear transformation. The inverse is clearly (a, b) 7→ a + bi. Thus the two spaces are isomorphic as vector spaces over R.

Exercise 2: Let V be a vector space over the field of complex numbers, and suppose there is an isomorphism T of V into C3.Let α1, α2, α3, α4 be vectors in V such that

Tα1 = (1, 0, i), Tα2 = (−2, 1 + i, 0),

Page 63: Linear Algebra - Greg Grantgreggrant.org/hoffman_and_kunze.pdf · Linear Algebra Ho man & Kunze 2nd edition ... Section 1.2: Systems of Linear Equations 3 Solution: We must check

Section 3.3: Isomorphism 61

Tα3 = (−1, 1, 1), Tα4 = (√

2, i, 3).

(a) Is α1 in the subspace spanned by α2 and α3?

(b) Let W1 be the subspace spanned by α1 and α2, and let W2 be the subspace spanned by α3 and α4. What is the intersectionof W1 and W2?

(c) Find a basis for the subspace of V spanned by the four vectors α j.

Solution: (a) Since T is an isomorphism, it suffices to determine whether Tα1 is contained in the subspace spanned by Tα2and Tα3. In other words we need to determine if there is a solution to −2 −1

1 + i 10 1

[

xy

]=

10i

.To do this we row-reduce the augmented matrix −2 −1 1

1 + i 1 00 1 i

→ 1 1/2 −1/2

1 + i 1 00 1 i

→ 1 1/2 −1/2

0 1 i1 + i 1 0

1 1/2 −1/20 1 i0 1−i

21+i2

→ 1 0 −1−i

20 1 i0 0 0

The zero row on the left of the dividing line has zero also on the right. This means the system has a solution. Therefore wecan conclude that α1 is in the subspace generated by α2 and α3.

(b) Since Tα1 and Tα2 are linearly independent, and Tα3 and Tα4 are linearly independent, dim(W1) = dim(W2) = 2. Werow-reduce the matrix whose columns are the Tαi: 1 −2 −1

√2

0 1 + i 1 ii 0 1 3

which yields 1 0 −i 0

0 1 1−i2 0

0 0 0 1

,from which we deduce that Tα1,Tα2,Tα3,Tα4 generate a space of dimension three, thus dim(W1+W2) = 3. Since dim(W1) =

dim(W2) = 2 it follows from Theorem 6, page 46 that dim(W1∩W2) = 1. Now AX = 0⇔ RX = 0 where R is the row reducedechelon form of A. This follows from the fact that R = PA; multiply both sides of AX = 0 on the left by P. Solving for X inRX = 0 gives the general solution is of the form (ic, i−1

2 c, c, 0). Letting c = 2 gives

2iTα1 + (i − 1)Tα2 + 2Tα3 = 0

which implies Tα3 = −iTα1+ 1−i2 Tα2 which implies Tα3 ∈ TW1. Thus α3 ∈ W1. Thus α3 ∈ W1∩W2. Since dim(W1∩W2) = 1

it follows that W1 ∩W2 = Cα3.

(c) We have determined in part (b) that the {α1, α2, α3, α4} span a space of dimension three, and that α3 is in the space gener-ated by α1 and α2. Thus {α1, α2, α4} give a basis for the subspace spanned by {α1, α2, α3, α4}, which in fact is all of C3.

Page 64: Linear Algebra - Greg Grantgreggrant.org/hoffman_and_kunze.pdf · Linear Algebra Ho man & Kunze 2nd edition ... Section 1.2: Systems of Linear Equations 3 Solution: We must check

62 Chapter 3: Linear Transformations

Exercise 3: Let W be the set of all 2 × 2 complex Hermitian matrices, that is, the set of 2 × 2 complex matrices A such thatAi j = A ji (the bar denoting complex conjugation). As we pointed out in Example 6 of Chapter 2, W is a vector space over thefield of real numbers, under the usual operations. Verify that

(x, y, z, t)→[

t + x y + izy − iz t − x

]is an isomorphism of R4 onto W.

Solution: The function is linear since the four components are all linear combinations of the components of the domain(x, y, z, t). Identify C2×2 with C4 by A 7→ (A11, A12, A21, A22). Then the matrix of the transformation is given by

1 0 0 10 1 i 00 1 −i 0−1 0 0 1

.As usual, the transformation is an isomorphism if the matrix is invertible. We row-reduce to veryify the matrix is invertible.We will row-reduce the augmented matrix in order to find the inverse explicitly:

1 0 0 1 1 0 0 00 1 i 0 0 1 0 00 1 −i 0 0 0 1 0−1 0 0 1 0 0 0 1

.This reduces to

1 0 0 0 1/2 0 0 −1/20 1 0 0 0 1/2 1/2 00 0 1 0 0 −i/2 i/2 00 0 0 1 1/2 0 0 1/2

.Thus the inverse transformation is [

x yz w

]7→

(x − w

2,

y + z2

,i(z − y)

2,

x + w2

).

Exercise 4: Show that Fm×n is isomorphic to Fmn.

Solution: Define the bijection σ from {(a, b) | a, b ∈ N, 1 ≤ a ≤ m, 1 ≤ b ≤ n} to {1, 2, . . . ,mn} by (a, b) 7→ (a−1)n+b. Definethe function G from Fm×n to Fmn as follows. Let A ∈ Fm×n. Then map A to the mn-tuple that has Ai j in the σ(i, j) position. Inother words A 7→ (A11, A12, A13, . . . , A1n, A21, A22, A23, . . . , A2n, . . . . . . , Ann). Since addition in Fm×n and in Fmn is performedcompenent-wise, G(A + B) = G(A) + G(B). Similarly since scalar multiplication factors out of vectors component-wise in thesame way in Fm×n as in Fmn, we also have G(cA) = cG(A). Thus G is a linear function. G is clearly one-to-one (as well asclearly onto), and both Fm×n and Fmn have dimension mn (by Example 17, page 45 and Exercise 2.3.12, page 49), thus (byTheorem 9, page 81) it follows that G has an inverse and therefore is an isomorphism.

Exercise 5: Let V be the set of complex numbers regarded as a vector space over the field of real numbers (Exercise 1). Wedefine a function T from V into the space of 2 × 2 real matrices, as follows. If z = x + iy with x and y real numbers, then

T (z) =

[x + 7y 5y−10y x − 7y

].

(a) Verify that T is a one-one (real) linear transformation of V into the space of 2 × 2 matrices.

(b) Verify that T (z1z2) = T (z1)T (z2).

Page 65: Linear Algebra - Greg Grantgreggrant.org/hoffman_and_kunze.pdf · Linear Algebra Ho man & Kunze 2nd edition ... Section 1.2: Systems of Linear Equations 3 Solution: We must check

Section 3.4: Representation of Transformations by Matrices 63

(c) How would you describe the range of T?

Solution:

(a) The four coordinates of T (z) are written as linear combinations of the coordinates of z (as a vector over R). Thus T isclearly a linear transformation. To see that T is one-to-one, let z = x + yi and w = a + bi and suppose T (z) = T (w). Thenconsidering the top right entry of the matrix we see that 5y = 5b which implies b = y. It now follows from the top left entryof the matrix that x = a. Thus T (z) = T (w)⇒ z = w, thus T is one-to-one.

(b) Let z1 = x + yi and z2 = a + bi. Then

T (z1z2) = T ((ax − by) + (ay + bx)i) =

[(ax − by) + 7(ay + bx) 5(ay + bx)−10(ay + bx) (ax − by) − 7(ay + bx)

].

On the other hand,

T (z1)T (z2) =

[x + 7y 5y−10y x − 7y

] [a + 7b 5b−10b a − 7b

]=

[(ax − by) + 7(ay + bx) 5(ay + bx)−10(ay + bx) (ax − by) − 7(ay + bx)

].

Thus T (z1z2) = T (z1)T (z2).

(c) The range of T has (real) dimension equal to two by part (a), and so the range of T is isomorphic to C as real vectorspaces. But both spaces also have a natural multiplication and in part (b) we showed that T respects the multiplication. Thusthe range of T is isomorphic to C as fields and we have essentially found an isomorphic copy of the field C in the algebra of2 × 2 real matrices.

Exercise 6: Let V and W be finite-dimensional vector spaces over the field F. Prove that V and W are isomorphic if and onlyif dim(V) = dim(W).

Solution: Suppose dim(V) = dim(W) = n. By Theorem 10, page 84, both V and W are isomorphic to Fn, and consequently,since isomorphism is an equivalence relation, V and W are isomorphic to each other. Conversely, suppose T is an isomor-phism from V to W. Suppose dim(W) = n. Then by Theorem 10 again, there is an isomorphism S : W → Fn. Thus S T is anisomorphism from V to Fn implying also dim(V) = n.

Exercise 7: Let V and W be vector spaces over the field F and let U be an isomorphism of V onto W. Prove that T → UTU−1

is an isomorphism of L(V,V) onto L(W,W).

Solution: L(V,V) is defined on page 75 as the vector space of linear transformations from V to V , and likewise L(W,W) is thevector space of linear transformations from W to W.

Call the function f . We know f (T ) is linear since it is a composition of three linear tranformations UTU−1. Thus indeed fis a function from L(V,V) to L(W,W). Now f (aT + T ′) = U(aT + T ′)U−1 = (aUT + UT ′)U−1 = aUTU−1 + UT ′U−1 =

a f (T ) + f (T ′). Thus f is linear. We just must show f has an inverse. Let g be the function from L(W,W) to L(V,V) given byg(T ) = U−1TU. Then g f (T ) = U−1(UTU−1)U = T . Similarly f g = I. Thus f and g are inverses. Thus f is an isomorphism.

Section 3.4: Representation of Transformations by MatricesPage 90: Typo. Four lines from the bottom it says “Example 12” where they probably meant Example 10 (page 78).

Page 91: Just before (3-8) it says ”By definition”. I think it’s more than just by definition, see bottom of page 88.

Page 66: Linear Algebra - Greg Grantgreggrant.org/hoffman_and_kunze.pdf · Linear Algebra Ho man & Kunze 2nd edition ... Section 1.2: Systems of Linear Equations 3 Solution: We must check

64 Chapter 3: Linear Transformations

Exercise 1: Let T be the linear operator on C2 defined by T (x1, x2) = (x1, 0). Let B be the standard ordered basis for C2 andlet B′ = {α1, α2} be the ordered basis defined by α1 = (1, i), α2 = (−i, 2).

(a) What is the matrix of T relative to the pair B, B′?

(b) What is the matrix of T relative to the pair B′, B?

(c) What is the matrix of T in the ordered basis B′?

(d) What is the matrix of T in the ordered basis {α2, α1}?

Solution: (a) According to the comments at the bottom of page 87, the i-th column of the matrix is given by [T εi]B′ , whereε1 = (1, 0) and ε2 = (0, 1), the standard basis vectors of C2. Now T ε1 = (1, 0) and T ε2 = (0, 0). To write these in terms of α1and α2 we use the approach of row-reducing the augmented matrix[

1 −i 1 0i 2 0 0

]→

[1 −i 1 00 1 −i 0

]→

[1 0 2 00 1 −i 0

].

Thus T ε1 = 2α1 − iα2 and T ε2 = 0 · α1 + 0 · α2 and the matrix of T relative to B, B′ is[2 0−i 0

].

(b) In this case we have to write Tα1 and Tα2 as linear combinations of ε1, ε2.

Tα1 = (1, 0) = 1 · ε1 + 0 · ε2

Tα2 = (−i, 0) = −i · ε1 + 0 · ε2.

Thus the matrix of T relative to B′, B is [1 −i0 0

].

(c) In this case we need to write Tα1 and Tα2 as linear combinations of α1 and α2. Tα1 = (1, 0), Tα2 = (−i, 0). We row-reducethe augmented matrix: [

1 −i 1 −ii 2 0 0

]→

[1 −i 1 −i0 1 −i −1

]→

[1 0 2 −2i0 1 −i −1

].

Thus the matrix of T in the ordered basis B′ is [2 −2i−i −1

].

(d) In this case we need to write Tα2 and Tα1 as linear combinations of α2 and α1. In this case the matrix we need torow-reduce is just the same as in (c) but with columns switched:[

−i 1 −i 12 i 0 0

]→

[1 i 1 i2 i 0 0

]→

[1 i 1 i0 −i −2 −2i

]

[1 i 1 i0 1 −2i 2

]→

[1 0 −1 −i0 1 −2i 2

]Thus the matrix of T in the ordered basis {α2, α1} is [

−1 −i−2i 2

].

Exercise 2: Let T be the linear transformation from R3 to R2 defined by

T (x1, x2, x3) = (x1 + x2, 2x3 − x1).

Page 67: Linear Algebra - Greg Grantgreggrant.org/hoffman_and_kunze.pdf · Linear Algebra Ho man & Kunze 2nd edition ... Section 1.2: Systems of Linear Equations 3 Solution: We must check

Section 3.4: Representation of Transformations by Matrices 65

(a) If B is the standard ordered basisfor R3 and B′ is the standard ordered basis for R2, what is the matrix of T relative tothe pair B, B′?

(b) If B = {α1, α2, α3} and B′ = {β1, β2}, where

α1 = (1, 0,−1), α2 = (1, 1, 1), α3 = (1, 0, 0), β1 = (0, 1), β2 = (1, 0)

what is the matrix of T relative to the pair B, B′?

Solution: With respect to the standard bases, the matrix is simply[1 1 0−1 0 2

].

(b) We must write Tα1,Tα2,Tα3 in terms of β1, β2.

Tα1 = (1,−3)

Tα2 = (2, 1)

Tα3 = (1, 0).

We row-reduce the augmented matrix[0 1 1 2 11 0 −3 1 0

]→

[1 0 −3 1 00 1 1 2 1

].

Thus the matrix of T with respect to B, B′ is [−3 1 01 2 1

].

Exercise 3: Let T be a linear operator on Fn, let A be the matrix of T in the standard ordered basis for Fn, and let W be thesubspace of Fn spanned by the column vectors of A. What does W have to do with T?

Solution: Since {α1, . . . , αn} is a basis of Fn, we know {T ε1, . . . ,T εn} generate the range of T . But T εi equals the i-th columnvector of A. Thus the column vectors of A generate the range of T (where we identify Fn with Fn×1). We can also concludethat a subset of the columns of A give a basis for the range of T .

Exercise 4: Let V be a two-dimensional vector space over the field F, and let B be an ordered basis for V . If T is a linearoperator on V and

[T ]B =

[a bc d

]prove that T 2 − (a + d)T + (ad − bc)I = 0.

Solution: The coordinate matrix of T 2 − (a + d)T + (ad − bc)I with respect to B is

[T 2 − (a + d)T + (ad − bc)I]B =

[a bc d

]2

− (a + d)[

a bc d

]+ (ad − bc)

[1 00 1

]Expanding gives

=

[a2 + bc ab + bdac + cd bc + d2

]−

[a2 + ad ab + bdac + cd ad + d2

]+

[ad − bc 0

0 ad − bc

]=

[0 00 0

].

Page 68: Linear Algebra - Greg Grantgreggrant.org/hoffman_and_kunze.pdf · Linear Algebra Ho man & Kunze 2nd edition ... Section 1.2: Systems of Linear Equations 3 Solution: We must check

66 Chapter 3: Linear Transformations

Thus T 2 − (a + d)T + (ad − bc)I is represented by the zero matrix with respect to B. Thus T 2 − (a + d)T + (ad − bc)I = 0.

Exercise 5: Let T be the linear operator on R3, the matrix of which in the standard ordered basis is 1 2 10 1 1−1 3 4

.Find a basis for the range of T and a basis for the null space of T .

Solution: The range is the column-space, which is the row-space of the following matrix (the transpose): 1 0 −12 1 31 1 4

which we can easily determine a basis of by putting it in row-reduced echelon form. 1 0 −1

2 1 31 1 4

→ 1 0 −1

0 1 50 1 5

→ 1 0 −1

0 1 50 0 0

.So a basis of the range is {(1, 0,−1), (0, 1, 5)}.

The null space can be found by row-reducing the matrix 1 2 10 1 1−1 3 4

→ 1 2 1

0 1 10 5 5

→ 1 0 −1

0 1 10 0 0

So {

x − z = 0y + z = 0

which implies {x = z

y = −z

The solutions are parameterized by the one variable z, thus the null space has dimension equal to one. A basis is obtained bysetting z = 1. Thus {(1,−1, 1)} is a basis for the null space.

Exercise 6: Let T be the linear operator on R2 defined by

T (x1, x2) = (−x2, x1).

(a) What is the matrix of T in the standard ordered basis for R2?

(b) What is the matrix of T in the ordered basis B = {α1, α2}, where α1 = (1, 2) and α2 = (1,−1)?

(c) Prove that for every real number c the operator (T − cI) is invertible.

(d) Prove that if B is any ordered basis for R2 and [T ]B = A, then A12A21 , 0.

Solution: (a) We must write T ε1 = (0, 1) and T ε2 = (−1, 0) in terms of ε1 and ε2. Clearly T ε1 = ε2 and T ε2 = −ε1. Thus thematrix is [

0 −11 0

].

Page 69: Linear Algebra - Greg Grantgreggrant.org/hoffman_and_kunze.pdf · Linear Algebra Ho man & Kunze 2nd edition ... Section 1.2: Systems of Linear Equations 3 Solution: We must check

Section 3.4: Representation of Transformations by Matrices 67

(b) We must write Tα1 = (−2, 1) and Tα2 = (1, 1) in terms of α1, α2. We can do this by row-reducing the augmented matrix[1 1 −2 12 −1 1 1

]

[1 1 −2 10 −3 5 −1

]→

[1 1 −2 10 1 −5/3 1/3

]→

[1 0 −1/3 2/30 1 −5/3 1/3

]Thus the matrix of T in the ordered basis B is

[T ]B =

[−1/3 2/3−5/3 1/3

].

(c) The matrix of T − cI with respect to the standard basis is[0 −11 0

]− c

[1 00 1

][

0 −11 0

]−

[c 00 c

][−c −11 −c

].

Row-reducing the matrix [−c −11 −c

]→

[1 −c−c −1

]→

[1 −c0 −1 − c2

].

Now −1 − c2 , 0 (since c2 ≥ 0). Thus we can continue row-reducing by dividing the second row by −1 − c2 to get

[1 −c0 1

]→

[1 00 1

].

Thus the matrix has rank two, thus T is invertible.

(d) Let {α1, α2} be any basis. Write α1 = (a, b), α2 = (c, d). Then Tα1 = (−b, a), Tα2 = (−d, c). We need to write Tα1 andTα2 in terms of α1 and α2. We can do this by row reducing the augmented matrix[

a c −b −db d a c

].

Since {α1, α2} is a basis, the matrix[

a bc d

]is invertible. Thus (recalling Exercise 1.6.8, page 27), ad − bc , 0. Thus the

matrix row-reduces to 1 0 ac+bdad−bc

c2+d2

ad−bc

0 1 a2+b2

ad−bcac+bdad−bc

.Assuming a , 0 this can be shown as follows:

[1 c/a −b/a −d/ab d a c

].

Page 70: Linear Algebra - Greg Grantgreggrant.org/hoffman_and_kunze.pdf · Linear Algebra Ho man & Kunze 2nd edition ... Section 1.2: Systems of Linear Equations 3 Solution: We must check

68 Chapter 3: Linear Transformations

[1 c/a −b/a −d/a0 ad−bc

aa2+b2

aac+bd

a

].

[1 c/a −b/a −d/a0 1 a2+b2

ad−bcac+bdad−bc

].

1 0 ac+bdad−bc

c2+d2

ad−bc

0 1 a2+b2

ad−bcac+bdad−bc

.If b , 0 then a similar computation results in the same thing. Thus

[T ]B =

ac+bdad−bc

c2+d2

ad−bca2+b2

ad−bcac+bdad−bc

.Now ad− bc , 0 implies that at least one of a or b is non-zero and at least one of c or d is non-zero, it follows that a2 + b2 > 0and c2 + d2 > 0. Thus (a2 + b2)(c2 + d2) , 0. Thus

a2 + b2

ad − bc·

c2 + d2

ad − bc, 0

Exercise 7: Let T be the linear operator on R3 defined by

T (x1, x2, x3) = (3x1 + x3, −2x1 + x2, −x1 + 2x2 + 4x3).

(a) What is the matrix of T in the standard ordered basis for R3.

(b) What is the matrix of T in the ordered basis(α1, α2, α3)

where α1 = (1, 0, 1), α2 = (−1, 2, 1), and α3 = (2, 1, 1)?

(c) Prove that T is invertible and give a rule for T−1 like the one which defines T .

Solution: (a) As usual we can read the matrix in the standard basis right off the definition of T :

[T ]{ε1,ε2,ε3} =

3 0 1−2 1 0−1 2 4

.(b) Tα1 = (4,−2, 3), Tα2 = (−2, 4, 9) and Tα3 = (7,−3, 4). We must write these in terms of α1, α2, α3. We do this byrow-reducing the augmented matrix 1 −1 2 4 −2 7

0 2 1 −2 4 −31 1 1 3 9 4

1 −1 2 4 −2 70 2 1 −2 4 −30 2 −1 −1 11 −3

1 −1 2 4 −2 70 2 1 −2 4 −30 0 −2 1 7 0

1 −1 2 4 −2 70 1 1/2 −1 2 −3/20 0 1 −1/2 −7/2 0

Page 71: Linear Algebra - Greg Grantgreggrant.org/hoffman_and_kunze.pdf · Linear Algebra Ho man & Kunze 2nd edition ... Section 1.2: Systems of Linear Equations 3 Solution: We must check

Section 3.4: Representation of Transformations by Matrices 69

1 0 5/2 3 0 11/20 1 1/2 −1 2 −3/20 0 1 −1/2 −7/2 0

1 0 0 17/4 35/4 11/20 1 0 −3/4 15/4 −3/20 0 1 −1/2 −7/2 0

Thus the matrix of T in the basis {α1, α2, α3} is

[T ]{α1,α2,α3} =

17/4 35/4 11/2−3/4 15/4 −3/2−1/2 −7/2 0

.(c) We row reduce the augmented matrix (of T in the standard basis). If we achieve the identity matrix on the left of thedividing line then T is invertible and the matrix on the right will represent T−1 in the standard basis, from which we will beable read the rule for T−1 by inspection. 3 0 1 1 0 0

−2 1 0 0 1 0−1 2 4 0 0 1

−1 2 4 0 0 13 0 1 1 0 0−2 1 0 0 1 0

1 −2 −4 0 0 −13 0 1 1 0 0−2 1 0 0 1 0

1 −2 −4 0 0 −10 6 13 1 0 30 −3 −8 0 1 −2

1 −2 −4 0 0 −10 0 −3 1 2 −10 −3 −8 0 1 −2

1 −2 −4 0 0 −10 −3 −8 0 1 −20 0 −3 1 2 −1

1 −2 −4 0 0 −10 1 8/3 0 −1/3 2/30 0 −3 1 2 −1

1 0 4/3 0 −2/3 1/30 1 8/3 0 −1/3 2/30 0 −3 1 2 −1

1 0 4/3 0 −2/3 1/30 1 8/3 0 −1/3 2/30 0 1 −1/3 −2/3 1/3

1 0 0 4/9 2/9 −1/90 1 0 8/9 13/9 −2/90 0 1 −1/3 −2/3 1/3

Page 72: Linear Algebra - Greg Grantgreggrant.org/hoffman_and_kunze.pdf · Linear Algebra Ho man & Kunze 2nd edition ... Section 1.2: Systems of Linear Equations 3 Solution: We must check

70 Chapter 3: Linear Transformations

Thus T is invertible and the matrix for T−1 in the standard basis is 4/9 2/9 −1/98/9 13/9 −2/9−1/3 −2/3 1/3

.Thus T−1(x1, x2, x3) =

(49 x1 + 2

9 x2 −19 x3,

89 x1 + 13

9 x2 −29 x3, −

13 x1 −

23 x2 + 1

3 x3

).

Exercise 8: Let θ be a real number. Prove that the following two matrices are similar over the field of complex numbers:[cos θ − sin θsin θ cos θ

],

[eiθ 00 e−iθ

](Hint: Let T be the linear operator on C2 which is represented by the first matrix in the standard ordered basis. Then findvectors α1 and α2 such that Tα1 = eiθα1, Tα2 = e−iθα2, and {α1, α2} is a basis.)

Solution: Let B be the standard basis. Following the hint, let T be the linear operator on C2 which is represented by the firstmatrix in the standard ordered basis B. Thus [T ]B is the first matrix above. Let α1 = (i, 1), α2 = (i,−1). Then α1, α2 areclealry linearly independent so B′ = {α1, α2} is a basis for C2 (as a vector space over C). Since eiθ = cos θ + i sin θ, it followsthat Tα1 = (i cos θ− sin θ, i sin θ+cos θ) = (cos θ+ i sin θ)(i, 1) = eiθα1 and similarly since and e−iθ = cos θ− i sin θ, it followsthat Tα2 = e−iθα2. Thus the matrix of T with respect to B′ is

[T ]B′ =

[eiθ 00 e−iθ

].

By Theorem 14, page 92, [T ]B and [T ]B′ are similar.

Exercise 9: Let V be a finite-dimensional vector space over the field F and let S and T be linear operators on V . We ask:When do there exist ordered bases B and B′ for V such that [S ]B = [T ]B′? Prove that such bases exist if and only if there isan invertible linear operator U on V such that T = US U−1. (Outline of proof: If [S ]B = [T ]B′ , let U be the operator whichcarries B onto B′ and show that S = UTU−1. Conversely, if T = US U−1 for some invertible U, let B be any ordered basisfor V and let B′ be its image under U. Then show that [S ]B = [T ]B′ .)

Solution: We follow the hint. Suppose there exist bases B = {α1, . . . , αn} and B = {β1, . . . , βn} such that [S ]B = [T ]B′ . LetU be the operator which carries B onto B′. Then by Theorem 14, page 92, [US U−1]B′ = [U]−1

B[US U−1]B[U]B and by the

comments at the very bottom of page 90, this equals [U]−1B

[U]B[S ]B[U]−1B

[U]B which equals [S ]B, which we’ve assumedequals [T ]B′ . Thus [US U−1]B′ = [T ]B′ . Thus US U−1 = T .

Conversely, assume T = US U−1 for some invertible U. Let B be any ordered basis for V and let B′ be its image under U.Then [T ]B′ = [US U−1]B′ = [U]B′ [S ]B′ [U]−1

B′, which by Theorem 14, page 92, equals [S ]B (because U−1 carries B′ into B).

Thus [T ]B′ = [S ]B.

Exercise 10: We have seen that the linear operator T on R2 defined by T (x1, x2) = (x1, 0) is represented in the standardordered basis by the matrix

A =

[1 00 0

].

This operator satisfies T 2 = T . Prove that if S is a linear operator on R2 such that S 2 = S , then S = 0, or S = I, or there is anordered basis B for R2 such that [S ]B = A (above).

Solution: Suppose S 2 = S . Let ε1, ε2 be the standard basis vectors for R2. Consider {S ε1, S ε2}.

If both S ε1 = S ε2 = 0 then S = 0. Thus suppose WLOG that S ε1 , 0.

Page 73: Linear Algebra - Greg Grantgreggrant.org/hoffman_and_kunze.pdf · Linear Algebra Ho man & Kunze 2nd edition ... Section 1.2: Systems of Linear Equations 3 Solution: We must check

Section 3.4: Representation of Transformations by Matrices 71

First note that if x ∈ S (R2) then x = S (y) for some y ∈ R2 and therefore S (x) = S (S (y)) = S 2(y) = S (y) = x. In other wordsS (x) = x ∀ x ∈ S (R2).

Case 1: Suppose ∃ c ∈ R such that S ε2 = cS ε1. Then S (ε2 − cε1) = 0. In this case S is singular because it maps anon-zero vector to zero. Thus since S ε1 , 0 we can conclude that dim(S (R2)) = 1. Let α1 be a basis for S (R2). Letα2 ∈ R

2 be such that {α1, α2} is a basis for R2. Then Sα2 = kα1 for some k ∈ R. Let α′2 = α2 − kα1. Then {α1, α′2}

span R2 because if x = aα1 + bα2 then x = (a + bk)α1 + bα′2. Thus {α1, α′2} is a basis for R2. We now determine the

matrix of S with respect to this basis. Since α1 ∈ S (R2) and S (x) = x ∀ x ∈ S (R2), it follows that Sα1 = α1. Andconsequently S (α1) = 1 · α1 + 0 · α′2. Thus the first column of the matrix of S with respect to α1, α

′2 is [1, 0]T. Also

Sα′2 = S (α2 − kα1) = Sα2 − kSα1 = Sα2 − kα1 = kα1 − kα1 = 0 = 0 · α1 + 0 · α′2. So the second column of the matrix is[0, 0]T. Thus the matrix of S with respect to the basis {α1, α

′2} is exactly A.

Case 2: There does not exist c ∈ R such that S ε2 = cS ε1. In this case S ε1 and S ε2 are linearly independent from each other.Thus if we let αi = S εi then {α1, α2} is a basis for R2. Now by assumption S (x) = x ∀ x ∈ S (R2), thus Sα1 = α1 and Sα2 = α2.Thus the matrix of S with respect to the basis {α1, α2} is exactly the identity matrix I.

Exercise 11: Let W be the space of all n × 1 column matrices over a field F. If A is an n × n matrix over F, then A defines alinear operator LA on W through left multiplication: LA(X) = AX. Prove that every linear operator on W is left multiplicationby some n × n matrix, i.e., is LA for some A.

Now suppose V is an n-dimensional vector space over the field F, and let B be an ordered basis for V . For each α inV , define Uα = [α]B. Prove that U is an isomorphism of V onto W. If T is a linear operator on V , then UTU−1 is a linearoperator on W. Accordingly, UTU−1 is left multiplication by some n × n matrix A. What is A?

Solution: Part 1: I’m confused by the first half of this question because isn’t this exactly Theorem 11, page 87 in the specialcase V = W where B = B′ is the standard basis of Fn×1. This special case is discussed on page 88 after Theorem 12, and inparticular in Example 13. I don’t know what we’re supposed to add to that.

Part 2: Since U(cα1 + α2) = [cα1 + α2]B = c[α1]B + [α2]B = cU(α1) + U(α2), U is linear, we just must show it is invertible.Suppose B = {α1, . . . , αn}. Let T be the function from W to V defined as follows:

a1a2...

an

7→ a1α1 + · · · anαn.

Then T is well defined and linear and it is also clear by inspection that TU is the identity transformation on V and UT is theidentity transformation on W. Thus U is an isomorphism from V to W.

It remains to deterine the matrix of UTU−1. Now Uαi is the standard n×1 matrix with all zeros except in the i-th place whichequals one. Let B′ be the standard basis for W. Then the matrix of U with respect to B and B′ is the identity matrix. Likewisethe matrix of U−1 with respect to B′ and B is the identity matrix. Thus [UTU−1]B′ = I[T ]BI−1 = [T ]B. Therefore the matrixA is simply [T ]B, the matrix of T with respect to B.

Problem 12: Let V be an n-dimensional vector space over the field F, and let B = {α1, . . . , αn} be an ordered basis for V .

(a) According to Theorem 1, there is a unique linear operator T on V such that

Tα j = α j+1, j = 1, . . . , n − 1, Tαn = 0.

What is the matrix A of T in the ordered basis B.?

(b) Prove that T n = 0 but T n−1 , 0.

Page 74: Linear Algebra - Greg Grantgreggrant.org/hoffman_and_kunze.pdf · Linear Algebra Ho man & Kunze 2nd edition ... Section 1.2: Systems of Linear Equations 3 Solution: We must check

72 Chapter 3: Linear Transformations

(c) Let S be any linear operator on V such that S n = 0 but S n−1 , 0. Prove that there is an ordered basis B′ for V such thatthe matrix of S in the ordered basis B′ is the matrix A of part (a).

(d) Prove that if M and N are n×n matrices over F such that Mn = Nn = 0 but Mn−1 , 0 , Nn−1, then M and N are similar.

Solution: (a) The i-th column of A is given by the coefficients obtained by writing αi in terms of {α1, . . . , αn}. Since Tαi =

αi+1, i < n and Tαn = 0, the matrix is therefore

A =

0 0 0 0 · · · 0 01 0 0 0 · · · 0 00 1 0 0 · · · 0 00 0 1 0 · · · 0 0...

......

.... . .

......

0 0 0 0 · · · 1 0

.

(b) A has all zeros except 1’s along the diagonal one below the main diagonal. Thus A2 has all zeros except 1’s along thediagonal that is two diagonals below the main diagonal, as follows:

A2 =

0 0 0 0 · · · 0 00 0 0 0 · · · 0 01 0 0 0 · · · 0 00 1 0 0 · · · 0 00 0 1 0 · · · 0 0...

......

.... . .

......

0 0 0 0 · · · 0 0

.

Similarly A3 has all zeros except the diagonal three below the main diagonal. Continuing we see that An−1 is the matrix thatis all zeros except for the bottom left entry which is a 1:

An−1 =

0 0 0 0 · · · 0 00 0 0 0 · · · 0 00 0 0 0 · · · 0 00 0 0 0 · · · 0 0...

......

.... . .

......

1 0 0 0 · · · 0 0

.

Multiplying by A one more time then yields the zero matrix, An = 0. Since A represents T with respect to the basis B, and Ai

represents T i, we see that T n−1 , 0 and T n = 0.

(c) We will first show that dim(S k(V)) = n − k. Suppose dim(S (V)) = n. Then dim(S k(V)) = n ∀ k = 1, 2, . . . , whichcontradicts the fact that S n = 0. Thus it must be that dim(S (V)) ≤ n − 1. Now dim(S 2(V)) cannot be greater than dim(S (V))because a linear transformation cannot map a space onto one with higher dimension. Thus dim(S 2(V)) ≤ n − 1. Suppose thatdim(S 2(V)) = n − 1. Thus n − 1 = dim(S 2(V)) ≤ dim(S (V)) ≤ n − 1. Thus it must be that dim(S (V)) = n − 1. Thus S isan isomorphism on S (V) because S (V) and S (S (V)) have the same dimension. It follows that S k is also an isomorphism onS (V) ∀ k ≥ 2. Thus it follows that dim(S k(V)) = n− 1 for all k = 2, 3, 4, . . . , another contradiction. Thus dim(S 2(V)) ≤ n− 2.Suppose that dim(S 3(V)) = n − 2, then it must be that dim(S 2(V)) = n − 2 and therefore S is an isomorphism on S 2(V), fromwhich it follows that dim(S k(V)) = n−2 for all k = 3, 4, . . . , a contradiction. Thus dim(S 3(V)) ≤ n−3. Continuing in this waywe see that dim(S k(V)) ≤ n − k. Thus dim(S n−1(V)) ≤ 1. Since we are assuming S n−1 , 0 it follows that dim(S n−1(V)) = 1.We have seen that dim(S k(V)) cannot equal dim(S k+1(V)) for k = 1, 2, . . . , n − 1, thus it follows that the dimension must godown by one for each application of S . In other words dim(S n−2(V)) must equal 2, and then in turn dim(S n−3(V)) must equal3, and generally dim(S k(V)) = n − k.

Page 75: Linear Algebra - Greg Grantgreggrant.org/hoffman_and_kunze.pdf · Linear Algebra Ho man & Kunze 2nd edition ... Section 1.2: Systems of Linear Equations 3 Solution: We must check

Section 3.4: Representation of Transformations by Matrices 73

Now let α1 be any basis vector for S n−1(V) which we have shown has dimension one. Now S n−2(V) has dimension two andS takes this space onto a space S n−1(V) of dimension one. Thus there must be α2 ∈ S n−2(V) \ S n−1(V) such that S (α2) = α1.Since α2 is not in the space generated by α1 and {α1, α2} are in the space S n−2(V) of dimension two, it follows that {α1, α2}

is a basis for S n−2(V). Now S n−3(V) has dimension three and S takes this space onto a space S n−2(V) of dimension two.Thus there must be α3 ∈ S n−3(V) \ S n−2(V) such that S (α3) = α2. Since α3 is not in the space generated by α1 and α2and {α1, α2, α3} are in the space S n−3(V) of dimension three, it follows that {α1, α2, α3} is a basis for S n−3(V). Continuingin this way we produce a sequence of elements {α1, α2, . . . , αk} that is a basis for S n−k(V) and such that S (αi) = αi−1 for alli = 2, 3, . . . , k. In particular we have a basis {α1, α2, . . . , αn} for V and such that S (αi) = αi−1 for all i = 2, 3, . . . , n. Reversethe ordering of this bases to give B = {αn, αn−1, . . . , α1}. Then B therefore is the required basis for which the matrix of S withrespect to this basis will be the matrix given in part (a).

(d) Suppose S is the transformation of Fn×1 given by v 7→ Mv and similarly let T be the transformation v 7→ Nv. ThenS n = T n = 0 and S n−1 , 0 , T n−1. Then we know from the previous parts of this problem that there is a basis B for whichS is represented by the matrix from part (a). By Theorem 14, page 92, it follows that M is similar to the matrix in part (a).Likewise there’s a basis B′ for which T is represented by the matrix from part (a) and thus the matrix N is also similar to thematrix in part (a). Since similarity is an equivalence relation (see last paragraph page 94), it follows that since M and N aresimilar to the same matrix that they must be similar to each other.

Exercise 13: Let V and W be finite-dimensional vector spaces over the field F and let T be a linear transformation from Vinto W. If

B = {α1, . . . , αn} and B′ = {β1, . . . , βn}

are ordered bases for V and W, respectively, define the linear transformations Ep,q as in the proof of Theorem 5: Ep,q(αi) =

δi,qβp. Then the Ep,q, 1 ≤ p ≤ m, 1 ≤ q ≤ n, form a basis for L(V,W), and so

T =

m∑p=1

n∑q=1

ApqEp,q

for certain scalars Apq (the coordinates of T in this basis for L(V,W)). Show that the matrix A with entries A(p, q) = Apq isprecisely the matrix of T relative to the pair B, B′.

Solution: Let Ep,qM be the matrix of the linear transformation Ep,q with respect to the bases B and B′. Then by the formula for

a matrix associated to a linear transformation as given in the proof of Theorem 11, page 87, Ep,qM is the matrix all of whose

entries are zero except for the p, q-the entry which is one. Thus A =∑

p,q Ap,qEp,qM . Since the association between linear

transformations and matrices is an isomorphism, T 7→ A implies∑

p,q ApqEp,q 7→∑

p,q ApqEp,qM . And thus A is exactly the

matrix whose entries are the Apq’s.

Section 3.5: Linear FunctionalsPage 100: Typo line 5 from the top. It says f (αi) = αi, should be f (αi) = ai.

Page 100: In Example 22, it says the matrix 1 1 1t1 t2 t3t21 t2

2 t23

is invertible “as a short computation shows.” The way to see this is with what we know so far is to row reduce the matrix. Aslong as t1 , t2 we can get to

1 0 t2−t3t2−t1

0 1 t3−t1t2−t1

0 0 (t3−t1)(t3−t2)t22−t2

1

.

Page 76: Linear Algebra - Greg Grantgreggrant.org/hoffman_and_kunze.pdf · Linear Algebra Ho man & Kunze 2nd edition ... Section 1.2: Systems of Linear Equations 3 Solution: We must check

74 Chapter 3: Linear Transformations

Now we can continue and obtain 1 0 t2−t3

t2−t1

0 1 t3−t1t2−t1

0 0 1

as long as (t3 − t1)(t3 − t2) , 0. From there we can finish row-reducing to obtain the identity. Thus we can row-reduce thematrix to the identity if and only if t1, t2, t3 are distinct, that is no two of them are equal.

Exercise 1: In R3 let α1 = (1, 0, 1), α2 = (0, 1,−2), α3 = (−1,−1, 0).

(a) If f is a linear functional on R3 such that

f (α1) = 1, f (α2) = −1, f (α3) = 3,

and if α = (a, b, c), find f (α).

(b) Describe explicitly a linear functional f on R3 such that

f (α1) = f (α2) = 0 but f (α3) , 0.

(c) Let f be any linear functional such that

f (α1) = f (α2) = 0 and f (α3) , 0.

If α = (2, 3,−1), show that f (α) , 0.

Solution: (a) We need to write (a, b, c) in terms of α1, α2, α3. We can do this by row reducing the following augmented matrixwhose colums are the αi’s. 1 0 −1 a

0 1 −1 b1 −2 0 c

1 0 −1 a0 1 −1 b0 −2 −1 c − a

1 0 −1 a0 1 −1 b0 0 −1 c − a + 2b

1 0 −1 a0 1 −1 b0 0 1 a − 2b − c

1 0 0 2a − 2b − c0 1 0 a − b − c0 0 1 a − 2b − c

Thus if (a, b, c) = x1α1 + x2α2 + x3α3 then x1 = 2a − 2b − c, x2 = a − b − c and x3 = a − 2b − c. Now f (a, b, c) =

f (x1α1 + x2α2 + x3α3) = x1 f (α1) + x2 f (α2) + x3 f (α3) = (2a − 2b − c) · 1 + (a − b − c) · (−1) + (a − 2b − c) · 3 =

(2a − 2b − c) − (a − b − c) + (3a − 6b − 3c) = 4a − 7b − 3c. In summary

f (α) = 4a − 7b − 3c.

(b) Let f (x, y, z) = x − 2y − z. The f (1, 0, 1) = 0, f (0, 1,−2) = 0, and f (−1,−1, 0) = 1.

Page 77: Linear Algebra - Greg Grantgreggrant.org/hoffman_and_kunze.pdf · Linear Algebra Ho man & Kunze 2nd edition ... Section 1.2: Systems of Linear Equations 3 Solution: We must check

Section 3.4: Representation of Transformations by Matrices 75

(c) Using part (a) we know that α = (2, 3,−1) = −α1 − 3α3 (plug in a = 2, b = 3, c = −1 for the formulas for x1, x2, x3). Thusf (α) = − f (α1) − 3 f (α3) = 0 − 3 f (α3) and since f (α3) , 0, −3 f (α3) , 0 and thus f (α) , 0.

Exercise 2: Let B = {α1, α2, α3} be the basis for C3 defined by

α1 = (1, 0,−1), α2 = (1, 1, 1), α3 = (2, 2, 0).

Find the dual basis of B.

Solution: The dual basis { f1, f2, f3} are given by fi(x1, x2, x3) =∑3

j=1 Ai jx j where (A1,1, A1,2, A1,3) is the solution to the system 1 0 −1 11 1 1 02 2 0 0

,(A2,1, A2,2, A2,3) is the solution to the system 1 0 −1 0

1 1 1 12 2 0 0

,and (A3,1, A3,2, A3,3) is the solution to the system 1 0 −1 0

1 1 1 02 2 0 1

,We row reduce the generic matrix 1 0 −1 a

1 1 1 b2 2 0 c

→ 1 0 0 a + b − 1

2 c0 1 0 c − b − a0 0 1 b − 1

2 c

.a = 1, b = 0, c = 0⇒ f1(x1, x2, x3) = x1 − x2a = 0, b = 1, c = 0⇒ f2(x1, x2, x3) = x1 − x2 + x3a = 0, b = 0, c = 1⇒ f3(x1, x2, x3) = − 1

2 x1 + x2 −12 x3.

Then { f1, f2, f3} is the dual basis to {α1, α2, α3}.

Exercise 3: If A and B are n × n matrices over the field F, show that trace(AB) = trace(BA). Now show that similar matriceshave the same trace.

Solution: (AB)i j =∑n

k=1 AikBk j and (BA)i j =∑n

k=1 BikAk j. Thus

trace(AB) =

n∑i=1

(AB)ii =

n∑i=1

n∑k=1

AikBki =

n∑i=1

n∑k=1

BkiAik =

n∑k=1

n∑i=1

BkiAik =

n∑k=1

(BA)kk = trace(BA).

Suppose A and B are similar. Then ∃ an invertible n × n matrix P such that A = PBP−1. Thus trace(A) = trace(PBP−1) =

trace((P)(BP−1)) = trace((BP−1)(P)) = trace(B).

Exercise 4: Let V be the vector space of all polynomial functions p from R into R which have degree 2 or less:

p(x) = c0 + c1x + c2x2.

Define three linear functionals on V by

f1(p) =

∫ 1

0p(x)dx, f2(x) =

∫ 2

0p(x)dx, f3(x) =

∫ 3

0p(x)dx.

Page 78: Linear Algebra - Greg Grantgreggrant.org/hoffman_and_kunze.pdf · Linear Algebra Ho man & Kunze 2nd edition ... Section 1.2: Systems of Linear Equations 3 Solution: We must check

76 Chapter 3: Linear Transformations

Show that { f1, f2, f3} is a basis for V∗ by exhibiting the basis for V of which it is the dual.

Solution: ∫ a

0c0 + c1x + c2x2dx

= c0x +12

c1x2 +13

c2x3 |a0

= c0a +12

c1a2 +13

c2a3.

Thus ∫ 1

0p(x)dx = c1 +

12

c1 +13

c2∫ 2

0p(x)dx = 2c1 + 2c1 +

83

c2∫ 3

0p(x)dx = 3c1 +

92

c1 + 9c2

Thus we need to solve the following system three timesc1 + 1

2 c1 + 13 c2 = u

2c1 + 2c1 + 83 c2 = v

3c1 + 92 c1 + 9c2 = w

Once when (u, v,w) = (1, 0, 0), once when (u, v,w) = (0, 1, 0) and once when (u, v,w) = (0, 0, 1).

We therefore row reduce the following matrix 1 1/2 1/3 1 0 02 2 8/3 0 1 03 9/2 9 0 0 1

1 1/2 1/3 1 0 00 1 2 −2 1 00 3 8 −3 0 1

1 0 −2/3 2 −1/2 00 1 2 −2 1 00 0 2 3 −3 1

1 0 −2/3 2 −1/2 00 1 2 −2 1 00 0 1 3/2 −3/2 1/2

1 0 0 3 −3/2 1/30 1 0 −5 4 −10 0 1 3/2 −3/2 1/2

.Thus

α1 = 3 − 5x +32

x2

α2 = −32

+ 4x −32

x2

α3 =13− x +

12

x2.

Page 79: Linear Algebra - Greg Grantgreggrant.org/hoffman_and_kunze.pdf · Linear Algebra Ho man & Kunze 2nd edition ... Section 1.2: Systems of Linear Equations 3 Solution: We must check

Section 3.4: Representation of Transformations by Matrices 77

Exercise 5: If A and B are n × n complex matrices, show that AB − BA = I is impossible.

Solution: Recall for n × n matrices M, trace(M) =∑n

i=1 Mii. The trace is clearly additive trace(M1 + M2) = trace(M1) +

trace(M2). We know from Exercise 3 that trace(AB) = trace(BA). Thus trace(AB − BA) = trace(AB) − trace(BA) =

trace(AB) − trace(AB) = 0. But trace(I) = n and n , 0 in C.

Exercise 6: Let m and n be positive integers and F a field. Let f1, . . . , fm be linear functionals on Fn. For α in Fn define

T (α) = ( f1(α), . . . , fm(α)).

Show that T is a linear transformation from Fn into Fm. Then show that every linear transformation from Fn into Fm is of theabove form, for some f1, . . . , fm.

Solution: Clearly T is a well defined function from Fn into Fm. We must just show it is linear. Let α, β ∈ Fn, c ∈ C. Then

T (cα + β) = ( f1(cα + β), . . . , fm(cα + β))

= (c f1(α) + f1(β), . . . , c fn(α) + fn(β))

= c( f1(α), . . . , fn(α)) + ( f1(β), . . . , fn(β))

= cT (α) + T (β).

Thus T is a linear transformation.

Let S be any linear transformation from Fn to Fm. Let M be the matrix of S with respect to the standard bases of Fn andFm. Then M is an m × n matrix and S is given by X 7→ MX where we identify Fn as Fn×1 and Fm with Fm×1. Now for eachi = 1, . . . ,m let fi(x1, . . . , xn) =

∑nj=1 Mi jx j. Then X 7→ MX is the same as X 7→ ( f1(X), . . . , fm(x)) (keeping in mind our

identification of Fm with Fm×1). Thus S has been written in the desired form.

Exercise 7: Let α1 = (1, 0,−1, 2) and α2 = (2, 3, 1, 1), and let W be the subspace of R4 spanned by α1 and α2. Which linearfunctionals f :

f (x1, x2, x3, x4) = c1x1 + c2x2 + c3x3 + c4x4

are in the annihilator of W?

Solution: The two vectors α1 and α2 are linearly independent since neither is a multiple of the other. Thus W has dimension2 and {α1, α2} is a basis for W. Therefore a functional f is in the annihilator of W if and only if f (α1) = f (α2) = 0. We findsuch f by solving the system {

f (α1) = 0f (α2) = 0

or equivalently {c1 − c3 + 2c4 = 02c1 + 3c2 + c3 + c4 = 0

We do this by row reducing the matrix [1 0 −1 22 3 1 1

]→

[1 0 −1 20 1 1 −1

]Therefore

c1 = c3 − 2c4

c2 = −c3 + c4.

Page 80: Linear Algebra - Greg Grantgreggrant.org/hoffman_and_kunze.pdf · Linear Algebra Ho man & Kunze 2nd edition ... Section 1.2: Systems of Linear Equations 3 Solution: We must check

78 Chapter 3: Linear Transformations

The general element of W0 is therefore

f (x1, x2, x3, x4) = (c3 − 2c4)x1 + (c3 + c4)x2 + c3x3 + c4x4,

for arbitrary elements c3 and c4. Thus W0 has dimension 2 as expected.

Exercise 8: Let W be the subspace of R5 which is spanned by the vectors

α1 = ε1 + 2ε2 + ε3, α2 = ε2 + 3ε3 + 3ε4 + ε5

α3 = ε1 + 4ε2 + 6ε3 + 4ε4 + ε5.

Find a basis for W0.

Solution: The vectors α1, α2, α3 are linearly independent as can be seen by row reducing the matrix 1 2 1 0 00 1 3 3 11 4 6 4 1

1 2 1 0 00 1 3 3 10 2 5 4 1

1 0 −5 −6 −20 1 3 3 10 0 −1 −2 −1

1 0 −5 −6 −20 1 3 3 10 0 1 2 1

1 0 0 4 30 1 0 −3 −20 0 1 2 1

.Thus W has dimension 3 and {α1, α2, α3} is a basis for W. We know every functional is given by f (x1, x2, x3, x4, x5) =

c1x2 + c2x2 + c3x3 + c4x4 + c5x5 for some c1, . . . , c5. From the row reduced matrix we see that the general solution for anelement of W0 is

f (x1, x2, x3, x4, x5) = (−4c4 − 3c5)x1 + (3c4 + 2c5)x2 − (2c4 + c5)x3 + c4x4 + c5x5.

Exercise 9: Let V be the vector space of all 2 × 2 matrices over the field of real numbers, and let

B =

[2 −2−1 1

].

Let W be the subspace of V consisting of all A such that AB = 0. Let f be a linear functional on V which is in the annihilatorof W. Suppose that f (I) = 0 and f (C) = 3, where I is the 2 × 2 identity matrix and

C =

[0 00 1

].

Find f (B).

Page 81: Linear Algebra - Greg Grantgreggrant.org/hoffman_and_kunze.pdf · Linear Algebra Ho man & Kunze 2nd edition ... Section 1.2: Systems of Linear Equations 3 Solution: We must check

Section 3.4: Representation of Transformations by Matrices 79

Solution: The general linear functional on V is of the form f (A) = aA11 +bA12 +cA21 +dA22 for some a, b, c, d ∈ R. If A ∈ Wthen [

x yz w

] [2 −2−1 1

]=

[0 00 0

]implies y = 2x and w = 2y. So W consists of all matrices of the form[

x 2xy 2y

]

Now f ∈ W0 ⇒ f([

x 2xy 2y

])= 0 ∀ x, y ∈ R⇒ ax + 2bx + cy + 2dy = 0 ∀ x, y ∈ R⇒ (a + 2b)x + (c + 2d)y = 0 ∀ x, y ∈ R

⇒ b = − 12 a and d = − 1

2 c. So the general f ∈ W0 is of the form

f (A) = aA11 −12

aA12 + cA21 −12

cA22.

Now f (C) = 3⇒ d = 3⇒ − 12 c = 3⇒ c = −6. And f (I) = 0⇒ a − 1

2 c = 0⇒ c = 2a⇒ a = −3. Thus

f (A) = −3A11 +32

A12 − 6A21 + 3A22.

Thusf (B) = −3 · 2 +

32· (−2) − 6 · (−1) + 3 · 1 = 0.

Exercise 10: Let F be a subfield of the complex numbers. We define n linear functionals on Fn (n ≥ 2) by

fk(x1, . . . , xn) =

n∑j=1

(k − j)x j, 1 ≤ k ≤ n.

What is the dimension of the subspace annihilated by f1, . . . , fn?

Solution: N fk is the subspace annihilated by fk. By the comments on page 101, N fk has dimension n − 1. Now the standardbasis vector ε2 is in N f2 but is not in N f1 . Thus N f1 and N f2 are distinct hyperspaces. Thus their intersection has dimensionn− 2. Now ε3 is in N f3 but is not in N f1 ∪N f2 . Thus N f1 ∩N f2 ∩N f3 is the intersection of three distinct hyperspaces and so hasdimension n − 3. Continuing in this way, εi < ∪

i−1j=1N fi . Thus ∪i

j=1N fi is the intersection of i distinct hyperspaces and so hasdimension n − i. Thus when i = n we have ∪n

j=1N fi has dimension 0.

Exercise 11: Let W1 and W2 be subspace of a finite-dimensional vector space V .

(a) Prove that (W1 + W2)0 = W01 ∩W0

2 .

(b) Prove that (W1 ∩W2)0 = W01 + W0

2 .

Solution: (a) f ∈ (W1 +W2)0⇒ f (v) = 0 ∀ v ∈ W1 +W2⇒ f (w1 +w2) = 0 ∀ w1 ∈ W1, w2 ∈ W2⇒ f (w1) = 0 ∀ w1 ∈ W1 (takew2 = 0) and f (w2) = 0 ∀ w2 ∈ W2 (take w1 = 0). Thus f ∈ W0

1 and f ∈ W02 . Thus f ∈ W0

1 ∩W02 . Thus (W1 + W2)0 ⊆ W0

1 ∩W02 .

Conversely, let f ∈ W01∩W0

2 . Let v ∈ W1+W2. Then v = w1+w2 where wi ∈ Wi. Thus f (v) = f (w1+w2) = f (w1)+ f (w2) = 0+0(since f ∈ W0

1 and f ∈ W02 ). Thus f (v) = 0 ∀ v ∈ W1 + W2. Thus f ∈ (W1 + W2)0. Thus W0

1 ∩W02 ⊆ (W1 + W2)0.

Since (W1 + W2)0 ⊆ W01 ∩W0

2 and W01 ∩W0

2 ⊆ (W1 + W2)0 it follows that W01 ∩W0

2 = (W1 + W2)0

(b) f ∈ W01 + W0

2 ⇒ f = f1 + f2, for some fi ∈ W0i . Now let v ∈ W1 ∩W2. Then f (v) = ( f1 + f2)(v) = f1(v) + f2(v) = 0 + 0.

Thus f ∈ (W1 ∩W2)0. Thus W01 + W0

2 ⊆ (W1 ∩W2)0.

Page 82: Linear Algebra - Greg Grantgreggrant.org/hoffman_and_kunze.pdf · Linear Algebra Ho man & Kunze 2nd edition ... Section 1.2: Systems of Linear Equations 3 Solution: We must check

80 Chapter 3: Linear Transformations

Now let f ∈ (W1 ∩W2)0. In the proof of Theorem 6 on page 46 it was shown that we can choose a basis for W1 + W2

{α1, . . . , αk, β1, . . . , βm, γ1, . . . , γn}

where {α1, . . . , αk} is a basis for W1 ∩W2, {α1, . . . , αk, β1, . . . , βm} is a basis for W1 and {α1, . . . , αk, γ1, . . . , γn} is a basisfor W2. We expand this to a basis for all of V

{α1, . . . , αk, β1, . . . , βm, γ1, . . . , γn, λ1, . . . , λ`}.

Now the general element v ∈ V can be written as

v =

k∑i=1

xiαi +

m∑i=1

yiβi +

n∑i=1

ziγi +∑i=1

wiλi (20)

and f is given by

f (v) =

k∑i=1

aixi +

m∑i=1

biyi +

n∑i=1

cizi +∑i=1

diwi

for some constants ai, bi, ci, di. Since f (v) = 0 for all v ∈ W1 ∩W2, it follows that a1 = · · · = ak = 0. So

f (v) =∑

biyi +∑

cizi +∑

diwi.

Definef1(v) =

∑cizi +

∑diwi

andf2(v) =

∑biyi.

Then f = f1 + f2. Now if v ∈ W1 then

v =

k∑i=1

xiαi +

m∑i=1

yiβi

so that the coefficients zi and wi in (20) are all zero. Thus f1(v) = 0. Thus f1 ∈ W01 . Similarly if v ∈ W2 then the coefficients yi

and wi in (20) are all zero and thus f2(v) = 0. So f2 ∈ W2. Thus f = f1 + f2 where f1 ∈ W01 and f2 ∈ W0

2 . Thus f ∈ W01 + W0

2 .Thus (W1 ∩W2)0 ⊆ W0

1 + W02 .

Thus (W1 ∩W2)0 ⊆ W01 + W0

2 .

Exercise 12: Let V be a finite-dimensional vector space over the field F and let W be a subspace of V . If f is a linearfunctional on W, prove that there is a linear functional g on V suvch that g(α) = f (α) for each α in the subspace W.

Solution: Let B be a basis for W and let B′ be a basis for V such that B ⊆ B′. A linear function on a vector space is uniquelydetermined by its values on a basis, and conversely any function on the basis can be extended to a linear function on the space.Thus we define g on B by g(β) = f (β) ∀ β ∈ B. Then define g(β) = 0 for all β ∈ B′ \ B. Since we have defined g on B′ itdefines a linear functional on V and since it agrees with f on a basis for W it agrees with f on all of W.

Exercise 13: Let F be a subfield of the field of complex numbers and let V be any vector space over F. Suppose that f andg are linear functionals on V such that the function h defined by h(α) = f (α)g(α) is also a linear functional on V . Prove thateither f = 0 or g = 0.

Solution: Suppose neither f nor g is the zero function. We will derive a contradiction. Let v ∈ V . Then h(2v) = f (2v)g(2v) =

4 f (v)g(v). But also h(2v) = 2h(v) = 2 f (v)g(v). Therefore f (v)g(v) = 2 f (v)g(v) ∀ v ∈ V . Thus f (v)g(v) = 0 ∀ v ∈ V .Let B be a basis for V . Let B1 = {β ∈ B | f (β) = 0} and B2 = {β ∈ B | g(β) = 0}. Since f (β)g(β) = 0 ∀ β ∈ B,we have B = B1 ∪ B2. Suppose B1 ⊆ B2. Then B2 = B and consequently g is the zero function. Thus B1 * B2. And

Page 83: Linear Algebra - Greg Grantgreggrant.org/hoffman_and_kunze.pdf · Linear Algebra Ho man & Kunze 2nd edition ... Section 1.2: Systems of Linear Equations 3 Solution: We must check

Section 3.4: Representation of Transformations by Matrices 81

similarly B2 * B1. Thus we can choose β1 ∈ B1 \ B2 and β2 ∈ B2 \ B1. So we have f (β2) , 0 and g(β1) , 0. Thenf (β1 + β2)g(β1 + β2) = f (β1)g(β1) + f (β2)g(β1) + f (β1)g(β2) + f (β2)g(β2). Since f (β1) = g(β2) = 0, this equals f (β2)g(β1)which is non-zero since each term is non-zero. And this contradicts the fact that f (v)g(v) = 0 ∀ v ∈ V .

Exercise 14: Let F be a field of characteristic zero and let V be a finite-dimensional vector space over F. If α1, . . . , αm arefinitely many vectors in V , each different from the zero vector, prove that there is a linear functional f on V such that

f (αi) , 0, i = 1, . . . ,m.

Solution: Re-index if necessary so that {α1, . . . , αk} is a basis for the subspace generated by {α1, . . . , αm}. So each αk+1, . . . , αm

can be written in terms of α1, . . . , αk. Extend {α1, . . . , αk} to a basis for V

{α1, . . . , αk, β1, . . . , βn}.

For each i = k + 1, . . . ,m write αi =∑k

j=1 Ai jα j. Since αk+1, . . . , αm are all non-zero, for each i = k + 1, . . . ,m ∃ ji ≤ ksuch that Ai ji , 0. Now define f by mapping α1, . . . , αk to k arbitrary non-zero values and map βi to zero ∀ i. Thenf (αk+1) =

∑kj=1 Ak+1, j f (α j). If f (αk+1) = 0 then leaving f (αi) fixed for all i ≤ k and adjusting f (α jk+1 ), it equals zero for ex-

actly one possible value of f (α jk+1 ) (since Ak+1, jk+1 , 0). Thus we can redefine f (α jk+1 ) so that f (αk+1) , 0 while maintainingf (α jk+1 ) , 0.

Now if f (αk+2) = 0, then leaving f (αi) fixed for i , jk+2, it equals zero for exactly one possible value of f (α jk+2 ) (sinceAk+2, jk+2 , 0) So we can adjust f (α jk+2 ) so that f (αk+2) , 0 and f (αk+1) , 0 and f (αk+2) , 0 simultaneously.

Continuing in this way we can adjust f (α jk+3 ), . . . , f (α jm ) as necessary until all f (αk+1), . . . , f (αm) are non-zero and also allof f (α1), . . . , f (αk) are non-zero.

Exercise 15: According to Exercise 3, similar matrices have the same trace. Thus we can define the trace of a linear operatoron a finite-dimensional space to be the trace of any matrix which represents the operator in an ordered basis. This is well-defined since all such representing matrices for one operator are similar.

Now let V be the space of all 2× 2 matrices over the field F and let P be a fixed 2× 2 matrix. Let T be the linear operatoron V defined by T (A) = PA. Prove that trace(T ) = 2trace(P).

Solution: Write

P =

[P11 P12P21 P22

].

Let

e11 =

[1 00 0

], e12 =

[0 10 0

]e21 =

[0 01 0

], e22 =

[0 00 1

]Then B = {e11, e12, e21, e22} is an ordered basis for V . We find the matrix of the linear transformation with respect to this basis.

T (e11) =

[P11 0P21 0

]= P11e11 + P21e21

T (e12) =

[0 P110 P21

]= P11e12 + P21e22

T (e21) =

[P21 0P22 0

]= P12e11 + P22e21

T (e22) =

[0 P120 P22

]= P12e12 + P22e22.

Page 84: Linear Algebra - Greg Grantgreggrant.org/hoffman_and_kunze.pdf · Linear Algebra Ho man & Kunze 2nd edition ... Section 1.2: Systems of Linear Equations 3 Solution: We must check

82 Chapter 3: Linear Transformations

Thus the matrix of T with respect to B is P11 0 P12 00 P11 0 P12

P21 0 P22 00 P21 0 P22

.The trace of this matrix is 2P11 + 2P22 = 2trace(P).

Exercise 16: Show that the trace functional on n × n matrices is unique in the following sense. If W is the space of n × nmatrices over the field F and if f is a linear functional on W such that f (AB) = f (BA) for each A and B in W, then f is ascalar multiple of the trace function. If, in addition, f (I) = n then f is the trace function.

Solution: Let A and B be n × n matrices. The `,m entry in AB is

(AB)`m =

n∑k=1

A`kBkm (21)

and the `,m entry in BA is

(BA)`m =

n∑k=1

B`kAkm. (22)

Fix i, j ∈ {1, . . . , n} such that i > j. Let A be the matrix where Ai j = 1 and all other entries are zero. Let B be the matrix whereBii = 1 and all other entries are zero. Consider the general element of AB

(AB)`m =

n∑k=1

A`kBkm.

The only non-zero A in the sum on the right is Ai j. But B jm = 0 since j > i and only Bii , 0. Thus AB is the zero matrix.

Now we compute BA. From (22) the only non-zero term is when ` = i, m = j and k = i.

Thus the matrix AB has zeros in every position except for the i, j position where it equals one.

Now the general functional on n × n matrices is of the form

f (M) =

n∑`=1

n∑m=1

c`mM`m

for some constants c`m. Now f (AB) = f (0) = 0 and f (BA) = ci j. So if f (AB) = f (BA) then it follows that ci j = 0.

Thus we have shown that ci j = 0 for all i > j. Similarly ci j = 0 for all i < j. Thus the only possible non-zero coefficients arec11, . . . , cnn.

f (M) =

n∑i=1

ciiMii.

We will be done if we show c11 = cmm for all m = 2, . . . , n. Fix 2 ≤ i ≤ n. Let A be the matrix such that A11 = Ai1 = 1and A`m = 0 in all other positions. Let B = AT. Then AB is zero in every position except A11 = A1i = Ai1 = Aii = 1. AndBA is zero in every position except (BA)11 = 2. Thus f (AB) = c11 + cii and f (BA) = 2c11. Thus if f (AB) = f (BA) thenc11 + cii = 2c11 which implies c11 = cii. Thus there’s a constant c such that cii = c for all i.

Thus f is given by

f (M) =

n∑k=1

cMii.

Page 85: Linear Algebra - Greg Grantgreggrant.org/hoffman_and_kunze.pdf · Linear Algebra Ho man & Kunze 2nd edition ... Section 1.2: Systems of Linear Equations 3 Solution: We must check

Section 3.4: Representation of Transformations by Matrices 83

If f (I) = n then c = 1 and we have the trace function.

Exercise 17: Let W be the space of n × n matrices over the field F, and let W0 be the subspace spanned by the matricesC of the form C = AB − BA. Prove that W0 is exactly the subspace of matrices which have trace zero. (Hint: What is thedimension of the space of matrices of trace zero? Use the matrix ’units,’ i.e., matrices with exactly one non-zero entry, toconstruct enough linearly independent matrices of the form AB − BA.)

Solution: Let W ′ = {w ∈ W | trace(w) = 0}. We want to show W ′ = W0. We know from Exercise 3 that trace(AB − BA) = 0for all matrices A, B. Since matrices of the form AB−BA span W0, it follows that trace(M) = 0 for all M ∈ W0. Thus W0 ⊆ W ′.

Since the trace function is a linear functional, the dimension of W ′ is dim(W)−1 = n2−1. Thus if we show the dimension of W0is also n2−1 then we will be done. We do this by exhibiting n2−1 linearly independent elements of W0. Denote by Ei j the ma-trix with a one in the i, j position and zeros in all other positions. Let Hi j = Eii − E j j. Let B = {Ei j | i , j} ∪ {H1,i | 2 ≤ i ≤ n}.We will show that B ⊆ W0 and that B is a linearly independent set. First, it clear that they are linearly independent be-cause Ei j is the only vector in B with a non-zero value in the i, j position and H1,i is the only vector in B with a non-zerovalue in the i, i position. Now 2Ei j = Hi jEi j − Ei jHi j and Hi j = Ei jE ji − E jiEi j. Thus Ei j ∈ W0 and Hi j ∈ W0. Now|B| = |{Ei j | i , j}| + |{H1,i | 2 ≤ i ≤ n}| = (n2 − n) + (n − 1) = n2 − 1 Thus we are done.

Section 3.6: The Double DualExercise 1: Let n be a positive integer and F a field. Let W be the set of all vectors (x1, . . . , xn) in Fn such that x1 +· · ·+xn = 0.

(a) Prove that W0 consists of all linear functionals f of the form

f (x1, . . . , xn) = cn∑

j=1

x j.

(b) Show that the dual space W∗ of W can be ‘naturally’ identified with the linear functionals

f (x1, . . . , xn) = c1x1 + · · · + cnxn

on Fn which satisfy c1 + · · · + cn = 0.

Solution: (a) Let g be the functional g(x1, . . . , xn) = x1 + · · · + xn. Then W is exactly the kernel of g. Thus dim(W) = n − 1.Let αi = ε1 − εi+1 for i = 1, . . . , n − 1. Then {α1, . . . , αn−1} are linearly independent and are all in W so they must be a basisfor W. Let f (x1, . . . , xn) = c1x1 + · · · + cnxn be a linear functional. Then f ∈ W0 ⇒ f (α1) = · · · = f (αn) = 0⇒ c1 − ci = 0 ∀i = 2, . . . , n⇒ ∃ c such that ci = c ∀ i. Thus f (x1, . . . , xn) = c(x1 + · · · + xn).

(b) Consider the sequence of functionsW → (Fn)∗ → W∗

where the first function is(c1, . . . , cn) 7→ fc1,...,cn

where fc1,...,cn (x1, . . . , xn) = c1x1 + · · · cnxn and the second function is restriction from Fn to W. We know both W and W∗ havethe same dimension. Thus if we show the composition of these two functions is one-to-one then it must be an isomorphism.Suppose (c1, . . . , cn) ∈ W 7→ fc1,...,cn = 0 ∈ W∗.

Then∑

ci = 0 and∑

cixi = 0 for all (x1, . . . , xn) ∈ W.

In other words∑

ci = 0 and∑

cixi = 0 for all (x1, . . . , xn) such that∑

xi = 0.

Page 86: Linear Algebra - Greg Grantgreggrant.org/hoffman_and_kunze.pdf · Linear Algebra Ho man & Kunze 2nd edition ... Section 1.2: Systems of Linear Equations 3 Solution: We must check

84 Chapter 3: Linear Transformations

Let {α1, . . . , αn−1} be the basis for W from part (a). Then fc1,...,cn (αi) = 0 ∀ i = 1, . . . , n−1; which implies c1 = ci ∀ i = 2, . . . , n.Thus

∑ci = (n − 1)c1. But

∑ci = 0, thus c1 = 0. Thus fc1,...,cn is the zero function.

Thus the mapping W → W∗ is a natural isomorphism. We therefore naturally identify each element in W∗ with a linearfunctional f (x1, . . . , xn) = c1x1 + · · · cnxn where

∑ci = 0.

Exercise 2: Use Theorem 20 to prove the following. If W is a subspace of a finite-dimensional vector space V and if{g1, . . . , gr} is any basis for W0, then

W =

r⋂i=1

Ngi .

Solution: {g1, . . . , gr} a basis for W0 ⇒ gi ∈ W0 ∀ i ⇒ gi(W) = 0 ∀ i ⇒ W ⊆ Ngi ∀ i ⇒ W ⊆⋂r

i=1 Ngi . Let n = dim(V).By Theorem 2, page 71, we know dim(Ng1 ) = n − 1. Since the gi’s are linearly independent, g2 is not a multiple of g1, thusby Theorem 20, Ng1 * Ng2 . Thus dim(Ng1 ∩ Ng2 ) ≤ n − 2. By Theorem 20 again, Ng3 * Ng1 ∩ Ng2 since g3 is not a linearcombination of g1 and g2. Thus dim(Ng1 ∩ Ng2 ∩ Ng3 ) ≤ n − 3. By induction dim(

⋂ri=1 Ngi ) ≤ n − r. Now by Theorem 16,

dim(W) = n − r. Thus since W ⊆⋂r

i=1 Ngi , it follows that dim(⋂r

i=1 Ngi ) ≥ n − r. Thus it must be that dim(⋂r

i=1 Ngi ) = n − rand it must be that W =

⋂ri=1 Ngi since we have shown the left hand side is contained in the right hand side and both sides

have the same dimension.

Exercise 3: Let S be a set, F a field, and V(S ; F) the space of all functions from S into F:

( f + g)(x) = f (x) + g(x)

(c f )(x) = c f (x).

Let W be any n-dimensional subspace of V(S ; F). Show that there exist points x1, . . . , xn in S and functions f1, . . . , fn in Wsuch that fi(x j) = δi j.

Solution: I’m not sure using the double dual is really the easiest way to prove this. It can be done rather easily directly byinduction on n (in fact see question 121704 on math.stackexchange.com). However, since H&K clearly want this done withthe double dual. At first glance you might try to think of W as a dual on S and W∗ as the double dual somehow. But thatdoesn’t work since S is just a set. Instead I think you have to consider the double dual of W, W∗∗ to make it work. I came upwith the following solution.

Let s ∈ S . We first show that the function

φs :W → F

w 7→ w(s)

is a linear functional on W (in other words for each s, we have φs ∈ W∗).

Let w1,w2 ∈ W, c ∈ F. Then φs(cw1 + w2) = (cw1 + w2)(s) which by definition equals cw1(s) + w2(s) which equalscφs(w1) + φs(w2). Thus φs is a linear functional on W.

Suppose φs(w) = 0 for all s ∈ S , w ∈ W. Then w(s) = 0 ∀ s ∈ S , w ∈ W, which implies dim(W) = 0. So as long as n > 0,∃ s1 ∈ S such that φs1 (w) , 0 for some w ∈ W. Equivalently there is an s1 ∈ S and a w1 ∈ W such that w1(s1) , 0. Thismeans φs1 , 0 as elements of W∗. It follows that 〈φs1〉, the subspace of W∗ generated by φs1 , has dimension one. By scalingif necessary, we can further assume w1(s1) = 1.

Now suppose ∀ s ∈ S that we have φs ∈ 〈φs1〉, the subspace of W∗ generated by φs1 . Then for each s ∈ S there is a c(s) ∈ Fsuch that φs = c(s)φs1 in W∗. Then for each s ∈ S , w(s) = c(s)w(s1) for all w ∈ W. In particular w1(s) = c(s) (recallw1(s1) = 1). Let w ∈ W. Let b = w(s1). Then w(s) = c(s)w(s1) = bw1(s) ∀ s ∈ S . Notice that b depends on w but does notdepend on s. Thus w = bw1 as functions on S where b ∈ F is a fixed constant. Thus w ∈ 〈w1〉, the subspace of W generated

Page 87: Linear Algebra - Greg Grantgreggrant.org/hoffman_and_kunze.pdf · Linear Algebra Ho man & Kunze 2nd edition ... Section 1.2: Systems of Linear Equations 3 Solution: We must check

Section 3.7: The Transpose of a Linear Transformation 85

by w1. Since w was arbitrary, it follows that dim(W) = 1. Thus as long as dim(W) ≥ 2 we can find w2 ∈ W and s2 ∈ Ssuch that 〈w1,w2〉 (the subspace of W generated by w1,w2) and 〈φs1 , φs2〉 (the subspace of W∗ generated by {φs1 , φs2 }) bothhave dimension two. Let W0 = 〈w1,w2〉. Then we’ve shown that {φs1 , φs2 } is a basis for W∗0 . Therefore there’s a dual basis{F1, F2} ⊆ W∗∗0 ; so that Fi(φs j ) = δi j, i, j ∈ {1, 2}. By Theorem 17, ∃ corresponding w1,w2 ∈ W so that Fi = Lwi (in thenotation of Theorem 17). Therefore, δi j = Fi(φs j ) = Lwi (φs j ) = φs j (wi) = wi(s j), for i, j ∈ {1, 2}.

Now suppose ∀ s ∈ S that we have φs ∈ 〈φs1 , φs2〉 ⊆ W∗. Then ∀ s ∈ S , there are constants c1(s), c2(s) ∈ F and we havew(s) = c1(s)w(s1) + c2(s)w(s2) for all w ∈ W. Similar to the argument in the previous paragraph, this implies dim(W) ≤ 2(for w ∈ W let b1 = w(s1) and b2 = w(s2) and argue as before). Therefore, as long as dim(W) ≥ 3 we can find s3 so that〈φs1 , φs2 , φs3〉 ⊆ W∗, the subspace of W∗ generated by φs1 , φs2 , φs3 , has dimension three. And as before we can find w3 ∈ Wsuch that wi(s j) = δi j, for i, j ∈ {1, 2, 3}.

Continuing in this way we can find n elements s1, . . . , sn ∈ S such that φs1 , . . . , φsn are linearly independent in W∗ and corre-sponding elements w1, . . . ,wn ∈ W such that wi(s j) = δi j. Let fi = wi and we are done.

Section 3.7: The Transpose of a Linear TransformationExercise 1: Let F be a field and let f be the linear functional on F2 defined by f (x1, x2) = ax1 +bx2. For each of the followinglinear operators T , let g = f t f , and find g(x1, x2).

(a) T (x1, x2) = (x1, 0);

(b) T (x1, x2) = (−x2, x1);

(c) T (x1, x2) = (x1 − x2, x1 + x2).

Solution:

(a) g(x1, x2) = T t f (x1, x2) = f (T (x1, x2)) = f (x1, 0) = ax1.

(b) g(x1, x2) = T t f (x1, x2) = f (T (x1, x2)) = f (−x2, x1) = −axs + bx1.

(c) g(x1, x2) = T t f (x1, x2) = f (T (x1, x2)) = f (x1 − x2, x1 + x2) = a(x1 − x2) + b(x1 + x2) = (a + b)x1 + (b − a)x2.

Exercise 2: Let V be the vector space of all polynomial functions over the field of real numbers. Let a and b be fixed realnumbers and let f be the linear functional on V defined by

f (p) =

∫ b

ap(x)dx.

If D is the differentiation operator on V , what is Dt f ?

Solution: Let p(x) = c0 + c1x + · · · + cnxn. Then

Dt f (p) = f (D(p))

= f (c1 + 2c2x + 3c3x2 + · · · + ncnxn−1)

=c1 + c2x2 + · · · cnxn |b0

=p(b) − p(a)

Exercise 3: Let V be the space of all n × n matrices over a field F and let B be a fixed n × n matrix. If T is the linear operatoron V defined by T (A) = AB − BA, and if f is the trace function, what is T t f ?

Page 88: Linear Algebra - Greg Grantgreggrant.org/hoffman_and_kunze.pdf · Linear Algebra Ho man & Kunze 2nd edition ... Section 1.2: Systems of Linear Equations 3 Solution: We must check

86 Chapter 3: Linear Transformation

Solution: By exercise 3 in section 3.5, we know trace(AB) = trace(BA). Thus T t f (A) = f (T (A)) = trace(AB − BA) =

trace(AB) − trace(BA) = 0.

Exercise 4: Let V be a finite-dimensional vector space over the field F and let T be a linear operator on V . Let c be a scalarand suppose there is a non-zero vector α in V such that Tα = cα. Prove that there is a non-zero linear functional f on V suchthat T t f = c f .

Solution: Consider the operator U = T − cI. Then U(α) = 0 so rank(U) < n. Therefore rank(U t) < n as an operator on V∗.It follows that there’s a f ∈ V∗ such that U t( f ) = 0. Now U t = T t − cI, thus T t( f ) = c f .

Exercise 5: Let A be an m × n matrix with real entries. Prove that A = 0 if and only if trace(AtA) = 0.

Solution: Suppose A is the m × n matrix with entries ai j and B is the n × k matrix with entries bi j. Then the i, j entry of AB is

n∑k=1

aikbk j.

Substituting At for A and A for B we get the i, j entry of AtA is (note that AtA has dimension n × n)

m∑k=1

akiak j.

Thus the diagonal entries of AtA arem∑

k=1

akiaki

for i = 1, . . . , n. Thus the trace isn∑

i=1

m∑k=1

a2ki.

If all ai j ∈ R then this sum is zero if and only if each ai j = 0 because every one of them appears in this sum.

Exercise 6: Let n be a positive integer and let V be the space of all polynomial functions over the field of real numbers whichhave degree at most n, i.e., functions of the form

f (x) = c0 + c1x + · · · + cnxn.

Let D be the differentiation operator on V . Find a basis for the null space of the transpose operator Dt.

Solution: The null space of Dt consists of all linear functionals g : V → R such that Dtg = 0, or equivalently g ◦ D( f ) = 0 ∀f ∈ V . Now g(a0 +a1x+ · · ·+anxn) = c0a0 +c1a1 + · · ·+cnan for some consants c0, . . . , cn ∈ R. So g◦D(a0 +a1x+ · · ·+anxn) =

g(a1 + 2a2x + · · · + nanxn−1) =∑n−1

i=0 (i + 1)ciai+1.

This sum∑n−1

i=0 (i + 1)ciai+1 equals zero for all vectors (a0, a1, . . . , an) ∈ Rn if and only if c0 = c1 = · · · = cn−1 = 0. While cn

can be anything. Therefore the null space has dimension one and a basis is given by taking cn = 1, which gives the functiong :

∑aixi 7→ an, the projection onto the xn coordinate.

Exercise 7: Let V be a finite-dimensional vector space over the field F. Show that T → T t is an isomorphism of L(V,V) ontoL(V∗,V∗).

Solution: Choose a basis B for V . This gives an isomorphism L(V,V)→ Mn the space of n × n matrices. And the dual basisB′ gives an isomorphism L(V∗,V∗)→ Mn. Now we know by Theorem 23 that the following diagram of functions commutes.

Page 89: Linear Algebra - Greg Grantgreggrant.org/hoffman_and_kunze.pdf · Linear Algebra Ho man & Kunze 2nd edition ... Section 1.2: Systems of Linear Equations 3 Solution: We must check

Section 3.7: The Transpose of a Linear Transformation 87

This means if we start at L(V,V) and follow two functinos to the Mn in the bottom left, it doesn’t matter which way aroundthe diagram we go, we end up at the same place.

L(V,V) −→ Mn

transpose ↓ ↓ transposeL(V∗,V∗) −→ Mn

Both horizontal arrows are isomorphisms (by Theorem 12, page 88). Now clearly transpose on matrices is a one-to-one andonto function from the set of matrices to itself. Also (rA)t = rAt and (A + B)t = At + Bt for any two n × n matrices A andB. Thus transpose is also a linear transformation. Thus transpose is an isomorphism. Therefore three of the arrows in thisdiagram are isomorphisms and it follows that the fourth arrow must also be an isomorphism.

Exercise 8: Let V be the vector space of n × n matrices over the field F.

(a) If B is a fixed n × n matrix, define a function fB on V by FB(A) = trace(BtA). Show that FB is a linear functional on V .

(b) Show that every linear functional on V is of the above form, i.e., is fB for some B.

(c) Show that B→ fB is an isomorphism of V onto V∗.

Solution:

(a) This follows from the fact that the trace function is a linear functional and left multiplication by a matrix is a linear transfor-mation from V to V . In other words FB(cA1 +A2) = trace(Bt(cA1 +A2)) = trace(cBtA1 +BtA2) = c · trace(BtA1)+ trace(BtA2) =

c · FB(A1) + FB(A2).

(b) Let f : V → F be a linear functional. Let A = (ai j) ∈ V . Then

f (A) =

n∑i, j=1

ci jai j (23)

for some fixed constants ci j ∈ F.

Now let B = (bi j) ∈ V be any matrix. Then the i, j element of BtA is∑n

k=1 bkiak j. Thus

trace(BtA) =

n∑i=1

n∑k=1

bkiaki. (24)

Comparing (23) and (24) we see each ai j apperas exactly once in each sum. So setting bki = cki for all i, k = 1, . . . , n we getthe appropriate matrix B such that trace(BtA) = f .

(c) Let F be the function F : V → V∗ such that F(B) = fB. Part (a) shows this function is into V∗. Part (b) shows it is onto V∗.We must show it is linear and one-to-one. Let r ∈ F and B1, B2 ∈ V . Then (rB1 + B2)tA = (rBt

1 + Bt2)A = rBt

1A + Bt2A. We

know the trace function itself is linear. Thus F is linear. Now suppose trace(BtA) = 0 ∀ A ∈ V . Fix i, j ∈ {1, 2, · · · , n}. Let Abe the matrix with a one in the i, j position and zeros elsewhere. Then by the proof of part (b) we know that trace(BtA) = bi j.So if F(A) = 0 then bi j = 0. Thus B must be the zero matrix. Thus F is one-to-one. It follows that F is an isomorphism.

Page 90: Linear Algebra - Greg Grantgreggrant.org/hoffman_and_kunze.pdf · Linear Algebra Ho man & Kunze 2nd edition ... Section 1.2: Systems of Linear Equations 3 Solution: We must check

88 Chapter 3: Linear Transformation

Page 91: Linear Algebra - Greg Grantgreggrant.org/hoffman_and_kunze.pdf · Linear Algebra Ho man & Kunze 2nd edition ... Section 1.2: Systems of Linear Equations 3 Solution: We must check

Chapter 4: Polynomials

Section 4.2: The Algebra of PolynomialsExercise 1: Let F be a subfield of the complex numbers and let A be the following 2 × 2 matrix over F

A =

[2 1−1 3

].

For each of the following polynomials f over F, compute f (A).

(a) f = x2 − x + 2;

(b) f = x3 − 1;

(c) f = x2 − 5x + 7;

Solution:

(a)

A2 =

[2 1−1 3

[2 1−1 3

]

=

[3 5−5 8

]Therefore

f (A) = A2 − A + 2

=

[3 5−5 8

]−

[2 1−1 3

]+

[2 00 2

]

=

[3 4−4 7

](b)

A2 =

[2 1−1 3

[2 1−1 3

[2 1−1 3

]

=

[3 5−5 8

[2 1−1 3

]

=

[1 18−18 19

]Therefore

f (A) = A3 − 1

89

Page 92: Linear Algebra - Greg Grantgreggrant.org/hoffman_and_kunze.pdf · Linear Algebra Ho man & Kunze 2nd edition ... Section 1.2: Systems of Linear Equations 3 Solution: We must check

90 Chapter 4: Polynomials

=

[1 18−18 19

]−

[1 00 1

]

=

[0 18−18 18

](c)

f (A) = A2 − 5A + 7

=

[3 5−5 8

]− 5

[2 1−1 3

]+

[7 00 7

]

=

[0 00 0

]Exercise 2: Let T be the linear operator on R3 defined by

T (x1, x2, x3) = (x1, x3,−2x2 − x3).

Let f be the polynomial over R defined by f = −x3 + 2. Find f (T ).

Solution: The matrix of T with respect to the standard basis is

A =

1 0 00 0 10 −2 −1

Thus

A2 =

1 0 00 0 10 −2 −1

· 1 0 0

0 0 10 −2 −1

=

1 0 00 −2 −10 2 −1

So

A3 =

1 0 00 −2 −10 2 −1

· 1 0 0

0 0 10 −2 −1

=

1 0 00 2 −10 2 3

Thus

−A3 + 2 = −

1 0 00 2 −10 2 3

+

2 0 00 2 00 0 2

=

−1 0 00 0 10 −2 −1

.This corresponds to the transformation

f (T )(x1, x2, x3) = (−x1, x3,−x2 − x3).

Page 93: Linear Algebra - Greg Grantgreggrant.org/hoffman_and_kunze.pdf · Linear Algebra Ho man & Kunze 2nd edition ... Section 1.2: Systems of Linear Equations 3 Solution: We must check

Section 4.2: The Algebra of Polynomials 91

Exercise 3: Let A be an n × n diagonal matrix over the field F, i.e., a matrix satisfying Ai j = 0 for i , j. Let f be thepolynomial over F defined by

f = (x − A11) · · · (x − Ann).

What is the matrix f (A)?

Solution: The product of n diagonal matrices is again a diagonal matrix, where the i, i element is the product of the entriesin the i, i position of the n individual matrices. For each i, the i, i terms of A − Aii is zero. Therefore each diagonal entry of(A − A11) · · · (A − Ann) is a product of numbers one of which is zero. Therefore (A − A11) · · · (A − Ann) is the zero matrix.

Exercise 4: If f and g are independent polynomials over the field F and h is a non-zero polynomial over F, show that f h andgh are independent.

Solution: Suppose there are scalars r, s ∈ F such that r f h + sgh =. Then r f h = −sgh and so by Corollary 2 on page 121,it follows that r f = −sg so that r f + sg = 0. Since f and g are independent, it follows that r = s = 0. Thus f h and gh areindependent.

Exercise 5: If F is a field, show that the product of two non-zero elements of F∞ is non-zero.

Solution: Let f = ( f0, f1, f2, . . . ) and g = (g0, g1, g2, . . . ) be two elements of F∞. Let n be the index of the first non-zero fiand let m be the index of the first non-zero gi (could be n = m = 0). Then what is the n + m coordinate of the product? It is

( f g)n+m =

n+m∑i=0

fign+m−i =

n+m∑i=n

fign+m−i

and if i > n then gn+m−i = 0 thusn+m∑i=n

fign+m−i =

n∑i=n

fign+m−i = fngm.

Now we’ve assumed fn and gm are non-zero thus fngm , 0 and thus f g , 0 in F∞.

Exercise 6: Let S be a set of non-zero polynomials over a field F. If no two elements of S have the same degree, show thatS is an independent set in F[x].

Solution: Let f1, . . . , fn ∈ S . Suppose deg( fi) = d and deg( f j) < d ∀ i , j. We can do this because by assumption allpolynomials in the set { f1, . . . , fn} have different degrees, so one of them must have the largest. Suppose the d-th coefficientof fi is r , 0. Then any linear combination a1 f1 + · · · an fn has d-th coefficient equal to rai. Thus if this linear combinationis zero then necessarily ai = 0. We can now apply the same argument to the f j with the second largest degree to show itscoefficient in the linear combination is zero. And then to the third largest, etc. Until we have eventually shown all coefficientsin the linear combination are zero. It follows that { f1, . . . , fn} is a linearly independent subset of S and since it was an arbitraryfinite subset of S it follows that S is a linearly independent set.

Exercise 7: If a and b are elements of a field F and a , 0, show that the polynomials 1, ax + b, (ax + b)2, (ax + b)3, . . . forma basis of F[x].

Solution: Let S = {1, ax + b, (ax + b)2, (ax + b)3, . . . }. And let 〈S 〉 be the subspace spanned by S . By the previous exercisewe know S is a linearly independent set. We must just show S spans the space of all polynomials. Since 1 ∈ S and ax + b ∈ Sit follows that b · 1 + 1

a (a + bx) ∈ 〈S 〉. Thus x ∈ 〈S 〉. Now we can subtract a multiple of 1 and a multiple of x from (a + bx)2

to get a2x2 ∈ 〈S 〉. Thus 1a2 · a2x2 ∈ 〈S 〉. Thus x2 ∈ S . Continuing in this way we can show that xn ∈ 〈S 〉 for all n. Since

{1, x, x2, . . . } span the space of all polynomials, it follows that S spans the space of all polynomials.

Page 94: Linear Algebra - Greg Grantgreggrant.org/hoffman_and_kunze.pdf · Linear Algebra Ho man & Kunze 2nd edition ... Section 1.2: Systems of Linear Equations 3 Solution: We must check

92 Chapter 4: Polynomials

Exercise 8: If F is a field and h is a polynomial over F of degree ≥ 1, show that the mapping f → f (h) is a one-one lineartransformation of F[x] into F[x]. Show that this transformation is an isomorphism of F[x] onto F[x] if and only if deg h = 1.

Solution: Let G : F[x] → F[x] be the function G( f ) = f (h). Clearly G is a well-defined function from F[x] to F[x]. Bydefinition G( f + g) = ( f + g)(h) = f (h) + g(h) = G( f ) + G(g) and for r ∈ F, G(r f ) = (r f )(h) = r · f (h) = rG(F). Thus G isa linear transformation. Suppose deg h > 1. Then the coefficient of x in f (h) is zero. Thus if deg h > 1 then G is not onto.Now suppose deg h = 0. Then f (h) is a scalar for all f . Thus G is not onto. Now suppose deg h = 1, so that h(x) = a + bx.Let h′ = 1

b x − ab and let G′ be the corresponding function on F[x], so G′ : F[x] → F[x] is given by G( f ) = f ( 1

b x − ab ). Then

G ◦G′ and G′ ◦G both give the identify function on F[x]. Thus G is an isomorphism.

Exercise 9: Let F be a subfield of the complex numbers and let T , D be the transformations on F[x] defined by

T

n∑i=0

cixi

=

n∑i=0

ci

1 + ixi+1

and

D

n∑i=0

cixi

=

n∑i=1

icixi−1.

(a) Show that T is a non-singular linear operator on F[x]. Show also that T is not invertible.

(b) Show that D is a linear operator on F[x] and find its null space.

(c) Show that DT = I, and T D , I.

(d) Show that T [(T f )g] = (T f )(Tg) − T [ f (Tg)] for all f , g in F[x].

(e) State and prove a rule for D similar to the one given for T in (d)

(f) Suppose V is a non-zero subspace of F[x] such that T f belongs to V for each f in V . Show that V is not finite-dimensional.

(g) Suppose V is a finite-dimensional subspace of F[x]. Prove there is an integer m ≥ 0 such that Dm f = 0 for each f in V .

Solution:

(a) Clearly T is a function from F[x] to F[x]. We must show T is linear.

T

n∑i=0

cixi +

n∑i=0

c′i xi

= T

n∑i=0

(ci + c′i)xi

=

n∑i=0

ci + c′ii + 1

xi+1

=

n∑i=0

(ci

1 + ixi+1 +

c′i1 + i

xi+1)

= T

n∑i=0

cixi

+ T

n∑i=0

c′i xi

.and

T

r n∑i=0

cixi

= T

n∑i=0

rcixi

=

n∑i=0

rci

i + 1xi+1 = r

n∑i=0

ci

i + 1xi+1 = r · T

n∑i=0

cixi

.Thus T is linear.

Page 95: Linear Algebra - Greg Grantgreggrant.org/hoffman_and_kunze.pdf · Linear Algebra Ho man & Kunze 2nd edition ... Section 1.2: Systems of Linear Equations 3 Solution: We must check

Section 4.2: The Algebra of Polynomials 93

Since F has characterisitc zero we can find a, b ∈ F, such that a , b. Consider a and b as constant polynomials in F. ThenT (a) = T (b) = 0. Thus T is not one-to-one. Thus T is not invertible.

(b) Clearly D is a function from F[x] to F[x]. We must show D is linear.

D

n∑i=0

cixi +

n∑i=0

c′i xi

= D

n∑i=0

(ci + c′i)xi

=

n∑i=1

i(ci + c′i)xi−1

=

n∑i=1

(icixi−1 + ic′i xi−1)

= D

n∑i=0

cixi

+ D

n∑i=0

c′i xi

.and

D

r n∑i=0

cixi

= D

n∑i=0

rcixi

=

n∑i=1

rcixi−1 = rn∑

i=1

cixi−1 = r · D

n∑i=0

cixi

.Thus D is linear.

Suppose f (x) =∑n

i=0 cixi is in the null space of D. Then D( f ) =∑n

i=1 icixi−1 = 0. A polynomial is zero if and only if everycoefficient is zero. Thus it must be that 0 = c1 = c2 = c3 = · · · . So it must be that f (x) = c0 a constant polynomial. Thus thenull space of D contains the constant polynomials. Since D( f ) = 0 for all constant polynomials, the null space of D consistsof exatly the constant polynomials.

(c)

D

T n∑i=0

cixi

= D

n∑i=0

ci

1 + ixi+1

.The first non-zero term of this sum is the linear term c0x. Thus when we apply D the sum still starts at zero:

=

n∑i=0

(i + 1)ci

1 + ixi+1−1

=

n∑i=0

cixi.

Thus

D

T n∑i=0

cixi

=

n∑i=0

cixi.

Thus DT = I.

Let f (x) = 1. Then T D( f ) = T (D( f )) = T (0) = 0. Thus T D(1) , 1. Thus T D , I.

(d) This follows rather easily from part (e). And likewise (e) follows rather easily from (d). Thus one can derive (d) straightfrom the definition of T and then derive (e) from it, or one can derive (e) straight from the definition of D and then derive (d)

Page 96: Linear Algebra - Greg Grantgreggrant.org/hoffman_and_kunze.pdf · Linear Algebra Ho man & Kunze 2nd edition ... Section 1.2: Systems of Linear Equations 3 Solution: We must check

94 Chapter 4: Polynomials

from it. I’ve chosen to do the latter.

In part (e) below the product formula is proven straight from the definition. So we will use it here to prove this part. Inparticular, we apply the product formula from part (e) to (T f )(Tg)

D[(T f )(Tg)] = (Tg)D(T f ) + (T f )D(Tg).

By part (c) DT = I so this is equivalent to

D[(T f )(Tg)] = f (Tg) + (T f )g.

ThusD[(T f )(Tg)] − f (Tg) = (T f )g.

Now apply T to both sides

T(D[(T f )(Tg)] − f (Tg)

)= T ((T f )g).

Since T is a linear transformation this is equivalent to

T(D[(T f )(Tg)]

)− T [ f (Tg)] = T ((T f )g).

We showed in part (c) that T D , I, however if f has constant term equal to zero then in fact T (D( f )) does equal f . Now T fand Tg have constant term equal to zero, so (T f )(Tg) has constant term zero, thus

T(D[(T f )(Tg)]

)= (T f )(Tg).

Thus(T f )(Tg) − T [ f (Tg)] = T ((T f )g).

(e) I believe they are after the product formula here:

D( f g) = f D(g) + gD( f ). (25)

We prove this by brute force appealing just to the definition and to the product formula for polynomials. Let f (x) =∑n

i=0 cixi

and g(x) =∑m

i=0 dixi. Then using the product formula (4-8) on page 121 we have

D( f g) = D

n∑i=0

cixim∑

i=0

dixi

= D

n+m∑i=0

i∑j=0

c jdi− j

xi

And using the linearity of the differentiation operator D this equals

n+m∑i=0

i

i∑j=0

c jdi− j

xi−1. (26)

Now we write down the sum for the right hand side of (25):

f D(g) + gD( f )

=

n∑i=0

cixim∑

i=1

idixi−1 +

n∑i=1

icixi−1m∑

i=0

dixi. (27)

Consider

x ·n∑

i=0

cixim∑

i=1

idixi−1.

Page 97: Linear Algebra - Greg Grantgreggrant.org/hoffman_and_kunze.pdf · Linear Algebra Ho man & Kunze 2nd edition ... Section 1.2: Systems of Linear Equations 3 Solution: We must check

Section 4.3: Lagrange Interpolation 95

This equalsn∑

i=0

cixim∑

i=1

idixi

and since 0d0 = 0 we can write this asn∑

i=0

cixim∑

i=0

idixi.

It’s straightforward to apply (4-8) page 121 to this product. In (4-8) we let fi = ci and gi = idi and it equals

m+n∑i=0

i∑j=0

(i − j)c jdi− j

xi.

The constant terms is zero thus we can write it as

m+n∑i=1

i∑j=0

(i − j)c jdi− j

xi.

And thus the sumn∑

i=0

cixim∑

i=1

idixi−1

equalsm+n∑i=1

i∑j=0

(i − j)c jdi− j

xi−1.

Similary the second sum isn+m∑i=1

i∑j=0

jc jdi− j

xi.

Thus (27) does equal (26).

(f) Suppose V is finite dimensional. Let {b1, · · · , bn} be a basis for V . Let d = maxi=1,···n deg(bi). It follows from Theo-rem 1(v) and induction that the degree of a linear combination of polynomials is no larger than the max of the degress ofthe individual polynomials involved in the linear combination. Thus no element of V has degree greater than d. Now letf ∈ V be any non-zero element. Let d′ = deg( f ). Then T f has degree d′ + 1, T 2 f has degree d′ + 2, etc. Thus for some n,deg(T n f ) > d. If T n f ∈ V then this is a contradiction. Thus if T n f ∈ V for all f ∈ V it must be that V is not finite dimensional.

(g) Let {b1, · · · , bn} be a basis for V . Let d = maxi=1,···n deg(bi). For any f ∈ F[x], we know deg(D f ) < deg( f ). ThusDd+1bi = 0 for all i = 1, . . . , n. Since Dd+1(bi) = 0 for all elements of the basis {b1, . . . , bn} it follows that Dd+1( f ) = 0 for allf ∈ V .

Section 4.3: Lagrange Interpolation

Exercise 1: Use the Lagrange interpolation formula to find a polynomial f with real coefficients such that f has degree ≤ 3and f (−1) = −6, f (0) = 2, f (1) = −2, f (2) = 6.

Page 98: Linear Algebra - Greg Grantgreggrant.org/hoffman_and_kunze.pdf · Linear Algebra Ho man & Kunze 2nd edition ... Section 1.2: Systems of Linear Equations 3 Solution: We must check

96 Chapter 4: Polynomials

Solution: t0 = −1, t1 = 0, t2 = 1, t3 = 2. Therefore

P0 =x(x − 1)(x − 2)(−1)(−2)(−3)

= −16 x(x − 1)(x − 2)

P1 =(x + 1)(x − 1)(x − 2)

(−1)(−2)= 1

2 (x − 1)(x + 1)(x − 2)

P2 =(x + 1)x(x − 2)

(2)(1)(−1)= −1

2 x(x + 1)(x − 2)

P3 =(x + 1)x(x − 1)

(3)(2)(1)= 1

6 (x − 1)(x + 1).

Thusf = f (−1) · P0 + f (0) · P1 + f (1) · P2 + f (2) · P3

= −6P0 + 2P1 − 2P2 + 6P3

= x(x − 1)(x − 2) + (x − 1)(x + 1)(x − 2) + x(x + 1)(x − 2) + x(x − 1)(x + 1)

= (x3 − 3x2 + 2x) + (x3 − 2x2 − x + 2) + (x3 − x2 − 2x) + (x3 − x)

= 4x3 − 6x2 − 2x + 2.

Checking:f (−1) = −4 − 6 + 2 + 2 = −6

f (0) = 2

f (1) = 4 − 6 − 2 + 2 = −2

f (2) = 32 − 24 − 4 + 2 = 6.

Exercise 2: Let α, β, γ, δ be real numbers. We ask when it is possible to find a polynomial f over R, of degree not more than2, such that f (−1) = α, f (1) = β, f (3) = γ and f (0) = δ. Prove that this is possible if and only if

3α + 6β − γ − 8δ = 0.

Solution: Let t0 = −1, t1 = 1, t2 = 3. We will apply the Lagrange interpolation formula to get a quadratic satisfying f (t0) = α,f (t1) = β, f (t2) = γ. Then we will figure out what condition on α, β, γ, δ will guarantee that it also satisfies f (0) = δ.

P0 =(x − 1)(x − 3)

(−2)(−4)= 1

8 (x − 1)(x − 3)

P1 =(x + 1)(x − 3)

(2)(−2)=−1

4 (x + 1)(x − 3)

P2 =(x + 1)(x − 1)

(4)(2)= 1

8 (x + 1)(x − 1)

Thereforef =

α

8(x − 1)(x − 3) −

β

4(x + 1)(x − 3) +

δ

8(x + 1)(x − 1)

= 18 (αx2 − 4αx + 3α − 2βx2 + 4βx + 6β + γx2 − γ).

Now f (0) = γ implies18

(3α + 6β − γ) = δ.

Simplifying gives3α + 6β − γ − 8δ = 0. (28)

Page 99: Linear Algebra - Greg Grantgreggrant.org/hoffman_and_kunze.pdf · Linear Algebra Ho man & Kunze 2nd edition ... Section 1.2: Systems of Linear Equations 3 Solution: We must check

Section 4.3: Lagrange Interpolation 97

Thus if (28) is satisfied then the four values of f are as required. Since three points determine a quadratic, there cannot be anyquadratic other than f that goes through (−1, α), (1, β), (3, δ). Thus this condition is not only sufficient but it is necessary.

Exercise 3: Let F be the field of real numbers,

A =

2 0 0 00 2 0 00 0 3 00 0 0 1

p =(x − 2)(x − 3)(x − 1).

(a) Show that p(A) = 0.

(b) Let P1, P2, P3 be the Lagrange polynomials for t1 = 2, t2 = 3, t2 = 1. Compute Ei = Pi(A), i = 1, 2, 3.

(c) Show that E1 + E2 + E3 = I, EiE j = 0 if i , j, E2i = Ei.

(d) Show that A = 2E1 + 3E2 + E3.

Solution: (a)(A − 2)(A − 3)(A − 1)

=

0 0 0 00 0 0 00 0 1 00 0 0 −1

−1 0 0 00 −1 0 00 0 0 00 0 0 −2

1 0 0 00 1 0 00 0 2 00 0 0 0

=

0 0 0 00 0 0 00 0 1 00 0 0 −1

−1 0 0 00 −1 0 00 0 0 00 0 0 0

=

0 0 0 00 0 0 00 0 0 00 0 0 0

.(b) t1 = 2, t2 = 3, t3 = 1.

P1 = − (x − 3)(x − 1)

P2 = 12 (x − 2)(x − 1)

P3 = 12 (x − 2)(x − 3)

Thus

E1 = P1(A) = −(A − 3I)(A − I) = −

−1 0 0 00 −1 0 00 0 0 00 0 0 −2

1 0 0 00 1 0 00 0 2 00 0 0 0

=

1 0 0 00 1 0 00 0 0 00 0 0 0

E2 = P2(A) =12

0 0 0 00 0 0 00 0 1 00 0 0 −1

1 0 0 00 1 0 00 0 2 00 0 0 0

=

0 0 0 00 0 0 00 0 1 00 0 0 0

Page 100: Linear Algebra - Greg Grantgreggrant.org/hoffman_and_kunze.pdf · Linear Algebra Ho man & Kunze 2nd edition ... Section 1.2: Systems of Linear Equations 3 Solution: We must check

98 Chapter 4: Polynomials

E3 = P3(A) =12

0 0 0 00 0 0 00 0 1 00 0 0 −1

−1 0 0 00 −1 0 00 0 0 00 0 0 −2

=

0 0 0 00 0 0 00 0 0 00 0 0 1

.(c) All three of these facts are basically obvious by visual inspection of the matrices in part (b). E1 + E2 + E3 = I is obviousby inspection. Likewise it is evident by inspection that EiE j = 0 if i , j. Lastly it is obvious that E2

i = Ei. I’m not sure whatthere is to prove here.

(d)2E1 + 3E2 + E3

= 2

1 0 0 00 1 0 00 0 0 00 0 0 0

+ 3

0 0 0 00 0 0 00 0 1 00 0 0 0

+

0 0 0 00 0 0 00 0 0 00 0 0 1

=

2 0 0 00 2 0 00 0 3 00 0 0 1

.Exercise 4: Let p = (x − 2)(x − 3)(x − 1) and let T be any linear operator on R4 such that p(T ) = 0. Let P1, P2, P3 be theLagrange polynomials of Exercise 3, and let Ei = Pi(T ), i = 1, 2, 3. Prove that

E1 + E2 + E3 = I, EiE j = 0 if i , j,

E2i = Ei, and T = 2E1 + 3E2 + E3.

Solution: Recall

P1 = − x2 + 4x − 3

P2 = 12 x2 − 3

2 x + 1

P2 = 12 x2 − 5

2 x + 3

By definition P(T ) + Q(T ) = (P + Q)(T ) and P(T )Q(T ) = (PQ)(T ). Now as polynomials P1 + P2 + P3 = 1. ThusE1 + E2 + E3 = P1(T ) + P2(T ) + P3(T ) = (P1 + P2 + P3)(T ) = I.

Notice that P divides PiP j whenever i , j. Thus PiP j = PQ for some Q (as polynomials). Thus EiE j = Pi(T )P j(T ) =

(PiP j)(T ) = P(T )Q(T ) = 0 · Q(T ) = 0. Thus EiE j(T ) = 0.

We prove the next part in general. Let

f1 =x − a

f2 =x − b

f3 =x − c

Thus

P1 =f2 f3

(a − b)(a − c)

P2 =f1 f3

(b − a)(b − c)

P3 =f1 f2

(c − a)(c − b)

Page 101: Linear Algebra - Greg Grantgreggrant.org/hoffman_and_kunze.pdf · Linear Algebra Ho man & Kunze 2nd edition ... Section 1.2: Systems of Linear Equations 3 Solution: We must check

Section 4.3: Lagrange Interpolation 99

Let d = 2c − b − a. Then it follows by simply multiplying it out that

d f3 + f 23

(c − a)(c − b)=

f1 f2(c − a)(c − b)

− 1.

Which is equivalent tod f3 + f 2

3

(c − a)(c − b)= P3 − 1. (29)

This equation is true as polynomials. We now evaluate things at T .

f1(T ) f2(T ) f3(T ) = 0

=⇒

f1(T ) f2(T ) f 23 (T ) = 0

=⇒

f1(T ) f2(T )(c − a)(c − b)

·f 23 (T )

(c − a)(c − b)= 0.

Since f1(T ) f2(T ) f3(T ) = 0, this implies

f1(T ) f2(T )(c − a)(c − b)

·d f3(T ) + f 2

3 (T )(c − a)(c − b)

= 0.

Equivalently

P3(T ) ·d f3(T ) + f 2

3 (T )(c − a)(c − b)

= 0.

By (29) this impliesP3(T )(P3(T ) − 1) = 0.

ThusP2

3(T ) = P3(T ).

Thus E23 = E3. Since a, b, c were general, the same follows for E1 and E2.

It remains to show T = 2E1 + 3E2 + E3. We first note that as polynomials

2P1 + 3P2 + P3

= (−2x2 + 8x − 6) + ( 32 x2 − 9

2 x + 3) + ( 12 x2 − 5

2 x + 3)

= x.

Plugging in T we get2P1(T ) + 3P2(T ) + P3(T ) = T.

Thus2E1 + 3E2 + E3 = T.

.Exercise 5: Let n be a positive integer and F a field. Suppose T is an n × n matrix over F and P is an invertible n × n matrixover F. If f is any polynomial over F, prove that

f (P−1T P) = P−1 f (T )P.

Page 102: Linear Algebra - Greg Grantgreggrant.org/hoffman_and_kunze.pdf · Linear Algebra Ho man & Kunze 2nd edition ... Section 1.2: Systems of Linear Equations 3 Solution: We must check

100 Chapter 4: Polynomials

Solution: First note that (P−1xP)n = P−1xn(T )P. This is obvious by inspection, it follows basically from the fact that multi-plication is associative and P−1P = I.

The general result now followsP−1 f (T )P

= P−1(a0 + a1T + a2T 2 + · · · + anT n)P

= P−1a0P + P−1a1T P + P−1a2T 2P + · · · + P−1anT nP

= a0 + a1(P−1T P) + a2(P−1T P)2 + · · · + an(P−1T P)n

= f (P−1T P).

Exercise 6: Let F be a field. We have considered certain special linear functionals on F[x] obtained via ‘evaluation at t’:

L( f ) = f (t).

Such functionals are not only linear but also have the property that L( f g) = L( f )L(g). Prove that if L is any linear functionalon F[x] such that

L( f g) = L( f )L(g)

for all f and g, then either L = 0 or there is a t in F such that L( f ) = f (t) for all f .

Solution: Let L be a non-zero linear transformation. First note that L(1) , 0 since otherwise L( f ) = L( f · 1) = L( f )L(1) =

L( f ) · 0 = 0 ∀ f . Next note that L(1) = L(1 · 1) = L(1)L(1)⇒ L(1) = 1. It follows that L(a) = L(a · 1) = aL(1) = a ∀ a ∈ F.Now let t = L(x). Let f (x) = a0 + a1x + · · · + anxn. Then

L( f )

= L(a0 + a1x + · · · + anxn)

= L(a0) + L(a1)L(x) + L(a2)L(x2) · · · + L(an)L(xn)

= a0 + a1L(x) + a2L(x)2 + · · · + anL(x)n

= a0 + a1t + a2t2 + · · · + antn

= f (t).

Section 4.4: Polynomial IdealsPage 129: In the statement of Theorem 5 it says “If f is a polynomial over f ” which I believe should be “If f is a polynomialover F”.

Page 133: The page’s running title says “Polynomial Ideas” it should say ”Polynomial Ideals”.

Page 134: In exercise 2 (a) and (c) I think they made a calculation mistake because both problems are extremely tedious todo with only what we know so far. I believe in part (a) they really meant 2x5 − x3 − 3x2 − 6x + 6 and in part (c) they reallymeant x3 + 6x2 + 7x + 2. If I were teaching out of this book I would change both problems to be like this. In any case, I solvedthem below as stated in the book, for the sake of completeness. But the derivations are rather ugly.

Exercise 1: Let Q be the field of rational numbers. Determine which of the following subsets of Q[x] are ideals. When theset is an ideal, find its monic generator.

(a) all f of even degree;

(b) all f of degree ≥ 5;

Page 103: Linear Algebra - Greg Grantgreggrant.org/hoffman_and_kunze.pdf · Linear Algebra Ho man & Kunze 2nd edition ... Section 1.2: Systems of Linear Equations 3 Solution: We must check

Section 4.4: Polynomial Ideals 101

(c) all f such that f (0) = 0;

(d) all f such that f (2) = f (4) = 0;

(e) all f in the range of the linear operator T defined by

T

n∑i=0

cixi

=

n∑i=0

ci

i + 1xi+1.

Solution:

(a) This is not an ideal. Let f (x) be any polynomial of even degree. Let g(x) = x. Then deg( f g) = deg( f ) + 1 which is oddand is therefore the set of polynomials of even degree does not satisfy the necessary property that f g is in the set whenever fis in the set.

(b) This is not an ideal. Let f (x) = x5 and g(x) = −x5 + x4. Then f and g are in the set and therefore f + g must be in theset if it is an ideal. But f (x) + g(x) = x4 has degree equal to four and therefore is not in the set. In other words the set is notclosed with respect to addition and therefore is not even a subspace.

(c) This is an ideal. Let I = { f ∈ F[x] | f (0) = 0}. If f (0) = 0 and g(0) = 0 then (c f + g)(0) = c f (0) + g(0) = 0 thus I isa subspace of F[x]. Now suppose f ∈ I and g ∈ F[x]. Then ( f g)(0) = f (0)g(0) = 0 · g(0) = 0. Thus f g ∈ I. Thus I is anideal. Let f (x) = a0 + a1x + · · · + anxn. Then f (0) = a0. Thus if f ∈ I then necessarily a0 = 0. Let g(x) = x. Then g ∈ I andf (x) = a1x + a2x2 + · · · anxn = x(a1 + a2x + · · · anxn−1) ∈ I. Thus g generates I.

(d) This is an ideal. Let I = { f ∈ F[x] | f (2) = f (4) = 0}. If f (2) = f (4) = 0 and g(2) = g(4) = 0 then (c f + g)(2) =

c f (2) + g(2) = 0 and (c f + g)(4) = c f (4) + g(4) = 0 thus I is a subspace of F[x]. Now suppose f ∈ I and g ∈ F[x].Then ( f g)(2) = f (2)g(2) = 0 · g(2) = 0 and ( f g)(4) = f (4)g(4) = 0 · g(4) = 0. Thus f g ∈ I. Thus I is an ideal. Letg(x) = (x − 2)(x − 4). We claim g generates I. Let f (x) ∈ I. Then since f (2) = 0, by Corollary 1 page 128 it follows thatf (x) = (x − 2)q(x) for some q(x) ∈ F[x]. Now since f (4) = 2 · q(4) = 0 it follows that q(4) = 0 Thus there is a p(x) such thatq(x) = (x − 4)p(x). Thus f (x) = (x − 2)(x − 4)p(x) = g(x)p(x) and therefore f (x) is in the ideal generated by g(x).

(e) This is an ideal. In fact it is the same ideal as in part (c). Note that T ( f ) has no constant term, thus T ( f )(0) = 0 ∀ f ∈ F[x].Now let f (x) = a1x + a2x2 + · · · + anxn. Let g(x) = a1 + 2a2x + 3a3x2 + · · · + nanxn−1. Then f = T (g). Thus f is in the imageof T . Thus any polynomial with zero constant term is in the image of T . Thus it is exactly the same ideal is in part (c).

Exercise 2: Find the g.c.d. of each of the following paris of polynomials

(a) 2x5 − x3 − 3x2 − 6x + 4, x4 + x3 − x2 − 2x − 2;

(b) 3x4 + 8x2 − 3, x3 + 2x2 + 3x + 6;

(c) x4 − 2x3 − 2x2 − 2x − 3, x3 + 6x2 + 7x + 1.

Solution:

This question on its face seems to depend on what the base field is. I’m going to assume it is C.

(a) Let f (x) = 2x5 − x3 − 3x2 − 6x + 4 and g(x) = x4 + x3 − x2 − 2x − 2. I have a feeling there’s a mistake and they reallymeant 2x5 − x3 − 3x2 − 6x + 6 because then both polynomials are divisible by x2 − 2. But as it is, the g.c.d. of these twopolynomials equals 1 - which does not seem that easy to prove using only what we know up to this point. If we knew aboutunique factorization in C[x] we could simply factor g(x) into linear factors and check none of the roots of g are roots of f . Infact the roots of g(x) are ±

√2 and −1±

√2i

2 .

Page 104: Linear Algebra - Greg Grantgreggrant.org/hoffman_and_kunze.pdf · Linear Algebra Ho man & Kunze 2nd edition ... Section 1.2: Systems of Linear Equations 3 Solution: We must check

102 Chapter 4: Polynomials

But we can’t use that argument yet. Instead we are probably expected to argue using the comments on page 132 right afterthe proof of Theorem 7, commonly known as the “euclidean algorithm”.

Since I believe there’s a mistake and it should be 2x5 − x3 − 3x2 − 6x + 6, I’m going to solve it both ways.

First we solve it exactly as stated in the book.

Solution 1 (as stated in book).

Let I = 〈 f (x), g(x)〉 be the ideal in C[x] generated by f and g. Then

2x5 − x3 − 3x2 − 6x + 4 = (x4 + x3 − x2 − 2x − 2)(2x − 2) + (3x3 − x2 − 6x).

Therefore 3x3 − x2 − 6x = f (x) − (2x − 2)g(x) ∈ I.

x4 + x3 − x2 − 2x − 2 = (3x3 − x2 − 6x)(

13 x + 4

9

)+

(139 x2 + 2

3 x − 2).

Therefore 139 x2 + 2

3 x − 2 ∈ I.

3x3 − x2 − 6x =(

139 x2 + 2

3 x − 2) (

2713 x − 9·31

132

)+

(−2·32·7

132 x − 2·32·31132

).

Therefore −2·32·7132 x − 2·32·31

132 ∈ I.

139 x2 + 2

3 x − 2 =(−2·32·7

132 x − 2·32·31132

) (−133

2·34·7 x + 132192

2·3472

)+

(132613272

).

Therefore 132613272 ∈ I. And 3272

13261 ·132613272 = 1 therefore 1 ∈ I. Therefore I = C[x]. Therefore the g.c.d. of f (x) and g(x) equals 1:

gcd( f , g) = 1.

Solution 2 (replacing the first polynomial with 2x5 − x3 − 3x2 − 6x + 6).

Now we’ll solve it assuming f (x) = 2x5−x3−3x2−6x+6 which is what I think they really intended. Again let I = 〈 f (x), g(x)〉be the ideal in C[x] generated by f and g. Dividing g into f we get

2x5 − x3 − 3x2 − 6x + 4 = (x4 + x3 − x2 − 2x − 2)(2x − 2) + (3x3 − x2 − 6x + 2). (30)

Thus (3x3 − x2 − 6x + 2) ∈ I. Dividing again

x4 + x3 − x2 − 2x − 2 = (3x3 − x2 − 6x + 2)(

13 x + 4

9

)+ 13

9 (x2 − 2). (31)

Thus x2 − 2 ∈ I. Dividing again we have

3x3 − x2 − 6x + 2 = (x2 − 2)(3x − 1), (32)

and the remainder is zero. Now we solve back, substituting (32) into (31) gives

x4 + x3 − x2 − 2x − 2 = (x2 − 2)(3x − 1)(

13 x + 4

9

)+ 13

9 (x2 − 2)

and factoring out x2 − 2 from both terms on the right hand side we get

x4 + x3 − x2 − 2x − 2 = (x2 − 2)(x2 + x + 1). (33)

Thus g ∈ (x2 − 2)C[x]. Now substituting (32) and (33) into (30) gives

2x5 − x3 − 3x2 − 6x + 4 = (x2 − 2)(x2 + x + 1)(2x − 2) + (x2 − 2)(3x − 1)

Page 105: Linear Algebra - Greg Grantgreggrant.org/hoffman_and_kunze.pdf · Linear Algebra Ho man & Kunze 2nd edition ... Section 1.2: Systems of Linear Equations 3 Solution: We must check

Section 4.4: Polynomial Ideals 103

and factoring out x2 − 2 from both terms on the right hand side we get f ∈ (x2 − 2)C[x]. Thus we’ve shown both f and g arein (x2 − 2)C. Since I equals 〈 f , g〉, it follows that I ⊆ (x2 − 2)C[x]. We’ve shown (x2 − 2)C[x] ⊆ I. Thus we can conclude(x2 − 2)C[x] = I. It follows that the g.c.d. of f and g is x2 − 2.

(b) Let f (x) = 3x4 + 8x2 − 3 and g(x) = x3 + 2x2 + 3x + 6. Let I = 〈 f (x), g(x)〉. We have

3x4 + 8x2 − 3 = (x3 + 2x2 + 3x + 6)(3x − 6) + (11x2 + 33)

Thus 11x2 + 33 ∈ I. Thus x2 + 3 ∈ I. Now

x3 + 2x2 + 3x + 6 = (x2 + 3)(x + 2).

Thus x2 + 3 divides both f (x) and g(x). In particular

f (x) = (3x2 − 1)(x2 + 3)

g(x) = (x + 2)(x2 + 3)

and therefore it follows that〈 f (x), g(x)〉 = 〈x2 + 3〉.

(c) As in part (a) I have a feeling there’s a typo and they really meant x3 + 6x2 + 7x + 2 because then both are divisible byx + 1. But as it is, x3 + 6x2 + 7x + 1 is irreducible and the calculations are even worse than part (a). As I did in part (a), I’llsolve this in both the way they actually stated it and the was I think they intended it. First the way they stated it.

Solution 1 (as stated in book).

Let f (x) = x4 − 2x3 − 2x2 − 2x − 3 and g(x) = x3 + 6x2 + 7x + 1 and I = 〈 f (x), g(x)〉.

x4 − 2x3 − 2x2 − 2x − 3 = (x3 + 6x2 + 7x + 1)(x − 8) + (33x2 + 53x + 5)

Thus 33x2 + 53x + 5 ∈ I.

x3 + 6x2 + 7x + 1 = (33x2 + 53x + 5)(

133 x + 145

1089·332

)+

(−2271089 x + 364

1089

).

Thus−2271089 x + 364

1089 ∈ I

And finally33x2 + 53x + 5 =

(−2271089 x + 364

1089

) (−1089·33

227 x − 2618282751529

)+

(900929751529

).

Thus 900929751529 ∈ I and it follows that 1 ∈ I and that gcd( f (x), g(x)) = 1.

Solution 2 (replacing the second polynomial with x3 + 6x2 + 7x + 2).

Exercise 3: Let A be an n×n matrix over a field F. Show that the set of all polynomials f in F[x] such that f (A) = 0 is an ideal.

Solution: Let I be the set { f ∈ F[x] | f (A) = 0}. Let f , g ∈ I and c ∈ F. Then (c f + g)(A) = c f (A) + g(A) = c ·0 + 0 = 0. ThusI is a subspace of F[x] as a vector space over F. Now suppose f ∈ I and g ∈ F[x]. Then (g f )(A) = g(A) f (A) = g(A) · 0 = 0.Thus I has the required property to be an ideal.

Exercise 4: Let F be a subfield of the complex numbers, and let

A =

[1 −20 3

].

Page 106: Linear Algebra - Greg Grantgreggrant.org/hoffman_and_kunze.pdf · Linear Algebra Ho man & Kunze 2nd edition ... Section 1.2: Systems of Linear Equations 3 Solution: We must check

104 Chapter 4: Polynomials

Find the monic generator of the ideal of all polynomials f in F[x] such that f (A) = 0.

Solution: Let I = { f (x) ∈ F[x] | f (A) = 0}. We know from the previous exercise that I is an ideal. It’s clear that f (A) , 0 ifdeg( f ) ≤ 1. Thus if we find any f (x) ∈ I such that deg( f ) = 2 then f must be a generator. Let f (x) = x2 − 4x + 3. Then

f (A) = A2 − 4A + 3

=

[1 −20 3

[1 −20 3

]− 4

[1 −20 3

]+ 3

[1 00 1

]=

[1 −80 9

]−

[4 −80 12

]+

[3 00 3

]=

[0 00 0

].

You will learn an easy way to find this polynomial in Theorem 4 of Section 6.3.

Exercise 5: Let F be a field. Show that the intersection of any number of ideals in F[x] is an ideal.

Solution: Let A be a set. Let Iα be an ideal in F[x] for each α ∈ A. Let I = ∩α∈AIα. By Theorem 2, page 36 (sec. 2.2), I is asubspace of F[x]. We must just show I satisfies the extra condition required to be an ideal. Specifically, let g(x) ∈ F[x] andf (x) ∈ I. We must show g(x) f (x) ∈ I. Since f (x) ∈ I, it follows that f (x) ∈ Iα ∀ α ∈ A. Since Iα is an ideal ∀α ∈ A, it followsthat g(x) f (x) ∈ Iα ∀ α ∈ A. Thus f (x)g(x) ∈ Iα ∀ α ∈ A. Thus f (x)g(x) ∈ I.

Exercise 6: Let F be a field. Show that the ideal generated by a finite number of polynomials f1, . . . , fn in F[x] is the inter-section of all ideals containing f1, . . . , fn.

Solution: Let I = 〈 f1, . . . , fn〉 be the ideal generated by { f1, . . . , fn}. Let J be any ideal containing { f1, . . . , fn}. Let g ∈ I.Then g = g1 f1 + · · · + gn fn for some g1, . . . , gn in F[x]. Since J is an ideal containing { f1, . . . , fn} it must also contain g. Thusg ∈ I ⇒ g ∈ J. Thus I ⊆ J. Now by the definition of intersection, J is contained in every ideal which contains { f1, . . . , fn}.Since I is such an ideal, it follows that J ⊆ I. Thus we’ve shown I ⊆ J and J ⊆ I. Thus J = I.

Exercise 7: Let K be a subfield of a field F, and suppose f , g are polynomials in K[x]. Let MK be the ideal generated by fand g in K[x] and MF be the ideal they generate in F[x]. Show that MK and MF have the same monic generator.

Solution:

Solution 1 We first note a general fact. Let E be any field. Let f , g ∈ E[x] such that gcd( f , g) = k. Let h ∈ E[x] be monic. Weclaim gcd(k f , kg) = kh in E[x]. Since gcd( f , g) = k, the ideal generated by f and g is the same as the ideal generated by k.In other words f E[x] + gE[x] = kE[x]. Thus h f E[x] + hgE[x] = hkE[x]. Since h is monic by assumption and k is monic bydefinition of gcd, it follows that hk is monic - and therefore hk satisifes the definition of gcd( f , g) in E[x].

Now let h = gcd( f , g) in K[x]. So h ∈ K[x] and h = f a + gb for some a, b ∈ K[x]. And ∃ u, v ∈ K[x] such that f = uhand g = vh. Once you can write the three equations h = f a + gb, f = uh and g = vh it follows that the ideal in K[x]generated by f and g is the same as the ideal generated by h. That’s what it means for h to be the g.c.d. of f and g. But thesethree equalities also hold in F[x] and consequently the ideal in F[x] generated by f and g is the same as the one generated by h.

Solution 2 This might be easier if they had formalized what is discussed informally on page 132 after the proof of Theorem 7.What they are describing is the so-called “eucidean algorithm”. If you have two polynomials f and g and say deg( f ) ≥ deg(g).Then you divide g into f and take the remainder r ∈ K[x]. That remainder is also in the ideal generated f and g. Then replacef and g with g and r and do the same thing, divide r into g and the remainder has degree less than r. Eventually you mustarrive at a remainder equal to zero or of degree equal to zero (i.e. a scalar). If the remainder is zero then the previous remaindergenerates the ideal 〈 f , g〉 in K[x]. If the remainder is a non-zero scalar then the ideal contains 1 and therefore 〈 f , g〉 = K[x].

Page 107: Linear Algebra - Greg Grantgreggrant.org/hoffman_and_kunze.pdf · Linear Algebra Ho man & Kunze 2nd edition ... Section 1.2: Systems of Linear Equations 3 Solution: We must check

Section 4.5: The Prime Factorization of a Polynomial 105

The euclidean algorithm allows us to always find the gcd in a finite number of steps.

Once we have the euclidean algorithm, all we need to do to solve this problem is to note that all operations in the euclideanalgorithm happen in K[x] and so if K ⊆ F then in F[x] the same operations will lead to the same generator which willtherefore still live in the smaller ring K[x].

Section 4.5: The Prime Factorization of a PolynomialPage 137: In the proof of Theorem 11, they definitely use the product and chain rules for derivatives. I don’t believe thosehave ever been proven or even stated. The product rule was actually proven as part of Exercise 9 part (e), from Section 4.2,page 123.

Page 139: In Exercise 7 it says “Use Exercise 7”. They meant to say “Use Exercise 6”.

Exercise 1: Let p be a monic polynomial over the field F, and let f and g be relatively prime polynomials ovef F. Prove thatthe g.c.d. of p f and pg is p.

Solution: I proved this in more generality as part of Exercise 7 of the previous section. That makes me think that was not thesolution they were looking for. Or maybe they want a proof here using prime factorization. Here are both proofs.

Solution 1 We use the definition of g.c.d. in terms of ideals. Since gcd( f , g) = 1, the ideal generated by f and g is the sameas the ideal generated by 1. In other words f F[x] + gF[x] = F[x]. Thus p f F[x] + pgF[x] = pF[x]. Since p is monic byassumption, it follows that p satisifes the definition of gcd( f , g) in F[x].

Solution 2 We use the comments at the top of page 137. Factor into primes f = f r11 · · · f r j

j , g = gs11 · · · g

skk and p = pt1

1 · · · pt``

.Since g.c.d.( f , g) = 1 none of the fi’s equal any of the gi’s. Thus the common factors of p f and pg are exactly the pi’s. Inother words the prime factorizations are

f p = f r11 · · · f r j

j · pt11 · · · p

t``

gp = gs11 · · · g

skk · p

t11 · · · p

t``.

Since none of the fi’s equal any of the gi’s, it follows that g.c.d( f , g) = pt11 · · · p

t``

which equals p.

Exercise 2: Assuming the Fundamental Theorem of Algebra prove the following. If f and g are polynomials over the fieldof complex numbers, then g.c.d( f , g) = 1 if and only if f and g have no common root.

Solution: By the Fundamental Theorem of Algebra, we can factor f and g all the way to linear factors

f = (x − a1)n1 · · · (x − ak)nk

g = (x − b1)m1 · · · (x − b`)m` .

The roots of f are exactly a1, . . . , ak, the roots of g are exactly b1, . . . , b`. If gcd( f , g) = 1 then (by the comments at the topof page 137) f and g have no common factors, and therefore ai , b j ∀ i, j. Thus (by Corollary 1, page 128) f and g have nocommon roots. And if f and g have no common roots then (by Corollary 1, page 128) none of the factors (x − ai) can equalany of the factors (x − b j). Thus gcd( f , g) = 1.

Exercise 3: Let D be the differentiation operator on the space of polynomials over the field of complex numbers. Let f be amonic polynomial over the field of complex numbers. Prove that

f = (x − c1) · · · (x − ck)

Page 108: Linear Algebra - Greg Grantgreggrant.org/hoffman_and_kunze.pdf · Linear Algebra Ho man & Kunze 2nd edition ... Section 1.2: Systems of Linear Equations 3 Solution: We must check

106 Chapter 4: Polynomials

where c1, . . . , ck are distinct complex numbers if and only if f and D f are relatively prime. In other words, f has no repeatedroots if and only if f and D f have no common root. (Assume the Fundamental Theorem of Algebra.)

Solution: First assume all ci’s are distinct. Then by Theorem 11, page 137, we know f and D f are relatively prime.

Now assume f and f ′ are relatively prime. Then again by Theorem 11 we know f is a product of distinct irreducibles.Since C is algebraically closed each of those irreducibles must be of the form x − a. It follows that f must be of the form(x − c1) · · · (x − cn) for distinct c1, . . . , cn.

Exercise 4: Prove the following generalization of Taylor’s formula. Let f , g, and h be polynomials over a subfield of thecomplex numbers, with deg f ≤ n. Then

f (g) =

n∑k=0

1k!

f (k)(h)(g − h)k.

(Here f (g) denotes ‘ f of g.’)

Solution: Since we are working over the base field C by Theorem 3 page 126 there is a natural isomorphism between thealgebra C[x] and the algebra of polynomial functions from C into C. Taylor’s Theorem is therefore also a theorem aboutpolynomial functions from C to C. So we may plug any complex numbers we want in for x and c in the statement of Taylor’sTheorem and the equality remains valid. In particular we can subsitute g in for x and h in for c to obtain the desired formula.We then simply translate over to the algebra C[x] via the isomorphism in Theorem 3 to obtain the corresponding result in C[x].

For the remaining exercises, we shall need the following definition. If f , g, and p are polynomials over the field F with p , 0,we say f is congruent to g modulo p if ( f − g) is divisible by p. If f is congruent to g modulo p, we write

f ≡ g (mod p).

Exercise 5: Prove for any non-zero polynomial p, that congruence modulo p is an equivalence relation.

(a) It is reflexive: f ≡ f (mod p).

(a) It is symmetric: if f ≡ g (mod p), then g ≡ f (mod p).

(a) It is transitive: if f ≡ g (mod p) and g ≡ h (mod p), then f ≡ h (mod p).

Solution:

(a). In this case p must divide f − f which equals zero. But everything divides zero. Just take d = 0 and then p = d( f − f ).Thus f ≡ f (mod p) is always true.

(b). Assume f ≡ g (mod p). Then p divides f − g. Thus ∃ d s.t. f − g = pd. Let d′ = −d. Then g − f = pd′ thus p dividesg − f . Thus g ≡ f (mod p).

(c). Assume f ≡ g (mod p) and g ≡ h (mod p). Then ∃ d and d′ such that f − g = pd and g − h = pd′. Adding these twoequations gives f − h = pd + pd′. Let d′′ = d + d′. Then f − h = pd′′. Thus f ≡ h (mod p).

Exercise 6: Suppose f ≡ g (mod p) and f1 ≡ g1 (mod p).

(a) Prove that f + f1 ≡ g + g1 (mod p).

(b) Prove that f f1 ≡ gg1 (mod p).

Solution:

Page 109: Linear Algebra - Greg Grantgreggrant.org/hoffman_and_kunze.pdf · Linear Algebra Ho man & Kunze 2nd edition ... Section 1.2: Systems of Linear Equations 3 Solution: We must check

Section 4.5: The Prime Factorization of a Polynomial 107

(a) By assumption there are polynomials d and d′ such that f − g = pd and f1 − g1 = pd′. Adding these two equations givesf + f1 − g − g1 = pd + pd′. Or equivalently ( f + f1) − (g + g1) = p(d + d′). Thus f + f1 ≡ g + g1 (mod p).

(b) By assumption there are polynomials d and d′ such that f − g = pd and f1 − g1 = pd′. Now f f1 − gg1 = f f1 − g1 f + g1 f −gg1 = f ( f1 − g1) + g1( f − g) = f pd′ + g1 pd = p( f d′ + g1d). Thus p divides f f1 − gg1. Thus f f1 ≡ gg1 (mod p).

Exercise 7: Use Exercise 6 to prove the following. If f , g, and p are polynomials over the field F and p , 0, and if f ≡ g(mod p), then h( f ) ≡ h(g) (mod p).

Solution: It follows from Exercise 6 part (b) that f ≡ g (mod p) ⇒ f 2 ≡ g2 (mod p) (since f ≡ f (mod p) and g ≡ g(mod p)). By induction f n ≡ gn (mod p) for all n = 1, 2, 3, . . . . Let c ∈ F. Then letting f1 = g1 = c we get c f ≡ cg (mod p)for any c ∈ F. Thus if h(x) = cxn we have shown the result is true. Thus the result is true for all monomials. Now we canobtain the result on the sum of monomials using part (a) of Exercise 6. Since the general h is a sum of monomials, the generalresult follows.

Exercise 8: If p is an irreducible polynomial and f g ≡ 0 (mod p), prove that either f ≡ 0 (mod p) or g ≡ 0 (mod p). Givean example which shows that this is false if p is not irreducible.

Solution: f g ≡ 0 (mod p) implies p divides f g. By the Corollary to Theorem 8, page 135, it follows that p divides f of pdivides g. Thus f ≡ 0 (mod p) or g ≡ 0 (mod p).

Page 110: Linear Algebra - Greg Grantgreggrant.org/hoffman_and_kunze.pdf · Linear Algebra Ho man & Kunze 2nd edition ... Section 1.2: Systems of Linear Equations 3 Solution: We must check

108 Chapter 4: Polynomials

Page 111: Linear Algebra - Greg Grantgreggrant.org/hoffman_and_kunze.pdf · Linear Algebra Ho man & Kunze 2nd edition ... Section 1.2: Systems of Linear Equations 3 Solution: We must check

Chapter 5: Determinants

Section 5.2: Determinant FunctionsExercise 1: Each of the following expressions defines a function D on the set of 3×3 matricces over the field of real numbers.In which of these cases is D a 3-linear function?

(a) D(A) = A11 + A22 + A33;

(b) D(A) = (A11)2 + 3A11A22;

(c) D(A) = A11A12A33;

(d) D(A) = A13A22A32 + 5A12A22A32;

(e) D(A) = 0;

(f) D(A) = 1;

Solution:

(a) No D is not 3-linear. Let

A =

2 0 00 1 00 0 1

.Then if D were 3-linear then it would be linear in the first row and we’d have to have D(A) = D(I) + D(I). But D(A) = 4 andD(I) = 3, so D(A) , D(I) + D(I).

(b) No D is not 3-linear. Let A be the same matrix as in part (a). Then D(A) = 10 and D(I) = 4, so D(A) , D(I) + D(I).

(c) No D is not 3-linear. Let

A =

2 2 00 0 00 0 1

,B =

1 1 00 0 00 0 1

.Then if D were 3-linear we’d have to have D(A) = D(B) + D(B). But D(A) = 4 and D(B) = 1. Thus D(A) , D(B) + D(B).

(d) Yes D is 3-linear. The two functions A 7→ A13A22A32 and A 7→ 5A12A22A32 are both 3-linear by Example 1, page 142. Thesum of these two functions is then 3-linear by the Lemma on page 143. Since D is exactly the sum of these two functions, itfollows that D is 3-linear.

109

Page 112: Linear Algebra - Greg Grantgreggrant.org/hoffman_and_kunze.pdf · Linear Algebra Ho man & Kunze 2nd edition ... Section 1.2: Systems of Linear Equations 3 Solution: We must check

110 Chapter 5: Determinants

(e) Yes D is 3-linear. We must show (5-1) on page 142 holds for all matrices A. But since D(A) = 0 ∀ A, both sides of (5-1)are always equal to zero. Thus (5-1) does hold ∀ A.

(f) No D is not 3-linear. Let A be the matrix from part (a) again. Then D(A) = 1 but D(I)+D(I) = 2. Thus D(A) , D(I)+D(I).Thus D is not 3-linear.

Exercise 2: Verify directly that the three functions E1, E2, E3 defined by (5-6), (5-7), and (5-8) are identical.

Solution:

E1(A) = A11(A22A33 − A23A32) − A21(A12A33 − A13A32) + A31(A12A23 − A13A22)

= A11A22A33term 1

− A11A23A32term 2

− A21A12A33term 3

+ A21A13A32term 4

+ A31A12A23term 5

− A31A13A22term 6

.

E2(A) = −A12(A21A33 − A23A31) + A22(A11A33 − A13A31) − A32(A11A23 − A13A21)

= −A12A21A33term 3

+ A12A23A31term 5

+ A22A11A33term 1

− A22A13A31term 6

− A32A11A23term 2

+ A32A13A21term 4

.

E3(A) = A13(A21A32 − A22A31) − A23(A11A32 − A12A31) + A33(A11A22 − A12A21)

= A13A21A32term 4

− A13A22A31term 6

− A23A11A32term 2

+ A23A12A31term 5

+ A33A11A22term 1

− A33A12A21term 3

.

I’ve expanded the three expressions and labelled corresponding terms. We see each of the six terms appears exactly once in

each expansion, and always with the same sign. Therefore the three expressions are equal.

Exercise 3: Let K be a commutative ring with identity. If A is a 2 × 2 matrix over K, the classical adjoint of A is the 2 × 2matrix adj A defined by

adj A =

[A22 −A12−A21 A11

].

If det denotes the unique determinant function on 2 × 2 matrices over K, show that

(a) (adj A)A = A(adj A) = (det A)I;

(a) det(adj A) = det(A);

(a) adj (At) = (adj A)t.

(At denotes the transpose of A.)

Solution:

(a)

(adj A)A =

[A22 −A12−A21 A11

] [A11 A12A21 A22

]=

[A11A22 − A12A21 A12A22 − A12A22−A11A21 + A11A21 −A12A21 + A11A22

]=

[A11A22 − A12A21 0

0 A11A22 − A12A21

]

Page 113: Linear Algebra - Greg Grantgreggrant.org/hoffman_and_kunze.pdf · Linear Algebra Ho man & Kunze 2nd edition ... Section 1.2: Systems of Linear Equations 3 Solution: We must check

Section 5.2: Determinant Functions 111

=

[det(A) 0

0 det(A)

].

A(adj A) =

[A11 A12A21 A22

] [A22 −A12−A21 A11

]=

[A11A22 − A12A21 −A11A12 + A12A11A21A22 − A22A21 −A21A12 + A22A11

]=

[det(A) 0

0 det(A)

].

Thus both (adj A)A and A(adj A) equal (det A)I.

(b)

det(adj A) = det([

A22 −A12−A21 A11

])= A11A22 − A12A21

= det(A).

(c)

adj(At) = adj([

A11 A12A21 A22

]t)= adj

([A11 A21A12 A22

])=

[A22 −A21−A12 A11

](34)

And

(adjA)t =

(adj

[A11 A12A21 A22

])t

=

[A22 −A12−A21 A11

]t

=

[A22 −A21−A12 A11

](35)

Comparing (34) and (35) gives the result.

Exercise 4: Let A be a 2 × 2 matrix over a field F. Show that A is invertible if and only if det A , 0. When A is invertible,give a formula for A−1.

Solution: We showed in Example 3, page 143, that det(A) = A11A22 − A12A21. Therefore, we’ve already done the first part inExercise 8 of section 1.6 (page 27). We just need a formula for A−1. The formula is

A−1 =1

det(A)

[A22 −A12−A21 A11

].

Checking:

A ·1

det(A)

[A22 −A12−A21 A11

]=

1det(A)

[A11 A12A21 A22

] [A22 −A12−A21 A11

].

Page 114: Linear Algebra - Greg Grantgreggrant.org/hoffman_and_kunze.pdf · Linear Algebra Ho man & Kunze 2nd edition ... Section 1.2: Systems of Linear Equations 3 Solution: We must check

112 Chapter 5: Determinants

=1

det(A)

[A11A22 − A12A21 −A11A12 + A12A11A21A22 − A22A21 −A21A12 + A22A11

]=

1det(A)

[det(A) 0

0 det(A)

]=

[1 00 1

]Exercise 5: Let A be a 2 × 2 matrix over a field F, and suppose that A2 = 0. Show for each scalar c that det(cI − A) = c2.

Solution: One has to be careful in proving this not to use implications such as 2x = 0⇒ x = 0; or x2 + y = 0⇒ y = 0. Theseimplications are not valid in a general field. However, we will need to use that fact that xy = 0⇒ x = 0 or y = 0, which istrue in any field.

Let A =

[x yz w

]. Then

A2 =

[x2 + yz xy + ywxz + wz yz + w2

].

If A2 = 0 thenx2 + yz = 0 (36)

y(x + w) = 0 (37)

z(x + w) = 0 (38)

yz + w2 = 0. (39)

Now det(cI − A) = det[

c − x −y−z c − w

]= (c − x)(c − w) − yz = c2 − c(x + w) + xw − yz.

Thusdet(cI − A) = c2 − c(x + w) + det(A). (40)

Suppose x + w , 0. Then (37) and (38) imply y = z = 0. Thus A =

[x 00 w

]. But then A2 =

[x2 00 w2

]. So if A2 = 0 then

it must be that also x = w = 0, which contradicts the assumption that x + w , 0.

Thus necessarily A2 = 0 implies x + w = 0. This implies A =

[x yz −x

]. Thus det(A) = −x2 − yz, which equals zero by (36).

Thus A2 = 0 implies x + w = 0 and det(A) = 0. Thus by (40) A2 = 0 implies det(cI − A) = c2.

Exercise 6: Let K be a subfield of the complex numbers and n a positive integer. Let j1, . . . , jn and k1, . . . , kn be positiveintegers not exceeding n. For an n × n matrix A over K define

D(A) = A( j1, k1)A( j2, k2) · · · A( jn, kn).

Prove that D is n-linear if and only if the integers j1, . . . , jn are distinct.

Solution: First assume the integers j1, . . . , jn are distinct. Since these n integers all satisfy 1 ≤ ji ≤ n, their being distinct nec-essarily implies { j1, . . . , jn} = {1, 2, 3, . . . , n} Thus A( j1, k1)A( j2, k2) · · · A( jn, kn) is just a rearrangement of A(1, k1)A(2, k2) · · · A(3, kn).It follows from Example 1 on page 142 that A( j1, k1)A( j2, k2) · · · A( jn, kn) is n-linear.

Now assume two or more of the ji’s are equal. Assume without loss of generality that j1 = j2 = · · · = j` = 1 where ` ≥ 2.Let A be the matrix with all 2’s in the first row and all ones in all other rows. Let B be the matrix of all 1’s. Then D(A) = 2`

and D(B) = 1. Since D is n-linear it must be that D(A) = D(B) + D(B). But ` > 1⇒ 2` , 2. Thus D(A) , D(B) + D(B) and

Page 115: Linear Algebra - Greg Grantgreggrant.org/hoffman_and_kunze.pdf · Linear Algebra Ho man & Kunze 2nd edition ... Section 1.2: Systems of Linear Equations 3 Solution: We must check

Section 5.2: Determinant Functions 113

D is not n-linear.

Exercise 7: Let K be a commutative ring with identity. Show that the determinant function on 2 × 2 matrices A over K isalternating and 2-linear as a function of the columns of A.

Solution:det

[ra1 + a2 brc1 + c2 d

]= (ra1 + a2)d − (rc1 + c2)b

= ra1d + a2d − rc1b − c2b

= r(ad − bc1) + (a2d − bc2)

= r det[

a1 bc1 d

]+ det

[a2 bc2 d

].

Likewise

det[

a rb1 + b2c rd1 + d2

]= a(rd1 + d2) − (rb1 + b2)c

= rad1 + ad2 − rcb1 − cb2

= (rad1 − rcb1) + (ad2 − cb2)

= r det[

a b1c d1

]+ det

[a b2c d2

].

Thus the determinant function is 2-linear on columns. Now

det[

a bc d

]= ad − bc

= −(bc − ad)

= − det[

b da c

].

Thus the determinant function is alternating on columns.

Exercise 8: Let K be a commutative ring with identity. Define a function D on 3 × 3 matrices over K by the rule

D(A) = A11 det[

A22 A23A32 A33

]− A12

[A21 A23A31 A33

]+ A13

[A21 A22A31 A32

].

Show that D is alternating and 3-linear as a function of the columns of A.

Solution: This is exactly Theorem 1 page 146 but with respect to columns instead of rows. The statement and proof gothrough without change except for chaning the word “row” to “column” everywhere. To make it work, however, we mustknow that det is an alternating 2-linear function on columns of 2 × 2 matrices over K. This is exactly what was shown in theprevious exercise.

Exercise 9: Let K be a commutative ring with identity and D and alternating n-linear function on n × n matrices over K.Show that

(a) D(A) = 0, if one of the rows of A is 0.

Page 116: Linear Algebra - Greg Grantgreggrant.org/hoffman_and_kunze.pdf · Linear Algebra Ho man & Kunze 2nd edition ... Section 1.2: Systems of Linear Equations 3 Solution: We must check

114 Chapter 5: Determinants

(b) D(B) = D(A), if B is obtained from A by adding a scalar multiple of one row of A to another.

Solution: Let A be an n×n matrix with one row all zeros. Suppose row αi is all zeros. Then αi +αi = αi. Thus by the linearityof the determinant in the ith row we have det(A) = det(A) + det(A). Subtracting det(A) from both sides gives det(A) = 0.

Now suppose B is obtained from A by adding a scalar multiple of one row to another. Assume row βi of B equals αi + cα j

where αi is the ith row of A and α j is the jth. Then the rows of B are α1, . . . , αi−1, αi + cα j, αi+1, . . . , αn. Thus

det(B) = det(α1, . . . , αi−1, αi + cα j, αi+1, . . . , αn)

= det(α1, . . . , αi−1, αi + cα j, αi+1, . . . , αn)

= det(α1, . . . , αi−1, αi, αi+1, . . . , αn) + c · det(α1, . . . , αi−1, α j, αi+1, . . . , αn).

The first determinant is det(A). And by the first part of this problem, the second determinant equals zero, since it has arepeated row α j. Thus det(B) = det(A).

Exercise 10: Let F be a field, A a 2 × 3 matrix over F, and (c1, c2, c3) the vector in F3 defined by

c1 =

∣∣∣∣∣∣ A12 A13A22 A23

∣∣∣∣∣∣ , c2 =

∣∣∣∣∣∣ A13 A11A23 A21

∣∣∣∣∣∣ , c3 =

∣∣∣∣∣∣ A11 A12A21 A22

∣∣∣∣∣∣ .Show that

(a) rank(A) = 2 if and only if (c1, c2, c3) , 0;

(b) if A has rank 2, then (c1, c2, c3) is a basis for the solution space of the system of equations AX = 0.

Solution: We will use the fact that the rank of a 2 × 2 matrix is 2⇔ the matrix is invertible⇔ the determinant is non-zero.The first equivalence follows from the fact that a matrix M with rank 2 has two linearly independent rows and therefore therow space of M is all of F2 which is the same as the row space of the identity matrix. Thus by the Corollary on page 58 M isrow-equivalent to the identity matrix, thus by Theorem 12 (page 23) it follows that M is invertible. The second equivalencefollows from Exercise 4 from Section 5.2 (page 149).

(a) If rank(A) = 0 then A is the zero matrix and clearly c1 = c2 = c3 = 0.

If rank(A) = 1 then the second row must be a multiple of the first row. This is then true for each of the 2 × 2 matrices[A12 A13A22 A23

],

[A13 A11A23 A21

],

[A11 A12A21 A22

](41)

because each one is obtained from A by deleting one column (and in the case of the second one, switching the two remainingcolumns). Thus each of them has rank ≤ 1. Therefore the determinant of each of these three matrices is zero. Thus (c1, c2, c3)is the zero vector.

If rank(A) = 2 then the second row of A is not a multiple of its first row. We must show the same is true of at least one of thematrices in (41). Suppose the second row is a multiple of the first for each matrix in (41). Since each pair of these matricesshares a column, it must be the same multiple for each pair; and therefore the same multiple for all three, call it c. Thereforethe seond row of the entire matrix A is c times the first row, which contradicts our assumption that rank(A) = 2. Thus at leastone of the matrices in (41) must have rank two and the result follow.

(b) Identify F3 with the space of 3×1 column vectors and F2 the space of 2×1 column vectors. Let T : F3 → F2 be the lineartransformation given by T X = AX. Then by Theorem 2 page 71 (the rank/nullity theorem) we know rank(T ) + nullity(T ) = 3.It was shown in the proof of Theorem 3 page 72 (the third displayed equation in the proof) that rank(T ) = column rank(A).And nullity(T ) is the solution space for AX = 0. Thus column rank(A) plus the rank of the solution space of AX = 0 equals

Page 117: Linear Algebra - Greg Grantgreggrant.org/hoffman_and_kunze.pdf · Linear Algebra Ho man & Kunze 2nd edition ... Section 1.2: Systems of Linear Equations 3 Solution: We must check

Section 5.3: Permutations and the Uniqueness of Determinants 115

three. Thus if rank(A) = 2 then the rank of the solution space of AX = 0 must equal one. Thus a basis for this space is anynon-zero vector in the space. Thus we only need show (c1, c2, c3) is in this space. In other words we must show[

A11 A12 A13A21 A22 A23

] c1c2c3

= 0.

It feels like we’re supposed to apply Exercise 8 to the following matrix A11 A12 A13A11 A12 A13A21 A22 A23

,but the problem with that is that we do not know that an alternating function on columns is necessarily zero on a matrix witha repeated row. That is true, but rather than prove it, it’s easier just prove this directly

c1 = A12A23 − A22A13

c1 = A13A21 − A11A23

c1 = A11A22 − A12A21.

Therefore [A11 A12 A13A21 A22 A23

] c1c2c3

=

[A11c1 + A12c2 + A13c3A21c1 + A22c2 + A23c3

]Expanding the first entry

A11c1 + A12c2 + A13c3

= A11(A12A23 − A22A13) + A12(A13A21 − A11A23) + A13(A11A22 − A12A21)

= A11A12A23 − A11A22A13 − A12A13A21 − A12A11A23 + A13A11A22 − A13A12A21

matching up terms we see everything cancels.

= A11A12A23term 1

− A11A22A13term 2

− A12A13A21term 3

− A12A11A23term 1

+ A13A11A22term 2

− A13A12A21term 3

= 0.

Expanding the second entryA21c1 + A22c2 + A23c3

= A21(A12A23 − A22A13) + A22(A13A21 − A11A23) + A23(A11A22 − A12A21)

= A21A12A23 − A21A22A13 + A22A13A21 − A22A11A23 + A23A11A22 − A23A12A21

matching up terms we see everything cancels.

= A21A12A23term 1

− A21A22A13term 2

+ A22A13A21term 2

− A22A11A23term 3

+ A23A11A22term 3

− A23A12A21term 1

= 0.

Section 5.3: Permutations and the Uniqueness of DeterminantsExercise 9: Let n be a positive integer and F a field. If σ is a permutation of degree n, prove that the function

T (x1, . . . , xn) = (xσ1, . . . , xσn)

is an invertible linear operator on Fn.

Solution:

Page 118: Linear Algebra - Greg Grantgreggrant.org/hoffman_and_kunze.pdf · Linear Algebra Ho man & Kunze 2nd edition ... Section 1.2: Systems of Linear Equations 3 Solution: We must check

116 Chapter 5: Determinants

Page 119: Linear Algebra - Greg Grantgreggrant.org/hoffman_and_kunze.pdf · Linear Algebra Ho man & Kunze 2nd edition ... Section 1.2: Systems of Linear Equations 3 Solution: We must check

Chapter 6: Elementary Canonical Forms

Section 6.2: Characteristic ValuesExercise 1: In each of the following cases, let T be the linear operator on R2 which is represented by the matrix A inthe standard ordered basis for R2, and let U be the linear operator on C2 represented by A in the standard ordered basis.Find the characyteristic polynomial for T and that for U, find the characteristic values of each operator, and for each suchcharactersistic value c find a basis for the corresponding space of characteristic vectors.

A =

[1 00 0

],

[2 3−1 1

],

[1 11 1

].

Solution:

A =

[1 00 0

]

xI − A =

[x − 1 0

0 x

]The characteristic polynomial equals |xI − A| = x(x − 1). So c1 = 0, c1 = 1. A basis for W1 is {(0, 1)}, α1 = (0, 1). A basisfor W2 is {(1, 0)}, α2 = (1, 0). This is the same whether the base field is R or C since the characteristic polynomial factorscompletely.

A =

[2 3−1 1

]

|xI − A| =

∣∣∣∣∣∣ x − 2 −31 x − 1

∣∣∣∣∣∣= (x − 2)(x − 1) + 3 = x2 − 3x + 5. This is a parabola opening up with vertex (3/2, 11/4). Thus there are no real roots. Usingthe quadratic formula c1 = 3+

√11i

2 and c2 = 3−√

11i2 . To find the a characteristic vector for c1 we solve −1+

√11i

2 3−1 1+

√11i

2

[ xy

]=

[00

].

This gives the characteristic vector α1 = ( 1+√

11i2 , 1). To find the a characteristic vector for c2 we solve −1−

√11i

2 3−1 1−

√11i

2

[ xy

]=

[00

].

This gives the characteristic vector α2 = ( 1−√

11i2 , 1).

117

Page 120: Linear Algebra - Greg Grantgreggrant.org/hoffman_and_kunze.pdf · Linear Algebra Ho man & Kunze 2nd edition ... Section 1.2: Systems of Linear Equations 3 Solution: We must check

118 Chapter 6: Elementary Canonical Forms

A =

[1 11 1

]

|xI − A| =

∣∣∣∣∣∣ x − 1 11 x − 1

∣∣∣∣∣∣ = (x − 1)2 − 1 = x(x − 2). So c1 = 0 for which α1 = (1,−1). And c2 = 2 for which α2 = (1, 1).

This is the same in both R and C since the characteristic polynomial factors completely.

Exercise 2: Let F be an n-dimensional vector space over F. What is the characteristic polynomial of the identity operator onV? What is the characteristic polynomial for the zero operator?

Solution: The identity operator can be represented by the n × n identity matrix I. The characteristic polynomial of the iden-tity operator is therefore (x − 1)n. The zero operator is represented by the zero matrix in any basis. Thus the characteristicpolynomial of the zero operator is xn.

Exercise 3: Let A be an n × n triangular matrix over the field F. Prove that the characteristic values of A are the diagonalentries of A, i.e., the scalars Aii.

Solution: The determinant of a triangular matrix is the product of the diagonal entries. Thus |xI − A| =∏

(x − aii).

Exercise 4: Let T be the linear operator of R3 which is represented in the standard ordered basis by the matrix −9 4 4−8 3 4−16 8 7

.Prove that T is diagonalizable by exhibiting a basis for R3, each vector fo which is a characteristic vector of T .

Solution:

|xI − A| =

∣∣∣∣∣∣∣∣x + 9 −4 −4

8 x − 3 −416 −8 x − 7

∣∣∣∣∣∣∣∣=

∣∣∣∣∣∣∣∣x + 9 0 −4

8 x + 1 −416 −x − 1 x − 7

∣∣∣∣∣∣∣∣= (x + 1)

∣∣∣∣∣∣∣∣x + 9 0 −4

8 1 −416 −1 x − 7

∣∣∣∣∣∣∣∣= (x + 1)

∣∣∣∣∣∣∣∣x + 9 0 −4

8 1 −424 0 x − 11

∣∣∣∣∣∣∣∣= (x + 1)

∣∣∣∣∣∣ x + 9 −424 x − 11

∣∣∣∣∣∣= (x + 1)[(x + 9)(x − 11) + 96] = (x + 1)(x2 − 2x − 3) = (x + 1)(x − 3)(x + 1) = (x + 1)2(x − 3). Thus c1 = −1, c2 = 3. For c1,xI − A equals

=

8 −4 −48 −4 −4

16 −8 −8

Page 121: Linear Algebra - Greg Grantgreggrant.org/hoffman_and_kunze.pdf · Linear Algebra Ho man & Kunze 2nd edition ... Section 1.2: Systems of Linear Equations 3 Solution: We must check

Section 6.2: Characteristic Values 119

This matrix evidently has rank one. Thus the null space has rank two. The two characteristic vectors (1, 2, 0) and (1, 0, 2) areindependent, so they form a basis for W1. For c2, xI − A equals

=

−12 −4 −48 0 −4

16 −8 −4

This is row equivalent to

=

1 −0 −1/20 1 −1/20 0 0

Thus the null space one dimensional and is given by (z/2, z/2, z). So (1, 1, 2) is a characteristic vector and a basis for W2. ByTheorem 2 (ii) T is diagonalizable.

Exercise 5: Let 6 −3 −24 −1 −210 −5 −3

.Is A similar over the field R to a diagonal matrix? Is A similar over the field C to a diagonal matrix?

Solution:

=

∣∣∣∣∣∣∣∣x − 6 3 2−4 x + 1 2−10 5 x + 3

∣∣∣∣∣∣∣∣=

∣∣∣∣∣∣∣∣x − 6 3 2−x + 2 x − 2 0−10 5 x + 3

∣∣∣∣∣∣∣∣= (x − 2)

∣∣∣∣∣∣∣∣x − 6 3 2−1 1 0−10 5 x + 3

∣∣∣∣∣∣∣∣= (x − 2)

∣∣∣∣∣∣∣∣x − 3 3 2

0 1 0−5 5 x + 3

∣∣∣∣∣∣∣∣= (x−2)((x−3)(x+3)+10) = (x−2)(x2 +1). Since this is not a product of linear factors over R, by Theorem 2, page 187, A isnot diagonalizable over R. Over C this factors to (x−2)(x− i)(x + i). Thus over C the matrix A has three distinct characteristicvalues. The space of characteristic vectors for a given characteristic value has dimension at least one. Thus the sum of thedimensions of the Wi’s must be at least n. It cannot be greater than n so it must equal n exactly. Thus A is diagonalizable overC.

Exercise 6: Let T be the linear operator on R4 which is represented in the standard ordered basis by the matrix0 0 0 0a 0 0 00 b 0 00 0 c 0

.Under what conditions on a, b, and c is T diagonalizable?

Solution:

|xI − A| =

∣∣∣∣∣∣∣∣∣∣∣x 0 0 0−a x 0 00 −b x 00 0 −c x

∣∣∣∣∣∣∣∣∣∣∣

Page 122: Linear Algebra - Greg Grantgreggrant.org/hoffman_and_kunze.pdf · Linear Algebra Ho man & Kunze 2nd edition ... Section 1.2: Systems of Linear Equations 3 Solution: We must check

120 Chapter 6: Elementary Canonical Forms

= x4. Therefore there is only one characteristic value c1 = 0. Thus c1I − A = A and W1 is the null space of A. So A isdiagonalizable⇔ dim(W) = 4⇔ A is the zero matrix⇔ a = b = c = 0.

Exercise 7: Let T be the linear operator on the n-dimensional vector space V , and suppose that T has n distinct characteristicvalues. Prove that T is diagonalizable.

Solution: The space of characteristic vectors for a given characteristic value has dimension at least one. Thus the sum of thedimensions of the Wi’s must be at least n. It cannot be greater than n so it must equal n exactly. Thus by Theorem 2, T isdiagonalizable.

Exercise 8: Let A and B be n × n matrices over the field F. Prove that if (I − AB) is invertible, then I − BA is invertible and

(I − BA)−1 = I + B(I − AB)−1A.

Solution:(I − BA)(I + B(I − AB)−1A)

= I − BA + B(I − AB)−1A − BAB(I − AB)−1A

= I − B(A − (I − AB)−1A + AB(I − AB)−1A)

= I − B(I − (I − AB)−1 + AB(I − AB)−1A

= I − B(I − (I − AB)(I − AB)−1)A

= I − B(I − I)A

= I.

Exercise 9: Use the result of Exercise 8 to prove that, if A and B are n × n matrices over the field F, then AB and BA haveprecisely the same characteristic values in F.

Solution: By Theorem 3, page 154, det(AB) = det(A) det(B). Thus AB is singular⇔ BA is singular. Therefore 0 is a charac-teristic values of AB⇔ 0 is a characteristic value of BA. Now suppose the characteristic value c of AB is not equal to zero.

Then |cI − AB| = 0⇔ cn|I − 1c AB| = 0

by #8⇔ cn|I − 1

c BA| = 0⇔ |cI − BA| = 0.

Exercise 10: Suppose that A is a 2× 2 matrix with real entries which is symmetrix (At = A). Prove that A is similar over R toa diagonal matrix.

Solution: A =

[a bc d

]. So |xI − A| =

∣∣∣∣∣∣ x − a −b−b x − a

∣∣∣∣∣∣ = (x − a)2 − b2 = (x − a − b)(x − a + b). So c1 = a + b, c2 = a = b.

If b = 0 then A is already diagonal. If b , 0 then c1 , c2 so by Exercise 7 A is diagonalizable.

Exercise 11: Let N be a 2 × 2 complex matrix such that N2 = 0. Prove that either N = 0 or N is similar over C to[0 01 0

].

Solution: Suppose N =

[a bc d

]. Now N2 = 0 ⇒

[ac

],[

bd

]are characteristic vectors for the characteristic value 0.

If[

ac

],[

bd

]are linearly independent then W1 has rank two and N is diagonalizable to

[0 00 0

]. If PNP−1 = 0 then

N = P−10P = 0 so in this case N itself is the zero matrix. This contradicts the assumption that[

ac

],[

bd

]are linearly

independent.

Page 123: Linear Algebra - Greg Grantgreggrant.org/hoffman_and_kunze.pdf · Linear Algebra Ho man & Kunze 2nd edition ... Section 1.2: Systems of Linear Equations 3 Solution: We must check

Section 6.2: Characteristic Values 121

So we can assume that[

ac

],[

bd

]are linearly dependent. If both equal the zero vector then N = 0. So we can assume at least

one vector is non-zero. If[

bd

]is the zero vector then N =

[a 0c 0

]. So N2 = 0⇒ a2 = 0→ a = 0. Thus N =

[a 0c 0

]. In

this case N is similar to N =

[0 01 0

]via the matrix P =

[c 00 1

]. Similary if

[ac

]is the zero vector, then N2 = 0 implies

d2 = 0 implies d = 0 so N =

[0 b0 0

]. In this case N is similar to N =

[0 0b 0

]via the matrix P =

[0 11 0

], which is

simiilar to[

0 01 0

]as above.

By the above we can assume neither[

ac

]or

[bd

]is the zero vector. Since they are linearly dependent we can assume[

bd

]= x

[ac

]so N =

[a axc cx

]. So N2 = 0 implies

a(a + cx) = 0

c(a + cx) = 0

ax(a + cx) = 0

cx(a + cx) = 0.

We know that at least one of a or c is not zero. If a = 0 then since c , 0 it must be that x = 0. So in this case N =

[0 0c 0

]which is similar to

[0 01 0

]as before. If a , 0 then x , 0 else a(a + cx) = 0 implies a = 0. Thus a + cx = 0 so

N =

[a ax−a/x −a

]. This is similar to

[a a−a −a

]via P =

[ √x 0

0 1/√

x

]. And

[a a−a −a

]is similar to

[0 0−a 0

]via

P =

[−1 −11 0

]. And this finally is similar to

[0 01 0

]as before.

Exercise 12: Use the result of Exercise 11 to prove the following: If A is a 2×2 matrix with complex entries, then A is similarover C to a matrix of one of the two types [

a 00 b

] [a 01 a

].

Solution: Suppose A =

[a bc d

]. Since the base field is C the characteristic polynomial p(x) = (x − c1)(x − c2). If c1 , c2

then A is diagonalizable by Exercise 7. If c1 = c2 then p(x) = (x − c1)2. If W has dimension two then A is diagonalizable by

Theorem 2. Thus we will be done if we show that if p(x) = (x − c1)2 and dim(W1) = 1 then A is similar to[

a 01 a

].

We will need the following three identities:[a 0c d

]∼

[a 01 d

]via p =

[c 00 1

](42)

[a bc d

]∼

[a − b c − d

b d

]via p =

[1 0−1 1

](43)

[a bc d

]∼

[a xb

c/x d

]via p =

[ √x 0

0 1/√

x

]for x , 0. (44)

Page 124: Linear Algebra - Greg Grantgreggrant.org/hoffman_and_kunze.pdf · Linear Algebra Ho man & Kunze 2nd edition ... Section 1.2: Systems of Linear Equations 3 Solution: We must check

122 Chapter 6: Elementary Canonical Forms

Now we know in this case that A is not diagonalizable. If d , 0 then[

a bc d

]∼

[a bc/dd d

]by (44) with x = c/d and this

in turn is similar to[

a − bc/d 0−a + 2bc/d d

]by (43).

Now we know the diagonal entries are the characteristic values, which are equal. Thus a − bcd = d. So this equals

[d 0x d

]where x = −a + 2bc

d and we know x , 0 since A is not diagonalizable. Thus A ∼[

d 01 d

]by (42). Now suppose d = 0. Then

A =

[a bc 0

]∼

[0 cb a

]via p =

[0 11 0

]. If b = 0 then A =

[a 0c 0

]and again since A has equal characteristic values it

must be that a = 0. So A =

[0 0c 0

]which is similar to

[0 01 0

]via P =

[c 00 1

]. So assume b , 0. Then A ∼

[0 cb a

]and we can argue exact as above were d , 0.

Exercise 13: Let V be the vector space of all functions from R into R which are continuous, i.e., the space of continuousreal-valued functions on the real line. Let T be the linear operator on V defined by

(T f )(x) =

∫ x

0f (t)dt.

Prove that T has no characteristic values.

Solution: Suppose ∃ c such that T f = c f ∀ f . Then∫ x

0 f (t)dt = c f (x). Let f (x) = 1. Then we must have∫ x

0 1dt = c · 1,which imples x = c. In other words this implies the functions f (x) = x and g(x) = c are the same, which they are not becauseone is constant and the other is not. This therefore is a contradiction. Thus it is impossible that ∃ c such that T f = c f ∀ f .Thus T has no characteristic values.

Exercise 14: Let A be an n × n diagonal matrix with characteristic polynomial

(x − c1)d1 · · · (x − ck)dk ,

where c1, . . . , ck are distinct. Let V be the space of n × n matrices B such that AB = BA. Prove that the dimension of V isd2

1 + · · · + d2k .

Solution: Write

A =

c1I

c2I 00 . . .

ckI

.Write

B =

B11 B12 · · · B1k

B21 B22 · · · B2k...

.... . .

...Bk1 Bk2 · · · Bkk

where Bi j has dimenson di × d j. Then AB = BA implies

c1B11 c1B12 · · · c1B1k

c2B21 c2B22 · · · c2B2k...

.... . .

...ckBkk ckBk2 · · · ckBkk

=

c1B11 c2B12 · · · ckB1k

c1B21 c2B22 · · · ckB2k...

.... . .

...c1Bk1 c2Bk2 · · · ckBkk

Page 125: Linear Algebra - Greg Grantgreggrant.org/hoffman_and_kunze.pdf · Linear Algebra Ho man & Kunze 2nd edition ... Section 1.2: Systems of Linear Equations 3 Solution: We must check

Section 6.3: Annihilating Polynomials 123

Thus ci , c j for i , j implies Bi j = 0 for i , j, while B11, B22, . . . , Bkk can be arbitrary. The dimension of Bii is therefore d2i

thus the dimension of the space of all such Bii’s is d21 + d2

2 + · · · + d2k .

Exercise 15: Let V be the space of n × n matrices over F. Let A be a fixed n × n matrix over F. Let T be the linear operator‘left multiplication by A’ on V . Is it true that A and T have the same characteristic values?

Solution: Yes. Represent an element of V as a column vector by stacking the columns of V on top of each other, with the

first column on top. Then the matrix for T is given by

A

A 00 . . .

A

. By the argument on page 157 the determinant of

this matrix is det(A)n. Thus if p is the characteristic polynomial of A then pn is the characteristic polynomial of T . Thus theyhave exactly the same roots and thus they have exactly the same characteristic values.

Section 6.3: Annihilating PolynomialsPage 198: Typo in Exercise 11, “Section 6.1” should be “Section 6.2”.

Exercise 1: Let V be a finite-dimensional vector space. What is the minimal polynomial for the identity operator on V? Whatis the minimal polynomial for the zero operator?

Solution: The minimal polynomial for the identity operator is x − 1. It annihilates the identity operator and the monic zerodegree polynomial p(x) = 1 does not, so it must be the minimal polynomial. The minimal polynomial for the zero operator isx. It is a monic polynomial that annihilates the zero operator and again the monic zero degree polynomial p(x) = 1 does not,so it must be the minimal polynomial.

Exercise 2: Let a, b and c be tlements of a field F, and let A be the following 3 × 3 matrix over F:

A =

0 0 c1 0 b0 1 a

.Prove that the characteristic polynomial for A is xx − ax2 − bx − c and that this is also the minimal polynomial for A.

Solution: The characteristic polynomial is∣∣∣∣∣∣∣∣x 0 −c−1 x −b0 −1 x − a

∣∣∣∣∣∣∣∣ =

∣∣∣∣∣∣∣∣x 0 −c−1 0 x2 − ax − b0 −1 x − a

∣∣∣∣∣∣∣∣ = 1 ·

∣∣∣∣∣∣ x −c−1 x2 − ax − b

∣∣∣∣∣∣ = x3 − ax2 − bx − c.

Now for any r, s ∈ F

A2 + rA + s =

0 c ac0 b c + ba1 a b + a2

+

0 0 rcr 0 rb0 r ra

+

s 0 00 s 00 0 s

=

s c ac + rcr b + s c + ba + br1 a + r b + a2 + ra + s

, 0.

Thus f (A) , 0 for all f ∈ F[x] such that deg(F) = 2. Thus the minimum polynomial cannot have degree two, it must thereforehave degree three. Since it divides x3 − ax2 − bx − c it must equal x3 − ax2 − bx − c.

Page 126: Linear Algebra - Greg Grantgreggrant.org/hoffman_and_kunze.pdf · Linear Algebra Ho man & Kunze 2nd edition ... Section 1.2: Systems of Linear Equations 3 Solution: We must check

124 Chapter 6: Elementary Canonical Forms

Exercise 3: Let A be the 4 × 4 real matrix 1 1 0 0−1 −1 0 0−2 −2 2 11 1 −1 0

.Show that the characteristic polynomial for A is x2(x − 1)2 and that it is also the minimal polynomial.

Solution: The characteristic polynomial equals∣∣∣∣∣∣∣∣∣∣∣x − 1 −1 0 0

1 x + 1 0 02 2 x − 2 −1−1 −1 1 x

∣∣∣∣∣∣∣∣∣∣∣ =

∣∣∣∣∣∣ x − 1 −11 x + 1

∣∣∣∣∣∣ ·∣∣∣∣∣∣ x − 2 −1

1 x

∣∣∣∣∣∣ by (5-20) page 158

= x2(x2 − 2x + 1) = x2(x − 1)2.

The minimum polynomial is clearly not linear, thus the minimal polynomial is one of x2(x−1)2, x2(x−1), x(x−1)2 or x(x−1).We will plug A in to the first three and show it is not zero. It will follow that the minimum polynomial must be x2(x − 1)2.

A2 =

0 0 0 00 0 0 0−3 −3 3 22 2 −2 −1

A − I =

0 1 0 0−1 −2 0 0−2 −2 1 11 1 −1 −1

and

(A − I)2 =

−1 −2 0 02 3 0 01 1 0 00 0 0 0

Thus

A2(A − I) =

0 0 0 00 0 0 0−1 −1 1 11 1 −1 −1

, 0

A(A − I)2 =

1 1 0 0−1 −1 0 00 0 0 00 0 0 0

, 0

and

A(A − I) =

−1 −1 0 01 1 0 0−1 −1 1 11 1 −2 −2

, 0.

Thus the minimal polynomial must be x2(x − 1)2.

Exercise 4: Is the matrix A of Exercise 3 similar over the field of complex numbers to a diagonal matrix?

Page 127: Linear Algebra - Greg Grantgreggrant.org/hoffman_and_kunze.pdf · Linear Algebra Ho man & Kunze 2nd edition ... Section 1.2: Systems of Linear Equations 3 Solution: We must check

Section 6.3: Annihilating Polynomials 125

Solution: Not diagonalizable, because for characteristic value c = 0 the matrix A − cI = A and A is row equivalent to1 1 0 00 0 1 00 0 0 10 0 0 0

which has rank three. So the null space has dimension one. So if W is the null space for A − cI then W has dimension one,which is less than the power of x in the characteristic polynomial. So by Theorem 2, page 187, A is not diagonalizable.

Exercise 5: Let V be an n-dimensional vector space and let T be a linear operator on V . Suppose that there exists somepositive integer k so that T k = 0. Prove tht T n = 0.

Solution: T k = 0⇒ the only characteristic value is zero. We know the minimal polynomial divides this so the minimal poly-nomial is of the form tr for some 1 ≤ r ≤ n. Thus by Theorem 3, page 193, the characteristic polynomial’s only root is zero,and the characteristic polynomial has degree n. So the characteristic polynomial equals tn. By Theorem 4 (Caley-Hamilton)T n = 0.

Exercise 6: Find a 3 × 3 matrix for which the minimal polynomial is x2.

Solution: If A2 = 0 and A , 0 then the minimal polynomial is x or x2. So any A , 0 such that A2 = 0 has minimal polynomialx2. E.g.

A =

0 0 01 0 00 0 0

.Exercise 7: Let n be a positive integer, and let V be the space of polynomials over R which have degree at most n (throw inthe 0-polynomial). Let D be the differentiation operator on V . What is the minimal polynomial for D?

Solution: 1, x, x2, . . . , xn is a basis.1 7→ 0

x 7→ 1

x2 7→ 2x

...

xn 7→ nxn−1

The matrix for D is therefore

0 1 0 0 · · · 00 0 2 0 · · · 00 0 0 3 · · · 0...

......

.... . .

...0 0 0 0 · · · n

Suppose A is a matrix such that ai j = 0 except when j = i + 1. Then A2 has ai j = 0 except when j = i + 2. A3 has ai j = 0except when j = i + 3. Etc., where finally An = 0. Thus if ai j , 0 ∀ j = i + 1 then Ak , 0 for k < n and An = 0. Thus theminimum polynomial divides xn and cannot be xk for k < n. Thus the minimum polynomial is xn.

Exercise 8: Let P be the operator on R2 which projects each vector onto the x-axis, parallel to the y-axis: P(x, y) = (x, 0).Show that P is linear. What is the minimal polynomial for P?

Page 128: Linear Algebra - Greg Grantgreggrant.org/hoffman_and_kunze.pdf · Linear Algebra Ho man & Kunze 2nd edition ... Section 1.2: Systems of Linear Equations 3 Solution: We must check

126 Chapter 6: Elementary Canonical Forms

Solution: P can be given in the standard basis by left multiplication by A =

[1 00 0

]. Since P is given by left multiplication

by a matrix, P is clearly linear. Since A is diagonal, the characteristic values are the diagonal values. Thus the characteristicvalues of A are 0 and 1. The characteristic polynomial is a degree two monic polynomial for which both 0 and 1 are roots.Therefore the characteristic polynomial is x(x − 1). If the characteristic polynomial is a product of distinct linear terms thenit must equal the minimal polynomial. Thus the minimal polynomial is also x(x − 1).

Exercise 9: Let A be an n × n matrix with characteristic polynomial

f = (x − c1)d1 · · · (x − ck)dk .

Show thatc1d1 + · · · + ckdk = trace(A).

Solution: Suppose A is n × n. Claim: |xI − A| = xn + trace(A)xn−1 + · · · . Proof by induction: case n = 2. A =

[a bc d

].

|xI − A| = x2 + (a + d)x + (ad − bc). The trace of A is a + d so we have established the claim for the case n = 2. Suppose truefor up to n − 1. Let r = a22 + a33 + · · · + ann. Then∣∣∣∣∣∣∣∣∣∣∣∣

x − a11 a12 · · · a1n

a21 x − a22 · · · a2n. . .

. . . · · ·. . .

an1 an2 · · · x − ann

∣∣∣∣∣∣∣∣∣∣∣∣Now expanding by minors using the first column, and using induction, we get that this equals

(x − a11)(xn−1 − rxn−2 + · · · )

−a21(polynomial of degree n − 2)

+a31(polynomial of degree n − 2)

+ · · ·

= xn + (r + a11)xn−1 + polynomial of degree at most n − 2

= xn − tr(A)xn−1 + · · ·

Now if f (x) = (x − c1)d1 · · · (x − ck)dk then the coefficient of xn−1 is c1d1 + · · · ckdk so it must be that c1d1 + · · · ckdk = tr(A).

Exercise 10: Let V be the vector space of n × n matrices over the field F. Let A be a fixed n × n matrix. Let T be the linearoperator on V defined by

T (B) = AB.

Show that the minimal polynomial for T is the minimal polynnomial for A.

Solution: If we represent a n × n matrix as a column vector by stacking the columns of the matrix on top of each other, withthe first column on the top, then the transformation T is represented in the standard basis by the matrix

M =

A

A 00 . . .

A

.

Page 129: Linear Algebra - Greg Grantgreggrant.org/hoffman_and_kunze.pdf · Linear Algebra Ho man & Kunze 2nd edition ... Section 1.2: Systems of Linear Equations 3 Solution: We must check

Section 6.4: Invariant Subspaces 127

And since

f (M) =

f (A)

f (A) 00 . . .

f (A)

it is evident that f (M) = 0⇔ f (A) = 0.

Exercise 11: Let A and B be n × n matrices over the field F. According to Exercise 9 of Section 6.2, the matrices AB andBA have the same characteristic values. Do they have the same characteristic polynomial? Do they have the same minimalpolynomial?

Solution: In Exercise 9 Section 6.2 we showed |xI = AB| = 0 ⇔ |xI − BA| = 0. Thus we have two monic polynomials ofdegree n with exactly the same roots. Thuse they are equal. So the characteristic polynomials are equal. But the minimum

polynomials need not be equal. To see this let A =

[0 01 0

]and B =

[1 00 0

]. Then AB =

[0 01 0

]and BA =

[0 00 0

]so

the minimal polynomial of BA is x and the minimal polynomial of AB is clearly not x (it is in fact x2).

Section 6.4: Invariant Subspaces

Exercise 1: Let T be the linear operator on R2, the matrix of which in the standard ordered basis is

A =

[1 −12 2

].

(a) Prove that the only subspaces of R2 invariant under T are R2 and the zero subspace.(b) If U is the linear operator on C2, the matrix of which in the standard ordered basis is A, show that U has 1-dimensional

invariant subspaces.

Solution: (a) The charactersistic polynomial equals

∣∣∣∣∣∣ x − 1 1−2 x − 2

∣∣∣∣∣∣ = (x−1)(x−2)+2 = x2−3x+4. This is a parabola open-

ing upwards with vertex (3/2, 7/4), so it has no real roots. If T had an invariant subspace it would have to be 1-dimensionaland T would therefore have a characteristic value.

(b) Over C the characteristic polylnomial factors into two linears. Therefore over C, T has two characteristic values andtherefore has at least one characteristic vector. The subspace generated by a characteristic vector is a 1-dimensional subspace.

Exercise 2: Let W be an invariant subspace for T . Prove that the minimal polynomial for the restriction operator TW dividesthe minimal polynomial for T , without referring to matrices.

Solution: The minimum polynomial of TW divides any polynomial f (t) where f (TW ) = 0. If f is the minimum polynomialfor T then F(T )v = 0 ∀ v ∈ V . Therefore, f (T )w = 0 ∀ w ∈ W. So f (TW )w = 0 ∀ w ∈ W since by definition f (TW )w = f (T )wfor w ∈ W. Therefore, f (TW ) = 0. Therefore the minimum polynomial for TW divides f .

Exercise 3: Let c be a characteristic value of T and let W be the space of characteristic vectors associated with the character-istic value c. What is the restriction operator TW?

Solution: For w ∈ W the transformation T (w) = cw. Thus TW is diagonalizable with single characteristic value c. In other

Page 130: Linear Algebra - Greg Grantgreggrant.org/hoffman_and_kunze.pdf · Linear Algebra Ho man & Kunze 2nd edition ... Section 1.2: Systems of Linear Equations 3 Solution: We must check

128 Chapter 6: Elementary Canonical Forms

words under which it is represented by the matrix c

c 00 . . .

c

where there are dim(W) c’s on the diagonal.

Exercise 4: Let

A =

0 1 02 −2 22 −3 2

.Is A similar over the field of real numbers to a triangular matrix? If so, find such a triangular matrix.

Solution:

A2 =

2 −2 20 0 0−2 2 −2

.And A3 = 0. Thus the minimal polynomial x3 and the only characteristic value is 0. We now follow the constructive proof

of Theorem 5. W = {0}, α1 a characteristic vector of A is

10−1

. We need α2 such that Aα2 ∈ {α1}. α2 =

−112

satisfies

Aα2 = α1. Now need α3 such that Aα3 ∈ {α1, α2}. α3 =

001

satisfies Aα3 = 2α1 + 2α2. Thus with respect to the basis

{α1, α2, α3} the transformation corresponding to A is

0 1 20 0 20 0 0

.Exercise 5: Every matrix A such that A2 = A is siimilar to a diagonal matrix.

Solution: A2 = A⇒ A satisfies the polynomial x2− x = x(x−1). Therefore the minimum polynomial of A is either x, x−1 orx(x−1). In all three cases the minimum polynomial factors into distinct linears. Therefore, by Theorem 6 A is diagonalizable.

Exercise 6: Let T be a diagonalizable linear opeartor on the n-dimensional vector space V , and let W be a subspace which isinvariant under T . Prove that the restriction operator TW is diagonalizable.

Solution: By the lemma on page 80 the minimum polynomial for TW divides the minimum polynomial for T . Now T di-agonalizable implies (by Theorem 6) that the minimum polynomial for T factors into distinct linears. Since the minimumpolynomial for TW divides it, it must also factor into distinct linears. Thus by Theorem 6 again TW is diagonalizable.

Exercise 7: Let T be a linear operator on a finite-dimensional vector space over the field of complex numbers. Prove that Tis diagonalizable if and only if T is annihilated by some polynomial over C which has distinct roots.

Solution: If T is diagonalizable then its minimum polynomial is a product of distinct linear factors, and the minimal poly-nomial annihilates T . This proves “⇒”. Now suppose T is annihilated by a polynomial over C with distinct roots. Sincethe base field is C this polynomial factors completely into distinct linear factors. Since the minimum polynomial divides thispolynomial the minimum polynomial factors completely into distinct linear factors. Thus by Theorem 6, T is diagonalizable.

Page 131: Linear Algebra - Greg Grantgreggrant.org/hoffman_and_kunze.pdf · Linear Algebra Ho man & Kunze 2nd edition ... Section 1.2: Systems of Linear Equations 3 Solution: We must check

Section 6.4: Invariant Subspaces 129

Exercise 8: Let T be a linear operator on V . If every subspace of V is invariant under T , then T is a scalar multiple of theidentity operator.

Solution: Let {αi} be a basis. The subspace generated by αi is invariant thus Tαi is a multiple of αi. Thus αi is a characteristicvector since Tαi = ciαi for some ci. Suppose ∃ i, j such that ci , c j. Then T (αi + α j) = Tαi + Tα j = ciαi + c jα j = c(αi + α j).Since the subspace generated by {αi, α j} is invariant under T . Thus ci = c and c j = c since coefficients of linear combinationsof basis vectors are unique. Thus Tαi = cαi ∀ i. Thus T is c times the identity operator.

Exercise 9: Let T be the indefinite inntegral operator

(T f )(x) =

∫ x

0f (t)dt

on the space of continuous functions on the interval [0, 1]. Is the space of polynomial functions invariant under T? Ths spaceof differentiable functions? The space of functions which vanish at x = 1/2?

Solution: The integral from 0 to x of a polynomial is again a polynomial, so the space of polynomial functions is invariantunder T . The integral from 0 to x of a differntiable function is differentiable, so the space of differentiable functions is invari-ant under T . Now let f (x) = x − 1/2. Then f vanishes at 1/2 but

∫ x0 f (t)dt = 1

2 x2 − 12 x which does not vanish at x = 1/2. So

the space of functions which vanish at x = 1/2 is not invariant under T .

Exercise 10: Let A be a 3 × 3 matrix with real entries. Prove that, if A is not similar over R to a triangular matrix, then A issimilar over C to a diagonal matrix.

Solution: If A is not similar to a tirangular matrix then the minimum polynomial of A must be of the form (x− c)(x2 + ax + b)where x2 + ax + b has no real roots. The roots of x2 + ax + b are then two non-real complex conjugates z and z. Thus over Cthe minimum polynomial factors as (x− c)(x− z)(x− z). Since c is real, c, z and z constintute three distinct numbers. Thus byTheorem 6 A is diagonalizable over C.

Exercise 11: True or false? If the triangular matrix A is similar to a diagonal matrix, then A is already diagonal.

Solution: False. Let A =

[1 10 0

]. Then A is triangular and not diagonal. The characteristic polynomial is x(x − 1) which

has distinct roots, so the minimum polynomial is x(x − 1). Thus by Theorem 6, A is diagonalizable.

Exercise 12: Let T be a linear operator on a finite-dimensional vector space over an algebraically closed field F. Let f be apolynomial over F. Prove that c is a characteristic value of f (T ) if and only if c = f (t), where t is a characteristic value of T .

Solution: Since F is algebraically closed, the corollary at the bottom of page 203 implies there’s a basis under which T isrepresented by a triangular matrix A. A = [ai j] where ai j = 0 if i > j and the aii, i = 1, . . . , n are the characteristic values of T .Now f (A) = [bi j] where bi j = 0 if i > j and bii = f (aii) for all i = 1, . . . , n. Thus the characteristic values of f (A) are exactlythe f (c)’s where c is a characteristic value of A. Since f (A) is a matrix representative of f (T ) in the same basis, we concludethe same thing about the tranformation T .

Exercise 13: Let V be the space of n × n matrices over F. Let A be a fixed n × n matrix over F. Let T and U be the linearoperators on V defined by

T (B) = AB

U(B) = AB − BA

(a) True or false? If A is a diagonalizable (over F), then T is diagonalizable.(b) True or false? If A is diagonalizable, then U is diagonalizable.

Page 132: Linear Algebra - Greg Grantgreggrant.org/hoffman_and_kunze.pdf · Linear Algebra Ho man & Kunze 2nd edition ... Section 1.2: Systems of Linear Equations 3 Solution: We must check

130 Chapter 6: Elementary Canonical Forms

Solution: (a) True by Exercise 10 Section 6.3 page 198 since by Theorem 6 diagonalizability depends entirely on the mini-mum polynomial.

(b) True. Find a basis so that A is diagonal

A =

c1

c2 00 . . .

cn

.Let B = [bi j]. Then U(B) = AB − BA. The n2 matrices Bi j such that bi j , 0 and all other entries equal zero form a basis forV . For any Bi j, U(Bi j) = ABi j − Bi jA = [di′ j′ ] where di′ j′ = ci′bi′ j′ − c j′bi′ j′ = (ci′ − c j′ )bi′ j′ . Thus di′ j′ , 0 only when i′ = iand j′ = j. Thus U(Bi j) = (ci − c j)Bi j. So ci − c j is a characteristic value and Bi j is a characteristic vector for all i, j. Thus Vhas a basis of characteristic vectors for U. Thus U is diagonalizable.

Section 6.5: Simultaneous Triangulation; Simultaneous DiagonliazationExercise 1: Find an invertible real matrix P such that P−1AP and P−1BP are both diagonal, where A and B are the real matrices

(a) A =

[1 20 2

], B =

[3 −80 −1

](b) A =

[1 11 1

], B =

[1 aa 1

].

Solution: The proof of Theorem 8 shows that if a 2 × 2 matrix has two characteristic values then the P that diagonalizes Awill necessarily also diagonalize any B that commutes with A.

(a) Characteristic polynomial equals (x − 1)(x − 2). So c1 = 1, c2 = 2.

c1 :[

0 −20 −2

] [10

]=

[00

]c2 :

[1 −20 0

] [21

]=

[00

]So P =

[0 20 1

]and P−1 =

[1 −20 1

].

P−1AP =

[1 00 2

]P−1BP =

[3 00 −1

](b) Characteristic polynomial equals x(x − 2). So c1 = 0, c2 = 2.

c1 :[−1 −1−1 −1

] [−11

]=

[00

]c2 :

[1 −1−1 1

] [11

]=

[00

]So P =

[−1 11 1

]and P−1 =

[−1/2 1/21/2 1/2

].

P−1AP =

[0 00 2

]

Page 133: Linear Algebra - Greg Grantgreggrant.org/hoffman_and_kunze.pdf · Linear Algebra Ho man & Kunze 2nd edition ... Section 1.2: Systems of Linear Equations 3 Solution: We must check

Section 6.5: Simultaneous Triangulation; Simultaneous Diagonliazation 131

P−1BP =

[1 − a 0

0 1 + a

]Exercise 2: Let F be a commuting family of 3 × 3 complex matrices. How many linearly independent matrices can Fcontain? What about the n × n case?

Solution: This turns out to be quite a hard question, so I’m not sure what Hoffman & Kunze had in mind. But there’s a generaltheorem from 1905 by I. Schur which says the answer is

⌊n2

4

⌋+1. A simpler proof was published in 1998 by M. Mirzakhani in

the American Mathematical Monthly. I have extracted the proof for the case n = 3 (which required also extracting the prooffor the case n = 2).

First we show:

The maximum size of a set of linearly independent commuting triangulizable 2 × 2 matrices is two (∗)

Suppose that {A1, A2, A3} are three linearly independent commuting upper-triangular 2 × 2 matrices. Let V be the space gen-erated by {A1, A2, A3}. So dim(V) = 3.

Write Ai =[

Ni0 Mi

]where Ni is 1 × 2. Since dim(V) = 3 it cannot be that all three Mi’s are zero. Assume WLOG that M1 , 0.

Then M2 = c2M1 and M3 = c3M1 for some constants c1, c2. Let B2 = A2 − c2A1 and B3 = A3 = c3A1. Then {B2, B3} arelineraly independent in V .

Write B2 =[

t20 0

]and B3 =

[t3

0 0

]where {t2, t3} are linearly independent 1 × 2 matrices.

Similarly ∃ B′2 and B′3 in V such that B′2 =[00 t′2

], B′3 =

[00 t′3

], where {t′2, t

′3} are linearly independent.

Since B2, B3, B′2, B′3 are all in V , they all commute with each other. Thus tit′j = 0 ∀ i, j.

Let A be the 2 × 2 matrix[t3t4

]. Then rank(A) = 2 but At′2 = 0 and At′3 = 0 thus null(A) = 2. Therefore rank(A) + null(A) = 4.

But rank(A) + null(A) cannot be greater than dim(V) = 2. This contradiction imples we cannot have {A1, A2, A3} all three becommuting linearly independent upper-triangular 2×2 matrices. But we know we can have two commuting linearly indepen-

dent upper-triangular 2 × 2 matrices because[

1 00 0

],

[0 00 1

]are such a pair.

We now turn to the case n = 3. Suppose F is a commuting family of linearly independent 3 × 3 matrices with |F | = 4. Weknow ∃ P such that P−1F P is a family of upper tirangular commuting matrices. Let V be the space generated by F . Thendim(V) = 4. Let A1, A2, A3, A4 be a linearly independent subset of V . For each i ∃ a 2 × 2 matrix Mi and a 1 × 3 matrix Ni

such that

A =

Ni

000

Mi

Since the Ai’s commute, for 1 ≤ i, j ≤ 4 we have MiM j = M jMi. Suppose W is the vector space spanned by the set{M1,M2,M3,M4} and let k = dim(W). We know by (∗) that k ≤ 2. Since {A1, A2, A3, A4} are independent we also know k ≥ 1.

First assume k = 1. Then WLOG assume M1 generates W. Then for i = 2, 3, 4 ∃ ni such that Mi = niM1. For i = 2, 3, 4 defineBi = Ai − niA1. Since {A1, A2, A3, A4} are linearly independent, {B2, B3, B4} are linearly independent and Bi =

[ti0

]where ti is

1 × 3 and {t2, t3, t4} are linearly independent.

Now assume k = 2. Then WLOG assume M1,M2 generate W. Then for each i = 3, 4 ∃ ni1, ni2 such that Mi = ni1M1 + ni2M2.For i = 3, 4 define Bi = Ai−ni1A1−ni2A2. Then {A1, A2, A3, A4} linearly independent implies {B3, B4} are linearly independentand for i = 3, 4, Bi =

[ti0

]where ti is 1 × n. Since {B3, B4} are linearly independent, {t3, t4} are linearly independent.

Page 134: Linear Algebra - Greg Grantgreggrant.org/hoffman_and_kunze.pdf · Linear Algebra Ho man & Kunze 2nd edition ... Section 1.2: Systems of Linear Equations 3 Solution: We must check

132 Chapter 6: Elementary Canonical Forms

Thus in both cases (k = 1, 2) we have produced a set of 4 − k linearly independent 1 × n matrices {ti} such that Bi =[ti0

].

By a similar argument we obtian a set of two or three linearly independent n × 1 matrices {t′3, t′4} or {t′2, t

′3, t′4} such that

B′i = [0 | t′i ] is a matrix in V .

Now since all Bi’s and B′i’s all belong to the commuting family V , one sees that tit′j = 0 ∀ i, j.

Let A be the m × 4 matrix (m = 2 or 3) such that its ith row is ti Since the ti’s are independent we have rank(A) ≥ m ≥ 2.On the other hand At′j = 0 for all j and the t′j’s are linearly independent. Thus the null space of A has rank greater or equalto the numnber of t′j’s. Thus rank(A) ≥ 2 and null(A) ≥ 2. But since A is 3 × 3 we know that rank(A) + null(A) = 3. Thiscontradiction implies the set {A1, A2, A3, A4} cannot be linearly independent.

Now we can achieve three independent such matrices because

1 0 00 1 00 0 0

, 0 0 0

0 1 00 0 0

and

0 0 00 0 00 0 1

are such a triple.

Exercise 3: Let T be a linear operator on an n-dimensional space, and suppose that T has n distinct characteristic values.Prove that any linear operator which commutes with T is a polynomial in T .

Solution: Since T has n distinct characteristic values, T is diagonalizable (exercise 6.2.7, page 190). Choose a basis B forwhich T is represented by a diagonal matrix A. Suppose the linear transformation S commutes with T . Let B be the matrixof S in the basis B. Then the i j-th entry of AB is aiibi j and the i j-th entry of BA is a j jbi j. Therefore if aiibi j = a j jbi j andaii , a j j, then it must be that bi j = 0. So we have shown that B must also be diagonal. So we have to show there exists apolynomial such that f (aii) = bii for all i = 1, . . . , n. By Section 4.3 there exists a polynomial with this property.

Exercise 4: Let A, B, C, and D be n × n complex matrices which commute. Let E be the 2n × 2n matrix

E =

[A BC D

].

Prove that det E = det(AD − BC).

Solution: By the corollary on paeg 203 we know A, B, C, and D are all triangulable. By Theorem 7 page 207 we know theyare simultaneously triangulable. Let P be the matrix that simultaneously triangulates them. Let

M =

[P 00 P

].

Then

M−1 =

[P−1 00 P−1

].

And

M−1EM =

[A′ B′

C′ D′

],

where A′, B′, C′, and D′ are upper triangular. Now det(E) = det(M−1EM). Suppose the result were true for upper triangularmatrices A, B, C, and D. Then det(E) = det(M−1EM) = det(P−1AP · P−1DP− P−1BP · P−1CP) = det(P−1ADP− P−1BCP) =

det(P−1(AD − BC)P) = det(AD − BC).

Thus it suffices to prove the result for upper triangular matrices. So in what follows we drop the primes and assume A, B, C,and D are upper triangular. We proceed by induction. Suppose first that n = 1. Then the theorem is clearly true. Suppose it istrue up to n − 1.

Page 135: Linear Algebra - Greg Grantgreggrant.org/hoffman_and_kunze.pdf · Linear Algebra Ho man & Kunze 2nd edition ... Section 1.2: Systems of Linear Equations 3 Solution: We must check

Section 6.5: Simultaneous Triangulation; Simultaneous Diagonliazation 133

If A, B, C, and D are upper triangular then it is clear that det(AD − BC) =∏n

i=1(aiidii − biicii). So by induction we assumedet(E) =

∏mi=1(aiidii − biicii) whenever E has dimension 2m for m < n (of couse always assuming A, B, C, and D commute).

Now we can write E in the following form for some A′, B′, C′ and D′:

E =

a11 A′ b11 B′

. . .. . .

0 ann 0 bnn

c11 C′ d11 D′

. . .. . .

0 cnn 0 dnn

Expanding E by cofactors of the 1st column gives

det(E) = a11

∣∣∣∣∣∣∣∣∣∣∣∣∣∣∣∣∣∣∣∣∣∣∣∣∣∣∣

a22 A′′ 0 b22 B′′

. . .. . .

0 ann 0 bnn

c22C′ d11 D′

. . .. . .

0 cnn 0 dnn

∣∣∣∣∣∣∣∣∣∣∣∣∣∣∣∣∣∣∣∣∣∣∣∣∣∣∣

+ (−1)nc11

∣∣∣∣∣∣∣∣∣∣∣∣∣∣∣∣∣∣∣∣∣∣∣∣∣∣∣

a22A′ b11 B′

. . .. . .

0 ann 0 bnn

c22 C′′ 0 d22 D′′

. . .. . .

0 cnn 0 dnn

∣∣∣∣∣∣∣∣∣∣∣∣∣∣∣∣∣∣∣∣∣∣∣∣∣∣∣for some A′′, B′′, C′′ and D′′.

The n + 1 column of each of these matrices has only one non-zero element. So we next expand by cofactors of the n + 1-thcolumn of each matrix, which gives

(−1)2na11d11

∣∣∣∣∣∣∣∣∣∣∣∣∣∣∣∣∣∣∣∣∣∣∣∣∣

a22 A′′ b22 B′′

. . .. . .

0 ann 0 bnn

c22 C′′ d22 D′′

. . .. . .

0 cnn 0 dnn

∣∣∣∣∣∣∣∣∣∣∣∣∣∣∣∣∣∣∣∣∣∣∣∣∣+ (−1)2n+1c11b11

∣∣∣∣∣∣∣∣∣∣∣∣∣∣∣∣∣∣∣∣∣∣∣∣∣

a22 A′′ b22 B′′

. . .. . .

0 ann 0 bnn

c22 C′′ d22 D′′

. . .. . .

0 cnn 0 dnn

∣∣∣∣∣∣∣∣∣∣∣∣∣∣∣∣∣∣∣∣∣∣∣∣∣

= (a11d11 − c11b11)

∣∣∣∣∣∣∣∣∣∣∣∣∣∣∣∣∣∣∣∣∣∣∣∣∣

a22 A′′ b22 B′′

. . .. . .

0 ann 0 bnn

c22 C′′ d22 D′′

. . .. . .

0 cnn 0 dnn

∣∣∣∣∣∣∣∣∣∣∣∣∣∣∣∣∣∣∣∣∣∣∣∣∣.

Page 136: Linear Algebra - Greg Grantgreggrant.org/hoffman_and_kunze.pdf · Linear Algebra Ho man & Kunze 2nd edition ... Section 1.2: Systems of Linear Equations 3 Solution: We must check

134 Chapter 6: Elementary Canonical Forms

By induction this is equal to

(a11d11 − c11b11)n∏

i=2

(aiidii − biicii) =

n∏i=1

(aiidii − biicii).

QED

Exercise 5: Let F be a field, n a positive integer, and let V be the space of n × n matrices over F. If A is a fixed n × nmatrix over F, let TA be the linear operator on V defined by TA(B) = AB − BA. Consider the family of linear operators TA

obtained by letting A vary over all diagonal matrices. Prove that the operators in that family are simultaneously diagonalizable.

Solution: If we stack the cloumns of an n × n matrix on top of each other with column one at the top, the matrix of TA in the

standard basis is then given by

A

A. . .

A

. Thus if A is diagonal then TA is diagonalizable.

Now TATB(C) = ABC − ACB − BCA + CBA and TBTA(C) = BAC − BCA − ACB + CAB. Therefore we must show thatBAC + CAB = ABC + CBA. The i, j-th entry of BAC + CAB is ci j(aiibii + a j jb j j). And this is exactly the same as the i, j-thentry of ABC + CBA. Thus TA and TB commute. Thus by Theorem 8 the family can be simultaneously diagonalized.

Section 6.6: Direct-Sum DecompositionsExercise 1: Let V be a finite-dimensional vector space and let W1 be any subspace of V . Prove that there is a subspace W2 ofV such that V = W1 ⊕W2.

Solution:

Exercise 2: Let V be a finite-dimensional vector space and let W1, . . . ,Wk be subspaces of V such that

V = W1 + · · · + Wk and dim(V) = dim(W1) + · · · + dim(Wk).

Prove that V = W1 ⊕ · · · ⊕Wk.

Solution:

Exercise 3: Find a projection E which projects R2 onto the subspace spanned by (1,−1) along the subspace spanned by (1, 2).

Solution:

Exercise 4: If E1 and E2 are projections onto independent subspaces, then E1 + E2 is a projection. True or false?

Solution:

Exercise 5: If E is a projection and f is a polynomial, then f (E) = aI + bE. What are a and b in terms of the coefficents of f ?

Solution:

Exercise 6: True or false? If a diagonalizable operator has only the characteristic values 0 and 1, it is a projection.

Solution:

Page 137: Linear Algebra - Greg Grantgreggrant.org/hoffman_and_kunze.pdf · Linear Algebra Ho man & Kunze 2nd edition ... Section 1.2: Systems of Linear Equations 3 Solution: We must check

Section 6.6: Direct-Sum Decompositions 135

Exercise 7: Prove that if E is the projection on R along N, then (I − E) is the projection on N along R.

Solution:

Exercise 8: Let E1, . . . , Ek be linear operators on the space V such that E1 + · · · + Ek = I.

(a) Prove that if EiE j = 0 for i , j, then E2i = Ei for each i.

(b) In the case k = 2, prove the converse of (a). That is, if E1 + E2 = I and E21 = E1, E2

2 = E2, then E1E2 = 0.

Solution:

Exercise 9: Let V be a real vector space and E an idempotent linear operator on V , i.e., a projection. Prove that (I + E) isinvertible. Find (I + E)−1.

Solution:

Exercise 10: Let F be a subfield of the complex numbers (or, a field of characteristic zero). Let V be a finite-dimensinalvector space over F. Suppose that E1, . . . , Ek are projections of V and that E1 + · · · Ek = I. Prove that EiE j = 0 for i , j(Hint: use the trace function and ask yourself what the trace of a porjection is.)

Solution:

Project 11: Let V be a vector space, let W1, . . . ,Wk be subspace of V , and let

V j = W1 + · · ·W j−1 + W j+1 + · · · + Wk.

Suppose that V = W1 ⊕ · · · ⊕Wk. Prove that the dual space V∗ has the direct-sum decomposition V∗ = V01 ⊕ · · · ⊕ V0

k .

Solution: